You are on page 1of 1226

Absite review

Question 1
• Which portion of the gastrointestinal tract is
most responsible for the absorption of iron?
– A. Stomach
– B. Duodenum
– C. Jejunum
– D. Ileum
– E. Colon
Answer 1
• B. Maximal absorption of iron occurs in the
duodenum
Question 2
• Which portion of the gastrointestinal tract is
most responsible for the absorption of bile
acids and folate?
– A. Stomach
– B. Duodenum
– C. Jejunum
– D. Ileum
– E. Colon
Answer 2
• D. Ileum
Question 3
• Which of the following parts of the duodenum
are not considered to be retroperitoneal?
– A. 1st proximal portion
– B. 2nd (descending)
– C. 3rd (transverse)
– D. All of the above are retroperitoneal
Answer 3
• A. The proximal portion of the first part of the
duodenum is intraperitoneal
Question 4
• All of the following are true regarding small
bowel anatomy and physiology except:
– A. Brunner’s glands produce an alkaline solution
– B. Enterochromaffin cells are involved in 5-
hydroxytryptamine release, APUD , serotonin
production
– C. Gastrin is produced by the parietal cells in the
stomach and are innervated by the vagus
– D. The ileum is around 150 cm in length while the
jejunum is about 100 cm
Answer 4
• C is false.
• Gastrin is produced by antral G cells in the stomach and
occasionally by pancreatic cells
• G cells are innvervated by the vagus.
• Gastrin releasing peptide is released by the post-ganglionic
fibers of the vagus during parasympathetic stimulation
• Bombesin also stimulates gastrin release
• Gastrin stimulates enterochromaffin-like cells to release
histamine
• Gastrin also targets parietal cells by increasing the amount of
histamine-> the parietal cells therefore increase HCl secretion
Answer 4
◦ The other statements are true.
◦ The jejunum is the maximum site of absorption, though
the terminal ileum absorbs B12, bile acids, and folate.
◦ Brunner’s glands produce an alkaline solution
◦ Enterochromaffin cells are involved in 5-
hydroxytryptamine release, APUD , serotonin
production
◦ The ileum is around 150 cm in length while the jejunum
is about 100 cm
Question 5
• A 52 yo who has had a complicated history of
multiple gastric and pancreatic procedures
complains of loose stools that float. Which of the
following is true of this condition?
– A. It can be caused by hyposecretion of gastric acid
– B. You should recommend higher intake of fats to
compensate for the loss
– C. Is a known sequlae of extensive terminal ileum
resection
– D. Pancrease is not effective
Answer 5
• C. Steatorrhea is a known sequlae of terminal
ileum resection. Interruption of bile salt
resorption interferes with micelle formation.
• Gastric hyper (not hypo) secretion of acid
increases intestinal motility and interferes with
fat absorption
• Treatment involves controlling diarrhea (Lomotil),
decrease PO intake (esp fats), Pancrease, and H2
blockers
Question 6
• Which of the following amino acids is a major
metabolic fuel for enterocytes?
– A. Alanine
– B. Glycine
– C. Leucine
– D. Glutamine
Answer 6
• D. Glutamine is a major metabolic fuel for
enterocytes and may play an important role in
modulating the cytokine release from
intestinal lymphocytes
Question 7
• You just performed a gastrectomy on a patient that had
previously been in-house receiving pre-operative TPN @ 90
cc/hr. 4 hours post-op, the patient seems unusually
somnolent, somewhat confused, and diaphoretic. His blood
glucose returns as 35 mg/dL. Which of the following is likely
to be true of this condition?
– A. It is unusual to see symptoms of hypoglycemia when serum glucose
levels are above 25 mg/dL
– B. This was probably caused by stopping the TPN altogether during
the operation
– C. TPN should not be administered intraoperatively as operative stress
puts the patient on TPN at risk for hyperosmolar, nonketotic coma
– D. This is likely due to post-operative dumping syndrome
Answer 7
• B. Most likely, this patient is hypoglycemic because
the TPN was not ran at all during the operation.
• TPN associated hypoglycemia is caused by the sudden
slowing of TPN administration, and is the most
common cause of hypoglycemia other than excessive
insulin administration
• Blood glucose levels less than 50 mg/dL can produce
profound symptoms (dizziness, coma, arrhythmia,
hypotension, diaphoresis)
Answer 7
• It is usually safe to run TPN @ ~ 50 cc/ hr
intraoperatively- it is unlikely that this will lead to
hyperglycemia that will cause nonketotic coma
• 10% dextrose can be given when TPN is to be
stopped
• Weaning from tPN can usually be stopped when
the infusions rates are 50 cc/hr or less
Question 8
• Which of the following is NOT true about pancreatic
exocrine secretions?
– A. The principal cations are sodium and potassium
– B. The concentration of the principal anions (bicarb and
chloride) varies depending on pancreatic stimulation
– C. When the stimulus to secrete fluid is minimal, the
chloride concentration is relatively high
– D. When the stimulus to secrete is maximal, the chloride
concentration is relatively high
Answer 8
• D is incorrect. When the stimulus for the pancreas to
secrete is maximal, the chloride concentration is
relatively low, not high
Pancreatic Exocrine
• The principal cations of pancreatic juice are sodium
and potassium
• The concentration of the principal anions (bicarb
and chloride) varies depending on pancreatic
stimulation
• When the stimulus to secrete fluid is minimal, the
chloride concentration is relatively high and bicarb
is low
• When the secretory stimulus is maximal, the bicarb
concentration rises and chloride falls
Pancreatic Exocrine
•The above is a result of the passive exchange of intraductal
bicarb for interstitial chloride
– This exchange occurs as juice flows through the larger pancreatic ducts on its
way to the duodenum.
– At slower rates, there is more opportunity for exchange to take place and bicarb
to subsequently be lost
– The alkaline juice helps to neutralize gastric acid in the duodenum, thus
providing optimal pH for the activity of pancreatic digestive enzymes.
Question 9
• Which of the following about bacterial sepsis is
NOT true?
– A. Gram positive septicemia usually has a poorer
prognosis than gram negative infection
– B. Lipid A of the LPS endotoxin is an importent
component for virulence
– C. TNF can cause self-stimulation of monocytes and
macrophages to their full state of activation
– D. IL-1 may stimulate T lymphocytes to produce TNF
Answer 9
• A is not true. Gram NEGATIVE septicemia
usually has a poorer prognosis than gram +
infection
Gram Negatives/ TNF
• Gram negative septicemia usually has a poorer
prognosis than gram + infection
• The endotoxin, which represents the LPS outer
membrane of these bacteria usually contains the
lipid A moiety, which is important in virulence
• TNF is produced primarily by T lymphocytes and
macrophages in response to many stimuli, including
IL-1, interferon, LPS, viruses, and BCG
Gram Negative Sepsis/ TNF release
• In gram – infections, a stimulated macrophage
produces TNF.
• TNF can cause self-stimulation of monocytes and
macrophages to their full state of activation
• Large array of cytokine and inflammatory signals
develop
• The paracrine function of TNF is to stimulate
neutrophils to full activation
Question 10
• A 45 yo male underwent extensive adhesiolysis with multiple
serosal tears. The operation took around 4 hours and at the
end of the case, his abdomen was slightly difficult to close. He
developed an ileus post-op, a wound dehiscence requiring
takeback, and then on day 9 started draining succus from his
midline excision. Which of the following is not true?
– A. Colonic fistulas are more likely to close compared to SB.
– B. High output fistula are more likely to be from the proximal bowel
and less likely to close with conservative management.
– C. 80% close spontaneously
– D. Iatrogenic enterocutaneous fistulas are more common than
spontaneous fistulas from inflammatory bowel disease
Answer 10
• C. Is not true. 40% of fistulas close spontaneously
• Colonic fistulas are more likely to close compared to SB.
• High output fistula are more likely to be from the proximal
bowel and less likely to close with conservative
management.
• Most fistulas are iatrogenic
• Tx initially with NPO, bowel rest, TPN, octreotide, output
management
• Best surgical option is takedown and resection
Answer 10
• FRIENDS
• Foreign body
• Radiation
• IBD
• Epithelization
• Neoplasm
• Distal obstruction
• Sepsis
Question 11
• Which of the following about bowel obstructions is
not correct?
– A. The most common cause of colonic obstruction without
previous surgery is cancer
– B. The most common cause of SBO in a patient without
surgery is cancer
– C. The most common cause of SBO with a history of
previous surgery are adhesions
– D. The most common cause of a SBO in a patient with no
history of abdominal surgery is an inguinal hernia
Answer 11
• B is false. The most common cause SBO with
no history of abdominal surgery is a hernia
(not malignancy)
• Most common cause of SBO with a surgical hx
is adhesions
• Most common cause of colon obstruction with
or with no PSuH is cancer
Question 12
• Which of the following is associated with vinyl
chloride exposure?
– A. Inflammatory bowel disease
– B. Ovarian cancer
– C. Angiosarcoma of the liver
– D. Leukocytosis
Answer 12
• C. Angiosarcoma of the liver is associated with
vinyl chloride exposure.
• Vinyl chloride is a known carcinogen of the
lung, liver, and brain.
• Is a chemical used in the plastics industry
Question 13
• Which of the following types of cautery
generates less heat on a slower frequency
causing tissue dehydration and vessel
thrombosis?
– A. Cutting mode
– B. Coagulation mode
– C. Bipolar mode
– D. None of the above
Answer 13
• B. Coagulation mode generates less heat on a
slower frequency causing tissue dehydration
and vessel thrombosis
Question 14
• Which of the following types of cautery leads
to heat being generated over the target area
quickly with minimum lateral spread?
– A. Cutting mode
– B. Coagulation mode
– C. Bipolar mode
– D. None of the above
Answer 14
• A. This best describes the cutting mode
Question 15
• Which of the following types of cautery is the
safest because it disperses energy only
between its tips?
– A. Cutting mode
– B. Coagulation mode
– C. Bipolar mode
– D. None of the above
Answer 15
• C. This best describes bipolar mode- this mode
is often safest as it only affects tissues within
the circuit or grasp of the instrument
Question 16
• Which of the following factors is NOT significant in
the amount of heat generated by a cautery device?
– A. Size of the contact area
– B. Power setting (frequency)
– C. Length of activation time
– D. Patient BMI
– E. Whether the waveform is continuous or intermittent
Answer 16
• D. Patient BMI has the least to do with the amount of
heat a cautery device can generate
Safest Electrosurgery Settings
• High frequency alternating current is delivered as either unipolar or
bipolar.
• Unipolar device has a generator and electrode
– When the system is activated the patient becomes a part of the circuit
– The energy is converted to heat inversely compared to the area of contact
– Thus, the application electrode is small compared to the returning
electrode to disperse the energy to prevent burning the patient
• In the cutting mode, heat is generated over the target area quickly
with minimum lateral spread (heating without coagulation.)
• In the coagulation mode, the electrocautery generates less heat on
a slower frequency, thus causing tissue dehydration and vessel
thrombosis
Safest Electrosurgery Settings
• Bipolar devices establish a short circuit between the tips of the
instrument without the need for a grounding pad.
– These are the most efficient and often safest as it only affects tissues within
the circuit or grasp of the instrument.
• Overall, the heat generated is dependent on 4 factors
– Size of the contact area
– Power setting (frequency)
– Length of activation time
– Whether the waveform is continuous or intermittent
• Other types of electro surgery include lasers, photodynamic therapy,
argon beam coagulator, high frequency sound wave techniques,
harmonic scalpel, ultrasonic tissue ablation, RFA, cryoablation,
microwave ablation, and ultrasonic cavitation devices
Question 17
• A 35 yo female with a known history of HSV and HPV in
the past presents with pelvic pain, fever, and
tachycardia. After you review her CT A/P you are
concerned for a small, contained tuboovarian abscess.
Which of the following management plans is most
appropriate at this time?
– A. Send UA, B-HCG, G & C cultures, and give her 14 days of PO augmentin
– B. Post her for a diagnostic laparoscopy
– C. Send UA, B-HCG, G & C cultures, admit her and initiate cefoxitin and
doxycycline
– D. Send UA, B-HCG, G & C cultures and consult interventional radiology for
aspiration
Answer 17
• C. In this patient with a contained TOA, send
UA, B-HCG, G & C cultures, admit her and
initiate cefoxitn and doxycycline
Question 18
• A 35 yo female that has been treated for
multiple STD’s presents with pelvic pain, fever,
and tachycardia. Which of the following are
indications to operate?
– A. Intraperitoneal rupture of a TOA on scan
– B. Persistence of pelvic abscess despite abx therapy
– C. Chronic pelvic pain
– D. All of the above are indications to operate
Answer 18
• D. All of the following are indications to
operate on a patient with PID
Question 19
• You work the above patient up and find a clearly
ruptured TOA. She states that she has one child
and would like to try to have another within the
next year. Which of the following is the most
reasonable treatment option?
– A. Observation and IV abx
– B. Unilateral salpingo-oophorectomy
– C. Hysterectomy and salpingooophorectomy
– D. Transvaginal drainage
Answer 19
• B. Unilateral salpingoophorectomy should be
attempted if possible
Treatment of Pelvic Inflammatory Disease (PID)

• PID is largely limited to sexually active females and is


classified as acute or chronic.
• The most common organisms that produce this
infection are N. Gonorrhoeae and Chlamydia
• Treatment-
– PID can be treated as an in- or outpatient depending on the severity
– Patients with peritonitis, high fever, or suspected tub-ovarian abscess (TOA)
should be admitted for IV abx Cefoxitin 2 g IM with oral probenecid or
equivalent cephalosporin for Gonorrhea.
– Doxycycline is used to cover chlamydia
Treatment of Pelvic Inflammatory Disease (PID)

• Surgical therapy becomes necessary when there is:


– Intraperitoneal rupture of a TOA
– Persistence of pelvic abscess despite abx therapy
– Chronic pelvic pain
• Unilateral salpingo-oophorectomy is the preferred
surgical tx for young women with reproductive goals
• With a ruptured TOA,, hysterectomy and
salpingooophorectomy are commonly indicated.
Question 20
• A 42 yo male is s/p MVA and just underwent a pan-
CT. The only significant finding is a moderate-sized
duodenal diverticula. The gallbladder otherwise
appears normal. He does not report any history of
abdominal pain or GI problems. Which of the
following should you recommend?
– A. Resection with gastrojejunostomy
– B. Resection with primary closure
– C. Observation
– D. ERCP with biopsy of the diverticulum base
Answer 20
• C. In this asymptomatic patient, observation is
appropriate
Treatment Duodenal Diverticulum
• Duodenal diverticula
– Need to r/o gallbladder disease
– Observe unless perforated, bleeding, obstructing, or
symptomatic
– Frequency of SB diverticula- duodenum > jejunum >
ileum
– Needs segmental resection though possibly
gastojejunostomy if perforated
Question 21
• You operate on a 35 yo male with a SBO and find a
lesion approximately 2 ft proximal to the ileocecal valve
that appears to be the point of obstruction. There is an
outpouching of the bowel is markedly inflamed
circumferentially at this point. Which of the following is
not true?
– A. Segmental resection is appropriate
– B. This is caused by failure of the omphalomesenteric duct to close
– C. Is most likely to present as SBO in both children and adults
– D. There is a high chance you will find pancreatic tissue in this lesion
Answer 21
•C is not true- though it is more likely to present
as a SBO in adults, in children it often presents
as a painless lower GI bleed.
Meckel’s Diverticulum
• Segmental resection is appropriate when there is
complicated diverticulitis, inflammation involving
the base, or the neck is 3 the diameter of the
small bowel
• It is caused by failure of the omphalomesenteric
duct to close
• Is most likely to present as SBO in adults but is
may present as a painless LGI bleed in children
– Accounts for 50% LGIB in children < 2 yrs
Meckel’s Diverticulum
• There is a high chance you will find pancreatic
tissue in this lesion as it is the most common tissue
found
• Gastric tissue is more likely to be symptomatic
• Do not have to remove incidental diverticulum
• Is a true diverticulum
• 2% IC valve, 2% population, usually presents in first
2 years of life with bleeding
Question 22
• A 42 yo male is on cyclosporine as a part of his
immunosuppressive regimen s/p renal transplant.
Which of the following is true about this drug?
– A. It is mostly eliminated through the kidneys
– B. It inhibits both resting and active T lymphocytes
– C. It is an amine that is a derivative of methotrexate
– D. It inhibits the activation of resting T lymphocytes
Answer 22
• D. It inhibits the activation of resting T lymphocytes
Question 23
• Which of the following would you least likely
expect as a side-effect?
– A. Hyperkalemia
– B. Nephrotoxicity
– C. Tremors
– D. Hypokalemia
Answer 23
• D. Hypokalemia
Cyclosporine
• Discovered in 1972; relatively new class of
immunosuppressive agents
• Cyclic peptide produced by a fungus
• Many of its effects are T-cell specific
• Inhibits activation of resting T-lymphocytes
resulting in an inhibition of IL-2 production
• However, once T-lymphocytes are activated,
cyclosporine is not effective in suppressing the
immune response
Cyclosporine
• Absorbed slowly and incompletely from GI tract
• Excretion is primarily through bile
• Well-documented enterohepatic cycle
• Adverse effects:
– Hirsuitism
– Neurotoxicities
– Hyperkalemia
– Nephrotoxicity
– Hepatotoxicity
– Nephrotoxicity, hypertension, and tremors most common
Question 24
• Which of the following factors are not a part of
the Harris-Benedict equation for energy
expenditure?
– A. Weight
– B. Height
– C. Body temperature
– D. Sex
– E. Age
Answer 24
• C. Body temperature is not a variable in the
equation of basal energy expenditure

• Basal energy expenditure (BEE)


– Male= 66.5 + 13.7 x weight (kg) + 5.0 x height (cm)
– 6.8 x age (yr)
– Female- 655.1 + 9.56 x weight + 1.85 x height (cm) –
1.68 x age (yr)
Question 25
• Fill in the blanks:
• Protein generates ( A ) calories per gram and it
takes ( B ) grams of protein to produce 1 gram of
nitrogen.
• The resting energy expenditure for most patients is
(c) calories per kg per day. Most non-protein
calories are provided by (D- choose from lipids,
glucose, or alcohol).
• Fat provides (E) calories per gram where proteins
and carbohydrates each produce around (F) calories
per gram
Answer 25
• A- 4
• B- 6.25
• C- 25
• D- glucose
• E- 9
• F- 4
Energy Requirements
• Harris-Benedict Equation- determines basal energy
expenditure
• Formulas based on body weight, height, sex, age
• Metabolic cart measurement- determines energy
expenditure from oxygen utilization and CO2
generation
• Basal energy expenditure (BEE)
– Male= 66.5 + 13.7 x weight (kg) + 5.0 x height (cm) – 6.8 x
age (yr)
– Female- 655.1 + 9.56 x weight + 1.85 x height (cm) – 1.68 x
age (yr)
Energy Requirements
• Resting energy expenditure of most pts- 25 kcal/
kg/day
• In stressed pts this may increase to 35-40 kcal/kg
• Adequate caloric replacement can be determined
by frequent metabolic measurements such as
nitrogen excretion
– This represents gluconeogenesis of protein to meet energy
requirements if adequate non-protein calories are
unavailable
Question 26
• Fill in the blanks:

• A normal, healthy man needs about (A) grams


of protein per kg of body weight per day.
• The ratio of grams nitrogen to non-protein
calories should be around (B:C)
Answer 26
• A- A normal, healthy man needs ~ 1 g protein
per kg per day
• B: 6.25 g protein will have 1 g nitrogen
• C. 1 gram nitrogen : 100 non-protein calories is
appropriate nitrogen balance
Question 12
• To maintain optimal nitrogen balance
(assuming you are going to give glucose to
provide all of the non-protein calories), how
many grams of glucose will a 72kg man need
daily?
– A. 350 g
– B. 400 g
– C. 450 g
– D. 500 g
Answer 12
• C. 450 grams of glucose
• Since 1 gram of nitrogen should be given for 100
non-protein calories, it would take 6.25 grams of
protein and 25 grams of glucose to provide this
• Assuming his energy requirements are ~ 25
kcal/kg/day ->
– This amounts to 1800 nonprotein calories per day (72 kg x
25 kcal/kg/day)
– Since glucose produces 4 calories per gram-> 1800 cal/ 4
cal/g = 450 gm glucose
Question 27
• On the same 72 yo male, to maintain nitrogen
balance, how many grams of protein should he
be provided daily?
• A. 115 g protein
• B. 122.5
• C. 125
• D. 130
Answer 27
• B. 122.5 grams of protein.

• Remember the ratio of 6.25 grams protein to


25 grams glucose?
• Therefore, the 450 grams glucose we calculated
should be divided by 25- gives you a factor of
18
• For protein, 6.25 grams is now multiplied by 18
to give 122.5 grams of protein
Protein Requirements in TPN
• Determination of the patient’s protein requirement is the
first step in formulating a nutritional support plan
• Normal, active man needs 0.9- 1.5 grams protein per kg of
body weight per day
• 6.25 protein make 1 g nitrogen
• Requirements dependent on clinical state
• 1 gm/kg/day needed early in refeeding after starvation
• 2-3 g/ kg/day in burned or severely septic pts
• Total intake may be limited to 40-50 g/day in liver failure
Protein Requirements in TPN
• Nitrogen / non-nitrogen protein calories 1:100 – 1:200 to obtain optimal
nitrogen balance
• Example:
• 1:100 = 1 gram N to 100 calories
• OR
• 6.25 g protein to 25 g glucose
A 70 kg male will require:
Non-protein calories- 70 kg x 25 = 1750 cal/day
Question 28
• Which of the following about Crohn’s disease is not
true?
– A. The most commonly involved bowel segment is the
terminal ileum
– B. Anal disease can be manifested as large skin tags and
should be treated with Flagyl, not excision
– C. Less than ½ of these patients eventually need an
operation
– D. Perirectal disease may respond to resection of diseased
small bowel
Answer 28
• C is false- most patients with Crohn’s ultimately
need an operation
Crohn Disease
• The most commonly involved bowel segment is the
terminal ileum
• Anal disease can be manifested as large skin tags and
should be treated with Flagyl, not excision
• 90% of these patients eventually need an operation
• Perirectal disease may respond to resection of diseased
small bowel
• Intermittent abdominal pain, n/v/d/f, weight loss
• Even after resection, recurrence rate is around 50%
Question 29
•Which of the following characteristics are not
consistent with Crohn’s Disease?
– A. Mucosal only involvement
– B. Skip lesions
– C. Cobblestoning
– D. Creeping fat
– E. fistulas
Answer 29
• A. Crohn disease tends to have transmural
bowel involvement
Crohn Disease
• Extraintestinal manifestations-
– Arthritis, pyoderma gangrenosum,
– Erythema nodosum, ocular disease
– Growth failure
– Megaloblastic anemia from B12 and folate malabsorption
• Pathology-
– Transmural involvement
– skip lesions
– cobblestoning
– narrow deep ulcerations
– creeping fat
– fistulas
Crohn Disease
• Medical agents-
– 5-ASA
– Infliximab (Remicade)- a TNF-alpha inhibitor (usually
for abscess & fistula)
– Steroids
– Azathioprine
– Loperamide
Question 30
• A 40 yo woman with a long standing history of
Crohn disease is found to have a perianal fistula
above the dentate line involving the sphincter
muscles. The fistula is well-drained and there is no
sign of pelvic sepsis. What is the best option for
treatment?
– A. Seton placement
– B. Anti-TNF antibody
– C. Treatment with cipro and flagyl
– D. Fistulotomy
– E. Diverting colostomy
Answer 30
• B. Anti-TNF antibody would be most
appropriate
Perianal Fistula in Crohn’s
• Perianal disease affects ~ 1/3 of patients with
Crohns
• Management- optimize medical treatment, then
surgery
• Fistulas above dentate line involving the
sphincter are complex
– Medical mgmt w/ anti-TNF ab preferable 1st choice
– Fisulotomy increases risk of incontinence
Perianal Fistula in Crohn’s
• Treatment with flagyl and cipro is best for non-
complex fistulae near the anus
• Diverting colostomy and advancement flap- for
more advanced disease that fails mgmt
• Seton- important to consider in cases of pelvic
sepsis
Question 31
• A 32 yo male has had multiple ileal resections
secondary to distal small bowel obstruction from
Crohn’s. Which of the following conditions is he at
risk for developing?
– A. Calcium oxalate kidney stones
– B. Macrocytic anemia
– C. Calcium bilirubinate gallstones
– D. All of the above
Answer 31
• D. All of the above are sequlae of extensive
ileal resection
Crohn Disease
• Surgical indications-
– Obstruction- partial can be tx conservatively
– Abscess- sometimes tx with perc drainage
– Megacolon- 15% perforation, usually contained
– Blind loop obstruction
– Fissures- Setons and conservative measures preferable; do not do lateral
internal sphincerotomy
– Enterocutaneous fistula
– Anorectal/ vaginal fistulas- may need advancement flap or diversion
– Do not need clear margins, just get 2 cm away from gross disease
Crohn Disease
• Complications from terminal ileum removal:
– Megaloblastic anemia from decreased B12 uptake
– Decrease bile salt uptake- increased bile salt delivery
to colon-> steatorrhea
– Decrease oxalate binding- increased intraluminal fat
binds calcium & allows for oxalate absorption
– Gallstones (calcium bilirubinate) from malabsorption
of bile acids
Question 32
• A 45 yo male presents with right lower
quadrant pain; you end up exploring him &
find a 2cm carcinoid that is close to the
appendiceal base. You next perform:
– A. Close surveillance
– B. Right hemicolectomy
– C. Appendectomy
– D. Right hemicolectomy with removal of at least 10
cm terminal ileum
Answer 32
• B. Right hemicolectomy is appropriate.
• For appendiceal carcinoids:
– Appendectomy adequate if less than 2 cm
– Need right hemicolectomy if near the base or
greater than 2 cm
Question 33
• He does well for a year at which point he returns
because of intermittent flushing and diarrhea. Though
your first CT doesn’t show anything, an octreotride scan
suggests there is a small mass in the left lateral
segment of the liver. Which of the following is not
true?
– A. If this anatomically appears resectable, you should attempt to do so.
– B. Cholecystectomy should be performed in case he needs embolization in the
future
– C. Octreotide can be helpful in managing symptoms
– D. You should not try to resect any metastatic carcinoids
Answer 33
• D. It is appropriate to attempt resection of
metastatic carcinoids.
Carcinoids
• Treatment-
– Should be approached oncologic ally like other GI tract cancers- segmental
resection and lymphadenectomy
– Cholecystectomy should be performed in patients with resectable liver mets in
case they need embolization in the future
– Octreotide can be helpful in managing symptoms
– Pentagastrin can exacerbate symptoms
– Bronchospams- tx w/ aprotinin
– Flushing- tx w/ alpha-blockers
– Chemotherapy (streptozocin and 5FU) for pts with unresectable mets and
carcinoid syndrome
Question 34
• A 75 yo man is sent to your clinic with complaints of food
sticking in in throat, embarrassing bad breath, and a nagging
cough. Of note, he recently was hospitalized with aspiration
pneumonia. Which of the follow are not true about his
condition?
– A. It is caused by a pulsion diverticulum that arises adjacent to the
inferior pharyngeal constrictor
– B. Gastrograffin swallow (instead of barium) should be performed
because gastrograffin is inert if aspirated
– C. Surgical intervention is indicated in symptomatic patients with this
condition regardless of size
– D. Open repair is performed through the left neck and always involves
a myotomy through the cricopharyngeus and thyropharyngeus
muscles
Answer 34
• B. is incorrect. Barium is the usual agent used
because it is inert if aspirated.
Question 35
• Which of the following about Zenker’s
diverticulum is true?
– A. Surgical repair is usually successful
– B. Large (> 2 cm) diverticula do not need to be
resected
– C. They are full-thickness diverticula
– D. Most successful treatment involves esophageal
resection
Answer 35
• A. Most patients do well after myotomy +/-
resection of the diverticulum
Zenker’s Diverticulum
• Usually presents in older patients
• Thought to be due to a loss of tissue elasticity and
muscle tone with age
• Mucosal and submucosal layers herniate between
thyropharyngeus fibers and horizontal cricopharyngeus
fibers
• Usually dissects down the left side of neck
• Symptoms- nagging cough, excess salivation,
regurgitation of foul-smelling undigested material,
halitosis, voice changes
• More severe- respiratory infections lung abscess,
aspiration pneumonia
Zenker’s Diverticulum
• Dx by barium swallow
• EGD not necessary to diagnose but is performed to
evaluate anatomy and r/o cancer
• Surgical therapy is indicated in symptomatic patients
regardless of diverticulum size
• Myotomy should always be performed
• Diverticulum larger than 2 cm should be excised
• Alternative treatments- diverticulopexy, endoscopic
obliteration of the common wall, botox injection
Zenker’s Diverticulum
• Complications- RLN trauma, fistulas, hematoma,
infection, aspiration, recurrence
• Fistulas usually controlled once prevertebral space
drained
• Aspiration most serious
– Division of cricopharyngeus may lead to massive
tracheobronchial aspiration in someone with incompetent
GE jxn
– Severe GER may be relative contraindication to this
procedure until corrected
Zenker’s Diverticulum
• 90% do well with surgical treatment
• Those without diverticula or with poor
pharyngeal contractility may not do as well
• Open repair vs. endoscopic-
– For diverticula less than 3 cm, surgical repair
superior
– For diverticula greater than 3 cm, results are similar
Question 36
• An 8 yo male is brought to you for evaluation of a
midline neck mass that moves with swallowing. He
otherwise reports no complaints or symptoms. You
suspect a thyroglossal duct cyst. Which of the following
is NOT true about this condition?
– A. May occasionally find an external fistula on the skin
– B. When papillary cancer is found in a TG cyst, there is a 25% chance that
papillary cancer exists elsewhere in the thyroid
– C. Squamous cell cancer is the most common that will arise from these cysts
– D. Almost always require excision
Answer 36
• C is false.
Thyroglossal Duct Cysts
• Remnants of the tract along which the thyroid gland
descended into the neck from the foramen cecum
• Account for ~ 70% of all congenital abnormalities of the neck
• May be found in any age; most common in the first decade of
life
• Can be a lone cyst, a cyst with a sinus tract, or a solid core of
thyroid tissue
• Variable in size
• Almost always found in the midline, at or below the level of
the hyoid bone
• Can occasionally be slightly lateral with external skin fistula
Thyroglossal Duct Cysts
• Differential dx-
– Dermoid cysts, lymphadenomegaly in the anterior jugular chain, and
cutaneous lesions
• Operative treatment indicated both due to cosmetic
considerations & high incidence of recurrent infection,
including abscess formation.
• 1% of thyroglossal duct cysts contain cancer
– Papillary cancer most common, then squamous cell
• 25% of patients with papillary thyroid cancer in thyroglossal
duct cysts have papillary thyroid cancer in other parts of the
thyroid gland
• About 10% have nodal metastases, occasionally bilateral
Question 37

• A 2 year old boy presents with a compressible


neck mass that transilluminates. You suspect this
is a cystic hygroma. Which of the following is true
about this condition?
– A. Usually present with pain and compressive symptoms
– B. 1/3 harbor focus of squamous cell carcinoma so
should be excised
– C. Most often present in 2nd decade of life
– D. Often recur after excision
Answer 37
• D is correct
Cystic Hygroma
• Lymphangioma that arises from vestigial lymph channels
in the neck
• Almost always noted by the second year of life
• Range from simple thin-walled cyst in the floor of the
mouth to involvement of all tissue of mouth to
mediastinum
• Painless cyst in the posterior cervical triangle or
supraclavicular area in 80%
Cystic Hygroma
• Diffuse, soft, doughy, irregular mass
• Transilluminates
• May feel like lipomas but have less well-defined
margins
• Aspiration- straw-colored fluid
• Can be confused with angiomas (which are
compressible), pneumatoceles from the apex of the
lung, or aneurysms
– Differentiate from vascular lesions by arteriography.
Cystic Hygromas
• Occasionally grows suddenly as a result of an upper
respiratory tract infection, infection of the hygroma
itself, or hemorrhage into the tissues
• Can compress the trachea and cause dysphagia.
• May be treated expectantly if asymptomatic
– Tend to regress spontaneously
– Reserve excision for those that are symptomatic or do not regress
• Satellite lesions and extension into surrounding
structures can make complete excision difficult
– Recurrences common
– Staged resections often necessary
Question 38
• A 67 yo male presents with a right cervical enlarged
lymph node. FNA reveals squamous cell carcinoma.
You suspect that it is somewhere in the head or neck
given his long smoking history but despite extensive
workup, you can’t find the primary. No distant disease
is found. Which of the following is the most
appropriate initial treatment?
– A. Chemotherapy and radiation
– B. Tracheostomy and PEG
– C. Right sided neck dissection with radiation
– D. Bilateral neck dissection
Answer 38
• C is most appropriate
Mass With Unknown Primary
• When cervical lymph nodes are found to contain
metastatic squamous cell carcinoma, the primary
tumor is in the head and neck about 90% of the time
• If no primary tumor is identified, the patient should
undergo endoscopic evaluation of the nasopharynx,
the hypopharynx, the esophagus, the larynx, and the
tracheobronchial tree under general anesthesia
– Biopsies of the nasopharynx, the tonsils, and the
hypopharynx often identify the site of origin
Mass With Unknown Primary
• If the biopsies do not reveal a primary source of
cancer, treatment should be unilateral neck
dissection, followed by radiation therapy directed
toward the neck, the entire pharynx, and the
nasopharynx
• 15 to 20% of cases, the primary cancer is
ultimately detected
• Overall 5-year survival in such cases ranges from
25 to 50%.
Mass With Unknown Primary
• Malignant melanoma found in a cervical lymph
node but no primary tumor is evident
– Ask about previous skin lesions & repeat head and neck
examination (pay attention to the scalp, the nose, the
oral cavities, and the sinuses)
– Ophthalmologic examination
– If physical examination and radiographic studies find no
evidence of metastases, modified neck dissection should
be performed on the involved side.
Mass With Unknown Primary
• Metastatic adenocarcinoma in a cervical lymph
node with no known primary tumor:
– Most common primary sites in the head and neck are the
salivary glands and the thyroid gland
– Possibility of an isolated metastasis from the breast, the GI
tract, or the genitourinary tract need to be considered
– If no primary site is identified, the patient should be
considered for protocol-based chemotherapy and
radiation therapy, directed according to what the primary
site is most likely to be in that patient
Question 1
• What phase of the cell cycle determines the
length of the cell cycle?
– A. M
– B. G1
– C. G0
– D. S
Answer 1
• B. The G1 phase determines how long the cell
cycle will take
Question 2
• Which of the following correctly describes the
Cori cycle?
– A. The hepatic process of glycogenolysis
– B. The metabolism of glucose by the heart
– C. The utilization of ketones by the brain
– D. The hepatic conversion of lactate into glucose
Answer 2
• D. The Cori cycle describes the hepatic
conversion of lactate into glucose
Question 3
• Which is NOT an initial response to vascular
injury?
– A. Vasoconstriction
– B. Platelet adhesion
– C. Thrombin generation
– D. Vascular dilation
Answer 3
• D. Vascular dilation
Question 4
• Which of the following factors is degraded by
plasmin?
– A. Factor V
– B. Factor VII
– C. Fibrinogen
– D. Von Willenbrand factor
– E. Fibrin
Answer 4
• D. Plasmin does not degrade VWF.
Question 5
• Which 2 factors do DDDAVP and conjugated
estrogens cause the epithelium to release?
– A. Factor II and VWF
– B. Plasmin and Factor VIII
– C. Plasmin and VWF
– D. Factor VIII and vWF
Answer 5
• D. Factor VIII and vWF are released from epithelium
in response to DDAVP and estrogens
Question 6
• A 38 year old kidney transplant patient is admitted with
fever and chills after he received 6 weeks of IV antibiotic
coverage for aspiration pneumonia. A thick walled cavity in
the chest apex on CT scan with underlying parenchymal
lung destruction is found. There is a crescent shaped
radiolucency within this cavity with a central mass that
shifts when the patient is placed in the prone position on
CT scanning. What is the most likely diagnosis?
– A. Adenocarcinoma of the lung
– B. Metastatic lesion
– C. Tuberculosis
– D. Aspergilloma
Answer 6
• D. Aspergilloma
• Also known as a "fungus ball“
• Aspergillosis is an opportunistic infection related to an
underlying lung disorder (bronchiectasis, chronic lung abscess,
tuberculosis or cavitary lung carcinoma)
• It infects a chronic lung abscess cavity and invades underlying
lung tissue, hence it is a 'complex' aspergilloma.
• Invasive aspergilloma is a life threatening aggressive disease
that affects immunocompromised patients.
• CT abnormalities- 'halo sign', air crescent lesions, a cavitary
lesion or consolidation of the lung associated with
pneumothorax.
Question 7
• You are performing an axillary dissection and
take only level 1 nodes. Where does your
medial border extend?
– A. The lateral edge of the pectoralis minor muscle
– B. The medial edge of the pectoralis minor muscle
– C. The thoracic inlet
– D. None of the above
Answer 7
• A. A level one dissection would extend to the
lateral border of pec minor
Levels of Axilla
• Remember the pectoralis minor separates
levels of axilla
• I= lateral to pec minor
• II=posterior
• III= medial to pec minor
– This extends to Halstedt’s ligament/thoracic inlet.
Question 8
• A 56 yo female presents with a sore, inflamed appearing right
breast that has puckering concerning for inflammatory breast
cancer. You confirm this on biopsy and her other films are
otherwise negative for metastatic disease. Which of the
following should you next recommend?
– A. Neoadjuvant radiation followed by mastectomy
– B. MRM with postoperative chemoradiation
– C. Neoadjuvant chemo, MRM, then radiation
– D. Neoadjuvant chemoradiation only
Answer 8
• C. Neoadjuvant chemo, MRM, then radiation
Rx inflammatory breast cancer

• Neoadjuvant chemo first


• Mastectomy (modified radical)
• Postoperatively they should receive XRT
Question 9
• A 67 yo male that you performed a Hartmann’s procedure 6 weeks ago who
had feculent peritonitis and a prolonged postoperative course is in clinic for
follow up. He notes some malaise and RUQ pain. You order a CT which
shows a hepatic abscess. Which of the following is not true about this
condition?
– A. It usually responds to drainage and antibiotics
– B. It is most common in patients with previous GI infection
– C. E. coli and Klebsiella are common causative organisms
– D. Primary therapy should be wedge resection
Answer 9
• D. Primary therapy should not be wedge resection
Pyogenic liver abscess

• Primary causes are biliary infection


(cholecystitis/cholangitis) or seeding from
portal vein drainage (appendicitis/diverticulitis)
• E. coli, klebsiella and strep are most common
organisms
• Rx with abx and/or percutaneous drainage and
search for primary source if not obvious
Question 11
• A 41 yo male that returned from a 6 month business stay in
Mexico presents with RUQ pain and fever. He otherwise has
no significant PMH. Scan reveals an abscess and cultures show
Entameoba histolytica, Which of the following should you
recommended for treatment?
– A. Laparoscopic drainage and ablation of cyst wall
– B. Wedge resection
– C. IV flagyl and drainage
– D. Mebendazole and drainage
Answer 11
• C. IV flagyl and drainage
Rx Amebic Liver Abscess

• Causative organism is Entamoeba histolytica which


enters liver via portal system from primary GI infection
• Often present with fever, RUQ pain and tenderness
• Indirect hemagglutination test may be helpful in
diagnosis
• Rx with Metronidazole
• Surgery or perc drainage reserved for abx failures.
Question 12
• A 65 yo female has a new diagnosis of a right colon
adenocarcinoma and 2 hepatic tumors involving segments 1
and 2. She has no other obvious disease and is in good health.
Which of the following should you recommend?
– A. Right hemicolectomy and right hepatic lobectomy
– B. Right hemicolectomy and RFA of the 2 lesions
– C. Neoadjuvant chemoradiation
– D. Right hemicolectomy and left hepatic resection
Answer 12
• D. Right hemicolectomy and left hepatic resection
Colon Cancer with Hepatic Mets
• Remember the anatomy:
– Seg 1 = caudate
– 2-4 = left lobe (2/3 compose left lat segment)
– 5-8 =right lobe.
• For board purposes, unilobar disease is usually
resected.
• Segment 1 and 2 would be resected with left hepatic
lobectomy
Question 13
• You stop the coumadin and begin workup for Protein C
deficiency. Another anticoagulant is initiated. Which of
the following are correct statements?
– A. Protein C is a hepatically made product and it has a relatively
short half life
– B. Protein C has a relatively long half life and is produced mostly
in the peripheral tissues
– C. Protein C is vitamin K dependent
– D. Protein C deficiency is an autosomal-dominant gene disorder
– E. A, C, and D
– F. B, C, and D
Answer 13
• E. Protein C is vitamin K dependent product
that is made in the liver. The deficiency is due
to an autosomal dominant gene disorder.
Question 14
• Which of the following is true about Coumadin?
– A. Coumadin is a vitamin K antagonist that can produce a
transient hypercoagulable state cue to the coagulation
inhibition normally provided by the short-half life Protein
C
– B. You can expect that a patient on anabolic steroids may
be particularly sensitive to coumadin
– C. A person taking corticosteroids may be less sensitive to
the effects of coumadin
– D. All of the above
– E. Only A and B
Answer 14
• D. All of the above are correct statements
about coumadin
Protein C
• Is a vitamin K dependent inhibitor of the procoagulant
system
• Made in liver
• Inactivates factor V and VII:C
• Inhibitory action is facilitated by protein S
• 4-5% of people under 45 have protein C deficiency
• Protein C deficiency most commonly presents as
unexplained venous thrombosis.
– Autosomal dominant genetic disorder due to either a CRM gene deletion or
dysfunction gene
Protein C
• Protein C and S have short half-lives
• Vitamin K antagonists such as coumadin can
produce a transient hypercoagulable state
– This is due to loss of coagulation inhibition normally
provided by protein C
• Protein C deficiency causes “Warfarin Induced
Skin Necrosis”
Warfarin
• Anticoagulant effect of coumadin is consistently REDUCED in pts on:
– Barbituates
– Contraceptives
– Corticosteroids
– ACTH
• Anticoagulant effect of coumadin is consistently ENHANCED in pts on:
– Phenylbutazone
– Clofibrate
– Anabolic Steroids
– D-thyroxine
– Glucagon
– Quinidine
– Many antibiotics
• Factors other than coumadin that can be evaluated by prothombin time and prolong it:
– II, V, VII, X
– Fibrinogen
– Heparin, warfarin, liver disease
Question 15
• Which of the following may have or cause vitamin K
deficiency and lead to impaired coagulation?
– A. A newborn baby
– B. Someone who has absolutely no vegetables in their diet
– C. A person with Crohn’s disease that has multiple bowel
resection and clinically has short bowel syndrome
– D. A cachectic patient with AIDS that has been taking
multiple prophylactic antibiotics for the last 8 months
– E. All of the above
– F. A, B, and C
Answer 15
• E. All of the above may be deficient in vitamin
K
Question 16
• Which of the following correctly states the mechanism of
coumadin?
– A. It causes anticoagulation by breaking down fully formed factors 2,
7, 9, and 10
– B. It affects the gamma carboxylation of the precursor proteins of the
clotting cascade
– C. The process is a competitive inhibition and can be overcome by
administering vitamin K
– D. The process is a noncompetitive inhibition so replacement of
factors with FFP is the only option to treat a patient with too much
coumadin on board
– E. A, B, and C
– F. B and C only
– G. None of the above
Answer 16
• F is correct
Vitamin K
• Present in most edible vegetables, particularly green leaves
• Produced also in vivo by intestinal bacteria
• Serves as component of specialized microsomal enzyme that
affects posttranslational gamma carboxylation of precursor
proteins of the clotting cascade
– Factors 2, 7, 9, 10 are vitamin K dependent
• 80% of ingested vitamin K is absorbed from small bowel into
intestinal lymph
• Is a fat-soluble substance
– Deficiencies can occur in fat malabsorption
– Long-term use of abx may eliminate intestinal bacteria as a source and
promote deficiency when dietary intake is suboptimal
Vitamin K
• Warfarin induces hypoprothrombinema and
anticoagulation by affecting the gamma
carboxylation of the precursor protein
– This is a competitive inhibition so it can be
overcome by Vit k administration
• Newborns tend to lack vitamin K
– Inherently low stores at birth
– Immature GI flora
– Limited dietary intake
Vitamin K
• Routine determination of PT should precede
surgical procedures when pts have any significant
risk factors for deficiency
– People with less than 70% normal activity should receive
corrective therapy with vitamin K
– Vit k deficient states can be differentiated from liver
disease related hypoprothombinemic states by
measuring noncarboxylated prothombin precursors that
accumulate in vitamin deficiency
Question 17
• You are performing a Whipple when you encounter a vessel in an
unexpected position that you think may be an aberrent right hepatic artery.
Which of the following is true about this condition?
– A. It is a right hepatic artery that arises from the celiac and travels inferiorly
towards the porta near the head of the pancreas.
– B. It arises from the SMA and passes posterior to the portal vein
– C. Ligation of this vessel will cause hepatic ischemia that will require a right
hepatic resection
– D. It arises directly off of the aorta
Answer 17
• B. It arises from the SMA and passes posterior to the portal
vein
Anatomy Aberrent RHA
• Replaced right hepatic arises from the SMA and
passes posterior to the portal vein
• Replaced left hepatic arises directly from the
celiac or from the left gastric and passes directly
into the liver
• Either can be ligated if needed without
significant issue relative to hepatic ischemia
Question 18
• A 28 yo female who has been on OCP’s for 5
years is found to have an asymptomatic hepatic
adenoma. Which of the following is the most
likely complication from this lesion
– A. Malignant transformation
– B. Rupture and hemorrhage
– C. Infection
– D. Portal vein thrombosis
Answer 18
• B. Rupture and hemorrhage
Hepatic Adenoma
• Benign neoplasm.
• Most common in young women.
• Assoc with OCP use.
• Primary complication is rupture and hemorrhage.
• If diagnosed and small, manage by stopping OCP use
• If not a/w with OCP use, is large, or symptomatic,
resection is indicated
Question 19
•A 96 yo man presents with rectal bleeding and is
found to have a 2 cm villous rectal polyp. Which of
the following would be a contraindication to transanal
excision?
• A. 25% circumference of the rectum
• B. 6 cm away from anal verge
• C. 2.1 cm in size
• D. Lymphovascular invasion
Answer 19
• D. Lymphovascular invasion
Villous Adenoma in Elderly Patient
• This is best treated with transanal excision
• Criteria for transanal excision:
• Must be polyp or no greater than T1 tumor
• Must be <40% circumference of rectum
• Must be within 8-10cm of anal verge
• If invasive, must not have lymphovascular invasion
(LVI) and must not be poorly differentiated (these
are risk factors for LN mets)
Question 20
• A 67 yo female is found to have a 2 cm mass in
the anal canal. Biopsy shows SCC pathology.
Which of the following should you
recommend?
– A. WLE
– B. APR
– C. Mitomycin , 5-FU, and XRT
– D. Fulgaration and podoxyflin
Answer 21
• C. Mitomycin , 5-FU, and XRT
SCC of Anal Canal
• Chemo (5-FU/Mitomycin) + Radiation is the
standard treatment
• Surgery reserved for failures of chemo/XRT
Question 22
• A 62 yo female undergoes FNA for a thyroid
mass. This reveals a follicular cell neoplasm so
you perform a lobectomy. Final pathology shows
Hurthle Cell neoplasm. Which of the following is
the next best management plan?
– A. Thyroid irridation
– B. Completion thyroidectomy
– C. Observation
– D. Completion thyroidectomy and LND
Answer 22
• C. Observation
Hurthle Cell Neoplasm
• Tx of Hurthle cell tumors of the thyroid is the same
as for follicular neoplasms
• They are difficult to diagnose by FNA- may appear
as “follicular neoplasm” on FNA
• Like FNA, frozen section also difficult to distinguish
carcinoma
• Surgery is always a lobectomy as only ~20%, of
these will be carcinoma, just like follicular
• If carcinoma identified on permanent section, then
completion lobectomy is indicated
Question 23
• 2 weeks s/p Whipple your pt has early satiety
with po intake. You start metoclopramide and
erythromycin. Which receptor does
erythromycin bind to increase gastrointestinal
motility?
– A. Somatostatin receptor
– B. Acetylcholine and dopaminergic receptors
– C. GABA receptor
– D. Motilin receptor
Answer 23
• D. Erythromycin binds to the motilin receptor.
The motilin receptor is found primarily in the
stomach, duodenum, and colon.
Metoclopramide improves motility by
stimulating acetylcholine release and blocking
dopaminergic receptors.
Question 24
• A 32 yo female with MEN-1 has both parathyroid hyperplasia
with hyperparathyroidism as well as a 1.5 cm gastrinoma. She
has no complaints of reflux or history of PUD. Which of the
following should you recommend?
– A. Gastrectomy then parthyroidectomy
– B. Resection of the gastrinoma then parathyroidectomy
– C. Parathyroidectomy with autotransplantation
– D. Medial management only
Answer 24
• C. Parathyroidectomy with autotransplantation
Sequence of Therapy in MEN 1
• Resect the parathyroids first then do
gastrinoma.
• School of thought is that once you correct the
hypercalcemia, it makes control of acid
hypersecretion easier and makes rx gastrinoma
purely elective.
Question 25
• A 50 yo undergoes total thyroidectomy for a large
goiter. Postoperatively, her voice somewhat hoarse
and it is slightly worse when she sees you in clinic 1
week later. You suspect a RLN injury. Which of the
following should you recommend?
– A. Re-exploration and repair of the nerve
– B. Radiotherapy
– C. ENT consult with injection of the vocal cords
– D. High dose steroids
Answer 25
• C. ENT consult with injection of the vocal cords
RLN injury
• If recognized immediately, then repair nerve
primarily.
• Recognized postop, then goal should be
“medialization” of the cords, usually by
injection directly into cords
– This can allow improved phonation and optimized
glottic closure to prevent aspiration.
Question 26
• A 75 yo man presents with a painless, small mass overlying his
left parotid. You obtain a biopsy which reveals cells suspicious
for pleomorphic adenoma. Which of the following is true
about this condition?
– A. It is the second most common benign tumor of the parotid
– B. There is a relatively high transformation rate into malignant disease
so total parotidectomy with facial nerve reconstruction is
recommended
– C. Though the tumor itself is benign, there is a slight chance it may
undergo malignant transformation so superficial parotidectomy is
recommended
– D. The histologic appearance tends to be very homogeneous
Answer 26
• C. Superficial parotidectomy is the
recommended treatment
Parotid Gland Tumors- Benign
• 70% are benign; 70% of all salivary tumors originate in parotid
• Pleomorphic adenoma- most common (70%); “mixed” tumor with
epithelial and myoepithelial elements
– 8.5% chance of ultimately becoming malignant; fast growth suggestive of
malignancy
– Superficial parotidectomy with facial nerve preservation usually suffice
• Warthin tumor- 2nd most common; papillary cystadenoma
lymphomatosum
– Older pts, smokers, traditionally thought to have male predilection (now
questionable)
– Slow growing, cystic, well-demarcated, very low malignant transformation
– Low recurrence after resection
Parotid Tumors- Benign
• Less common- adenomas and oncocytomas
• Treatment- superficial parotidectomy with
sparing of the facial nerve
• Pain is due to nerve infiltration/ fixation->
suspect malignancy in pts who present with
pain
Question 27
• A 51 yo man presents with a firm mass in the left
parotid. Biopsy is obtained which returns as suspicious
for acinic cell carcinoma. Which of the following is
true?
– A. This is a highly malignant tumor with 5-year disease free survival less than
10%
– B. This is considered a low grade tumor and when treated with complete
resection has a good prognosis
– C. Neoadjuvant chemoradiation should be administered first with surgery being
reserved for nonresponders
– D. This tends to present at a slightly older age than other salivary tumors so 51
is unusual
Answer 27
• B. Acinic cell carcinoma is low-grade and has
~85% 5 year survival with resection.
Question 28
• A 75 year old man who underwent a total
parotidectomy with wide resection for a malignant
tumor 1 year ago presents with a new mass in his lung
that is suspicious for metastatic disease. You review the
pathology and operative report from his record and it
states that there was perineural infiltration and skip
lesions along the facial nerve. Which of the following
tumors is this most likely to be?
– A. Acinic cell carcinoma
– B. Warthin’s tumor
– C. Adenoid cystic carcinoma
– D. Mucoepidermoid carcinoma
Answer 28
• C. Perineural invasion is most consistent with
adenoid cystic carcinoma
Parotid Gland Tumors- Malignant
• Includes primary SCC, mucoepidermoid
carcinoma, acinic cell carcinoma,
adenocarcinoma, adenoid cystic carcinoma,
carcinoma arising from pleomorphic adenoma,
and malignant mixed tumor
• Low grade- Acinic cell, low-grade
mucoepidermoid, adenoid-cystic
• High-grade- adenocarcinoma, squamous cell,
high grade mucoepidermoid
Malignant Parotid Tumors
• Treatment- Usually involves excision of all involved
tissues
– Complete parotidectomy
– Can also include facial nerve, mandible, skin, external
auditory canal, and neck dissection of lesion is high grade
or nodes are involved
• Acinic Cell-
– 6% of salivary gland tumors; 83% are in parotid
– Mean age- 44
– Usually do not invade nerves and metastasize late
– 5-yr survival ~ 85%
Parotid Tumors- Malignant
• Adenoid cystic carcinoma-
– Aggressive
– Exhibits high incidence of perineural spread with skip
metastasis along the facial nerve
– Incidence of local recurrence high
– Postoperative radiation therapy should be provided to
patients who are at high risk for relapse (close or positive
margins, perineural invasion
– Cervical LN mets low though distant mets found in 38%
– Long term f/u necessary
Parotid Tumors
• Mucoepidermoid carcinoma-
– Most common malignant tumor of the parotid- 1/3 of parotid cancers
– Subclassified into low-, intermediate-, and high-grade tumors
– High-grade tumors-
• Selective cervical node dissection & postoperative radiation therapy
often required
• 5-year survival rate less than 50% with multimodality tx
– Low-grade tumors-
• More circumscribed; more mucinous cells
• Surgical therapy without radiation yields a 5-year survival rate of 75%.
Question 29
• You perform a total parotidectomy on this patient from which he
seems to progress from well initially. At a 3 month follow up, he
states that he experiences redness and sweating over his cheek and
near his ear when he eats or even thinks about food. Which of the
following is most likely to be true?
– A. You should obtain a CT of his head and neck to evaluate for tumor
recurrence
– B. It is caused by transection of the facial nerve
– C. If the symptoms easily replicate by having him eat a lemon slice, you can
consider Botox injection to see if that relieves the symptoms
– D. It is most likely secondary to injury and inappropriate regeneration of
the auriculotemporal branch of the trigeminal nerve
– E. C and D only
– F. All of the above
Answer 29
• E. C and D are correct
Question 30
• A 50 yo male that underwent a Billroth II gastric resection for
gastric outlet obstruction 1 month ago presents for follow-up.
He states that he experiences nausea, occasional nonbilious
emesis, and dizziness about 2 hours after he eats. Which of
the following is most likely?
– A. If you were to check his glucose when the symptoms occur, it
would most likely be high
– B. This usually does not improve with conservative treatment so you
should anticipate converting him to a Roux-en-Y
– C. Octreotide may be helpful
– D. This is most likely due to retained antrum so you will need to
consider repeat resection
Answer 30
• C. This describes late dumping syndrome and
may respond to octreotide
Dumping Syndrome
• Gastrointestinal and vasomotor symptoms
• Thought to be due to unmetered entry of ingested food
into SB
• May be seen after vagotomy & resection or division of
pyloric sphincter
• Early sx- immediately after meal from hyperosmotic
load into the small bowel
– Nausea, epigastric discomfort, borborygmi, palpitations, dizziness
• Late- 1-3 hrs after meal
– Reactive hypoglycemia in addition to sx of early dumping
Dumping Syndrome
• Usually responsive to dietary modifications (small, low-
fat diet, limit liquids with meals)
• Octreotide- can improve symptoms
– 50-100 mg before meal
– Inhibit release of vasoactive peptides from gut
– Decrease peak insulin levels and slow intestinal transit
– Prevents changes in pulse, systolic BP, and RBC volume during early dumping and
blood glc levels during late dumping
Question 31
• A 50 yo male that underwent a subtotal gastrectomy with BII
reconstruction for a T2 adenocarcinoma 1 month ago presents for
follow-up. He experiences frequent epigastric pain after meals with
subsequent bilious vomiting that does not relieve the pain. He has
lost about 15 pounds though otherwise has been stable. Which of
the following is false?
– A. He will likely respond well with dietary modification and bile chelators
– B. This usually does not improve with conservative treatment so you should
anticipate converting him to a Roux-en-Y
– C. This is most likely due to retained antrum so you will need to consider
repeat resection
– D. All of the above
– E. A and C only
Answer 31
• B. This is consistent with alkaline reflux which
is less likely to respond to conservative
treatment and will ultimately need conversion
to a Roux-en-Y
Alkaline Reflux Gastritis
• Postprandial epigastric pain, nausea, vomiting of bile,
evidence of bile reflux into stomach with gastritis
• Persist in 1-2% of patients who have had vagotomy/
drainage or resection
• Differential dx- recurrent ulceration, biliary or pancreatic
disease, afferent loop obstruction, esophagitis
• Gastric acid analysis- basal hypochlorhydria with little
increase w/ pentagastrin administration
• Obtain serum gastrum measurements to r/o ZE syndrome
and retained antrum
Alkaline Reflux Gastritis
• Endoscopy- red, friable mucosa, edema, glandular
atrophy, intestinal metaplasia
• Antacids, H2 lockers, bile acid chelators, dietary
manipulation not definitively beneficial
• Reoperation to divert contents from gastric mucosa
– Convert to Roux-en-Y GJ with 50-60 cm intestinal limb
– Very effective in eliminating bilious vomiting
– 20-30% still have pain or persistent gastric emptying
dysfunction
Question 32
• You perform a subtotal gastrectomy with B2 reconstruction on
a 62 yo man with an antral GIST tumor. On postoperative day
1 he is experiencing severe abdominal pain and nonbilious
emesis. You obtain imaging which is concerning for a
markedly dilated bowel limb proximal to the anastomosis.
Which of the following is true?
– A. This is most likely secondary to anastomtic edema so you should
place an NGT and start octreotride
– B. You would be less likely to have this complication if you did an
antecolic gastrojejunostomy
– C. This requires urgent operative intervention
– D. In the chronic form (from partial obstruction) patients may develop
microcytic anemia
– E. B, C, and D
Answer 33
• C. This picture is concerning for afferent limb
obstruction- the patient needs intervention as
he is high risk for duodenal stump blowout.
Afferent Limb Syndrome
• Partial or complete obstruction of the afferent limb
proximal to the GJ anastomosis
• Afferent limb- duodenal remnant and segment of
jejunum proximal to gastrojejunostomy
• Jejunal limb is subject to adhesions, herniaton,
obstruction, volvulus
• Rare (~1%) after Billroth II
• Can occur immediately or remotely
– Acute- operative emergency; secretions remained trapped in limb so vomiting is
nonbilious
– Chronic- from partial obstruction- relief of postprandial discomfort with
projectile bilious vomiting
Afferent Limb Syndrome
• One of main causes of duodenal stump blowout
• Prolonged stasis in chronic disease can lead to bacterial
deconjugation of bile salts-> steatorrhea, malnutrition, Vit
B12 deficiency-> megaloblastic anemia
• Pts with increased risk of developing syndrome
– Jejunal portion of afferent limb longer than 30 cm
– Antecolic GJ
– Improper closure of mesocolic defects
• Surgical intervention usually necessary
– Some attempting endoscopic therapy, though not routine
Question 34
• During the workup for epigastric discomfort that began about 6 months
ago, you find a 4 cm mass in the mid-body of the stomach. Biopsy is
consistent with low-grade GIST. He otherwise is healthy with no prior
abdominal operations. Which of the following is the next best step in
management?
– A. Preoperative multiagent chemotherapy and radiation followed by resection 6
weeks after treatment
– B. Wedge resection of the gastric lesion
– C. Subtotal gastrectomy with Billroth 2 reconstruction
– D. Treatment with Gleevec
Answer 34
• B. Wedge resection is most appropriate
Characteristics/ Management of GIST

• GISTs are malignant soft tissue tumors in sarcoma


family
• Characterized by mutation in c-KIT (95%) and in
platelet-derived growth factor (PDGF)
• Gleevec- targeted inhibitor aimed at c-KIT
• They don’t go to nodes since they are in sarcoma
family
• Usual spread is to liver, lungs and peritoneum
Characteristics/ Management of GIST
• Treatment- primarily surgical primarily
• Do not need huge margins (wedge resections acceptable)
• Mesenteric resection can be limited since they don’t usually spread to nodes
• Risk factors for recurrence are size >5cm and grade (>5 mitoses/high power
field).
• Risk categories categorized into:
• Low (<5cm and low grade)
• Intermediate (<5cm/high grade or 5- 10cm/low grade)
• High (>5cm/high grade or >10cm)
Characteristics/ Management of GIST

• Intermediate and high risk tumors receive


adjuvant Gleevec for 1 year after resection
• Low risk get surgery only.
• Trend is toward organ preservation- in cases
involving multiple organs, Gleevec is given to
shrink tumor (>90% will respond) and later
operate with intent to preserve as much as
possible
Question 1 (of 3)
• The primary nutrition source for colonocytes
is:
– A. Glutamine
– B. Short chain fatty acids
– C. Glucose
– D. Alpha-ketoglutarate
Question 2 (of 3)
• The primary fuel source for small bowel is:
– A. Glutamine
– B. Short chain fatty acids
– C. Glucose
– D. Alpha-ketoglutarate
Question 3
• The primary fuel source for cancer cells is:
– A. Glutamine
– B. Short chain fatty acids
– C. Glucose
– D. Alpha-ketoglutarate
Answer 1-3
• 1. B; 2. A; 3. A
• The primary fuel source for colon cells are short
chain fatty acids. Occasionally patients with a
Hartmann’s pouch can get a disuse proctitis that can
be treated with SCFA’s.
• The primary source of nutrition for small bowel cells
is glutamine. It is thought that tube feeds with
glutamine may help decrease gut translocation in
patients with sepsis, trauma, etc.
• Glutamine also is the primary fuel source for cancer
cells.
Question 4
• During prolonged starvation the brain switches
from using glucose to using:
– A. Glutamine
– B. Short chain fatty acids
– C. Arginine
– D. Ketones
Answer 4
• D. The brain switched from glucose to ketones
after prolonged starvation
Question 5
• The p53 tumor suppresor gene is primarily
involved in
– A. Cell cycle regulation
– B. Cell adhesion
– C. Angiogenesis
– D. Chemotaxis
Answer 5
• A. P53 is primarily involved in cell cycle
regulation and apoptosis
Question 6
• Hyperacute rejection following organ
transplantation is most often due to:
– A. ABO incompatibility
– B. Rh incompatibility
– C. Previously sensitized T cells
– D. Macrophages
Answer 6
• A. Hyperacute rejection is most often due to
ABO incompatibility and involves pre-formed
antibodies of the recipient to donor antigens
Question 7
• Hyperacute rejection is an example of
hypersensitivity reaction:
– A. Type 1
– B. Type 2
– C. Type 3
– D. Type 4
Answer 7
• B. Hyperacute rejection is an example of a
Type II hypersensitivity reaction
Question 8
• New proteinuria in a pt following kidney
transplant is most consistent with:
– A. Acute rejection
– B. UTI
– C. Chronic rejection
– D. Renal vein thrombosis
Answer 8
• D. New proteinuria is most consistent with
renal vein thrombosis
Question 9
• The maximum amount of collagen in a wound
occurs at:
– A. 1 week
– B. 3 weeks
– C. 8 weeks
– D. 3 months
Answer 9
• B. Maximum collagen accumulation occurs at
2-3 weeks. After that the amount of collagen
stays the same but continued cross-linking
improves strength
Question 10
• Diaphragm injuries are:
– A. Most common on the right
– B. Most common on the left
– C. Occur most commonly with penetrating injury
– D. Are easy to find with CT
Answer 10
• B. Diaphragm injuries are more common with
blunt trauma and on the left as the liver
protects the right side.
• Diaphragm injuries can be hard to find on CT
unless there is gross herniation
Question 11
• A 22 yo man suffers a GSW to the left flank. He
is hypotensive but has a pulse. You start to
bolus fluid. The next step should be:
– A. Angiography
– B. CT scan
– C. Left thoracotomy
– D. Laparotomy
Answer 11
• D. Laparotomy. This patient is hypotensive
with a penetrating wound to the abdomen. He
needs to go straight to the OR.
Question 12
• You perform a laparotomy in the previous pt
and find a large left sided retroperitoneal
hematoma that extends above the celiac
artery. The patient is hypotensive after 6 units
of blood. The next step should be:
– A. Mattox manuever
– B. Cattell manuever
– C. Pelvic embolization
– D. Infra-diaphragmatic control of the aorta
Answer 12
• D. Because the hematoma extends above the celiac artery
a Mattox maneuver is not going to give you proper
exposure.
• Either perform a left thoracotomy and clamp the aorta or
place an infradiaphragmatic clamp across the aorta until
you get control of the situation.
• * Mattox maneuver- retroperitoneal mobilization and
medial rotation of all the left-sided viscera to expose
retroperitoneal vessels
• * Cattell- take down cecum though hepatic flexure @
white line then mobilize bowel medially to expose
retroperitoneal vasculature
Question 13
• You clamp the infradiaphragmatic aorta , perform a
Mattox maneuver, and find the left renal vein is
completely avulsed from the IVC. The next step
should be:
– A. Replace the infradiaphragmatic clamp with a
suprarenal clamp, oversew the renal vein, and ligate IVC
connection
– B. Replace the infradiaphragmatic clamp with a
suprarenal clamp, perform primary repair
– C. Left nephrectomy
– D. Pack and go to ICU
Answer 13
• A. The left renal vein has the adrenal vein and
gonadal vein collaterals making ligation of the
left renal vein safe.
• The right renal vein does not have these
collaterals so in this case you would have to
perform a nephrectomy if the pt was unstable
or a re-anastomosis.
Question 14
• All of the following are true about aldosterone
except:
– A. Causes resorption of Na
– B. Causes excretion of H and K ions
– C. Acts at the distal convoluted tubule
– D. Primarily acts on protein kinase C
Answer 14
• D. Aldosterone causes resorption of Na and
excretion of H and K at the DCT by stimulating
the transcription of Na/K ATPase and NA/H
ATPase. This effectively causes more water
reabsorption
Question 15
• All of the following are true about nitric oxide
except:
– A. Increases cAMP
– . The precursor is arginine
– C. Is primarily released from vascular endothelium
– D. The receptor is guanylate cyclase
Answer 15
• A. Arginine is the precursor to nitric oxide. NO
acts on guanylate cyclase to increase cGMP
and cause vasodilation
Question 16
• A 10 yo boy has a cyst in his lateral neck medial
to the anterior border of the SCM muscle.
This cyst most likely connects to the:
– A. External auditory canal
– B. The tonsillar pillar
– C. The nasal septum
– D. Thoracic duct
Answer 16
• B. Type II branchial cleft cysts extend from the
anterior border of the SCM through the carotid
bifurcation, to the tonsillar pillar
Question 17
• The most common branchial cleft cyst is:
– A. Type 1
– B. Type 2
– C. Type 3
– D. Type 4
Answer 17
• B. Type II is the most common type of
branchial cleft cyst
Question 18
• Treatment of branchial cleft cysts involves
– A. Antibiotics
– B. Resection
– C. XRT
– D. Chemotherapy
Answer 18
• B. Treatment of branchial cleft cysts involves
resection
Question 19
• The blood supply to the parathyroid glands is:
– A. Superior thyroid artery for both glands
– B. Inferior thyroid artery for both glands
– C. Superior a. for the superior glands; inferior a. for
the inferior glands
– D. Transverse cervical artery for both glands
Answer 19
• B. The blood supply to the superior and
inferior parathyroid glands is the inferior
thyroid arteries.
Question 20
• A 55 yo man undergoes a right pneumonectomy for
lung cancer. 6 days postop he develops fever,
serosanguinous sputum, and the air-fluid level in
the post-pneumonectomy space is lower on the
CXR. There is also new infiltrate in the left lower
lobe. The most appropriate step is:
– A. Bronchoscopy
– B. Chest CT
– C. V/Q scan
– D. Antibiotics
Answer 20
• A. The combination of serosanguinous sputum
production, change in air-fluid level, and a new
infiltrate is highly suggestive of a broncho-
pleural fistula.
• The new infiltrate is formed from aspiration
into the remaining lung of the residual fluid in
the post-pneumonectomy air space
Answer 20
• Bronchopulmonary fistula are more common
after pneumonectomy compared to lobectomy
• The best way to diagnose the problem is
bronchoscopy
Question 21
• An 18 yo female is referred by her PCP secondary to RLQ
abdominal pain that started 5 days ago. Her temp is 39C, WBC
18, urine BHCG is negative, and GYN exam is negative. A CT A/P
shows a large phlegmon in the RLQ. Which of the following is
the most appropriate treatment?
– A. Ex lap via a midline incision with drainage of the abscess and
appendectomy
– B. RLQ incision with drainage of the abscess and appendectomy
– C. Percutaneous drainage of the abscess with antibiotics; defer
appendectomy for 6-8 weeks after resolution
– D. IV antibiotics only
– E. Diagnostic laparoscopy with abscess drainage, appendectomy, and
drain placement
Answer 21
• C. Small abscesses can be treated with
antibiotics, hydration, and NPO status.
• Large phlegmons should be treated with
percutaneous drainage, antibiotics, and
interval appendectomy.
• Complex or non-contained abscesses should be
treated surgically.
Question 22
• Following a workup for hematuria, a 63 yo man is
found to have a renal cell carcinoma of the right
kidney. The finding least likely to affect this
patient’s survival adversely is:
– A. Tumor thrombus that extends to the junction of the
renal vein and IVC
– B. Single metastatic focus in the lung
– C. Tumor invasion into the renal vein
– D. Metastases to regional lymph nodes
– E. Direct extension of the tumor into the duodenum
Answer 22
• A. Renal cell carcinoma is poorly responsive to
chemotherapy or radiation
• Surgical excision is best hope for cure
• In 4-10% of pts, a tumor thrombus extends into the
lumen of the renal vein, and occasionally the IVC in right
sided tumors
• 5-year survival rates reported between 47-84% is the
tumor thrombus is removed without leaving residual
disease and the vessel wall is uninvolved.
Characteristics of RCC
• Tends to have hematogenous mets
• Metastatic disease worsens prognosis (very poor)
• Some evidence supports resection of single mets (lung
most common) though overall cure rate not affected
• Direct extension into Glisson’s capsule into surrounding
structures has a poorer prognosis- 5 year survival less
than 5%
• LN involvement also has a poorer prognosis (5 yr or 5-
30%)
Question 23
• A 34 yo man is brought in with a GSW to the back.
He has 5/5 strength un both upper extremities and
in the LLE. There is decreased sensation in the left
leg. The right leg has normal sensation but 2/5
strength. Which of the following conditions
describes this injury?
– A. Central cord syndrome
– B. Anterior cord syndrome
– C. Brown-Sequard syndrome
– D. Cauda equina syndrome
– E. Complete cord syndrome
Answer 23
• C. Brown-Sequard
• Incomplete spinal cord syndromes have variable
prognosis for recovery
• Greater recovery expected in patients in who there is
greater initial sparing of function below the level of
injury
• Brown-Sequard is incomplete/ hemitransection of the
cord usually from penetrating trauma
– Results in ipsilateral motor and propriorecpetion loss below the level of injury
and contralateral loss of pain and temperature beginning 1-2 levels below the
injury
– Has the best prognosis of all incomplete syndromes
Question 24
• A 75 yo female with longstanding osteoarthritis and
cervical scoliosis is brought in after a motor vehicle
accident. You note that she has 3/5 weakness in
both arms though her LE strength is 5/5 on both
sides. Her rectal tone is normal. Which of the
following injuries is this most consistent with?
– A. Anterior spinal syndrome
– B. Central cord syndrome
– C. Brown Sequard syndrome
– D. Cauda equina syndrome
Answer 24
• B. This is most consistent with central cord
syndrome
Incomplete Spinal Cord Injury Syndromes

• Central cord- occurs essentially in the cervical spine


– Injury to central portion of cord
– Results in sacral sparing (preservation of perianal sensation and tone) and
greater weakness in the UE’s with sparing of the motor function in the LE’s
– Reflects the topographic organization of motor tracts within the spinal cord in
which the upper extremity tracts are located in a more central position within
the cord
– Carries the second best prognosis for recovery with the lower extremities often
recovering better function than the upper
Spinal Cord Injury Syndromes
• Anterior cord syndrome-
– Results from damage to the anterior spinal artery
– Involves damage to the anterior two-third of the cord and sparing of
the posterior columns
– Patients have minimal (if any) motor function distally
– Pain and temperature sensation is lost as well
– Proprioception, deep pressure, and vibratory sensation are preserved
– Prognosis for motor recovery is poor
• Cauda equina syndrome-
– Involves injury to the lumbosacral nerve roots within the spinal canal
– Results in an areflexic bladder, bowel, and lower limbs
Question 25
• The resting energy requirements of a 70 kg
man are approximately (kcal/day)
– A. 1400-1500
– B. 1600-1700
– C. 1800-1900
– D. 2000-2100
– E. 2200-2300
Answer 25
• A. The average resting post-absorptive 7- kg man
consumes oxygen at a rate of about 200 ml/mn or
288 L/day. This equals about 1450 kcal/day.
• In general, energy needs increase as illness severity
increases
• Uncomplicated postop requirements- 1500-1700
• Sepsis- 2000-2400
• Multitrauma and mechanical ventilation- 2200-2600
kcal/day
• Major burn- 2500-3000
Question 26
• A 66 yo F with compensated cirrhosis due to
hepatitis C undergoes laparoscopic sigmoid
resection for a diverticular stricture.
Postoperatively her sodium falls from 128 to 110
meQ though she is asymptomatic. Which of the
following is the best initial treatment plan for this
patient?
– A. 1 liter of 3% NS given over 12 hours
– B. 60 mmol NaPhos IV over 6 hours
– C. Fluid restriction
– D. Spironolactone
Answer 26
• C. Fluid restriction is most appropriate in this
scenario
Question 27
• Which structure is most often injured when
performing a 4 compartment fasciotomy?
– A. Dorsalis pedis artery
– B. Peroneal nerve
– C. Sural nerve
– D. Achilles tendon
Answer 27
• B. The peroneal nerve is the most frequently
injured structure
Question 28
• You are placing a central line in the ICU via the right
subclavian vein. After you puncture the vein and reach
for the wire, the patient quickly takes a large breath and
becomes hypotensive. You are concerned he has an air
embolus. Which of the following management plans is
most appropriate?
– A. Continue line placement and get a STAT CXR
– B. Continue line placement and attempt to aspirate the bubble
– C. Place the patient in trendelenberg with his left side down
– D. Place him on 4 L O2 per nasal cannula for 24 hours
Answer 28
• C. When air embolism is suspected, place the
patient in Trendelenberg with left side down
Question 29
• The defect that causes familial hypercalcemic
hypocalcuria is:
– A. Unresponsive receptor in kidney to PTH
– B. Production of PTH molecule that doesn’t work
– C. Defect in distal convoluted tubule that causes
calcium resorption
– D. Defect in renal conversion of vitamin D
Answer 29
• C. Familial hypercalcemic hypocalcuria is due
to a defect in the distal convoluted tube that
causes calcium resorption
Tertiary Hypoparathyroidism
• Renal disease corrected with transplant
• PTH still overproduced
• 3 ½ gland parathyroidectomy or 4 gland
removal with autotransplantation usually cures
Familial Hypercalcemic Hypocalcuria

• Increased serum calcium and low urinary calcium


(should be high if hyperparathyroidism)
• Defect in receptor of distal convoluted tubule
that causes increased resorption of calcium
• Calcium levels usually 9-11; normal PTH, low
urinary calcium
• No treatment, no surgical indication
Pseudohyperparathyroidism
• Defect in kidney PTH receptor
• No response to PTH
Parathyroid Cancer
• Rare cause of hypercalcemia
• 50% 5 year survival
• Hypercalcemia causes mortality
• Increased calcium, PTH, alk phos
• Lung most common site for mets
• Wide en bloc resection
• 50% recur
Multiple Endocrine Neoplasia
• APUD cells
• Synchronously or metachronously occurring
lesions
• Autosomal dominant, 100% penetrance,
variable expression
Question 30
• Which of the following is appropriately matched?
• A. MEN1- prolactinoma, gastrinoma,
pheochromocytoma
• B. MEN II a- pheochromocytoma, parathyroid
hyperplasia, medullary ca thyroid
• C. MEN IIb- pheochromocytoma, gastrinoma,
medullary ca thyroid
• D. MEN II a- neuroma, medullary thyroid ca,
pheochromocytoma
Answer 30
• B correctly describes MEN IIa
MEN-1
• Parathyroid hyperplasia- Usually first to
become symptomatic, urinary sx, 4 gland rsxn
with autotransplantation
• Pancreatic Islet Cell tumors; gastrinoma #1; 50%
multiple, malignant major morbidity
• Pituitary adenoma; most common prolactinoma
• Need to correct hyperparathyroidism first
MEN 2a
• Parathyroid hyperplasia
• Pheochromocytoma; often bilateral, benign
• Medullary cancer of thyroid
– Nearly all patients, bilateral
– Diarrhea common; usually 1st part to be
symptomatic
– #1 cause of death
• Need to correct pheochromocytoma first
MEN 2b
• Pheochromocytoma (bilateral, benign)
• Medullary cancer of thyroid
– Nearly all patients, diarrhea most common symptom
– Bilateral
– #1 cause of death
• Mucosal neuromas
• Marfan’s habitus, musculoskeletal abnormalities
• Need to correct pheochromocytoma first
Question 31
• Gene associated with MEN 1?
• MEN 2?
Answer 31
• MEN 1- MENIN gene
• MEN 2- RET protooncogene
Hypercalcemia
• Hyperthyroidism, Hyperparathyroidism
• Immobilization, thiazide diuretics
• Excess vitamin D, milk-alkali syndrome
• Granulomatous disease
• Malignancy
– Hematologic 25%- Lytic bone lesions
– Nonhematologic- 75%- Cancers that release PTHrp
(small cell, breast)
Hypercalcemia
• Mithramycin- inhibits osteoclasts; significant side
effects
• Hypercalcemic crisis
• Breast cancer mets to bone release PTHrp->
hypercalcemia is not due to bony destruction
– High urinary cAMP (PTH action on kidney)
• Hematologic malignancies can cause bony
destruction
– High serum Ca and low urine cAMP
Question 32
• Which of the following lies anterior to the
scalene anticus muscle?
– A. Subclavian artery
– B. Brachial plexus
– C. Vagus nerve
– D. Subclavian vein
Answer 32
• D. Subclavian vein
Question 33
• A 40 yo woman undergoes a difficult
choledochojejunostomy secondary to a biliary stricture
from CBD injury after laparoscopic cholecystectomy 6
months ago. You examine her postoperatively and find
her abdomen markedly distended. She is in pain but
does not have peritonitis. Her BP is 85/48 mmHG.
Which of the following is the most likely cause of these
symptoms?
– A. Biliary leak
– B. Acute gastric distention
– C. Missed small bowel injury
– D. Obstructed foley catheter
Answer 33
• B. This is most consistent with acute gastric
distention.
• Remember, in the post-op period (most often
around 12 hours) abdominal distention with
pain and hypotension is likely due to acute
gastric dilatation and should be treated with
decompression
Question 34
• A 25 yo female with RLQ pain undergoes a
diagnostic laparoscopy for suspected appendicitis.
Her appendix appears normal but you note a 4.5
cm cyst on her right ovary that has clear fluid.
Which of the following is the most appropriate
management plan?
– A. Remove her appendix, follow the cyst with an
ultrasound in 3 months
– B. Aspirate the cyst
– C. Right salpingo-oophorectomy
– D. Right partial oophorectomy
Answer 34
• B. It is important the cyst be aspirated
Question 35
• Antibiotic prophylaxis in patients undergoing
exploratory laparotomy for gunshot wounds to
the abdomen has been shown to:
– A. Lower the risk of anastomotic leaks
– B. Lower midline wound infections
– C. Decrease the occurrence of intraabdominal
abscesses
– D. Improve survival
Answer 35
• B. Prophylactic abx decrease the risk of
midline wound infections
Question 36
• Which of the following in the Glasgow Coma
Scale best predicts immediate recovery after
injury?
– A. Ability to spontaneously open eyes
– B. Good verbal communication
– C. Full motor response
– D. None of the above
Answer 36
• C. Best motor response is the most predictive
factor for immediate recovery
Question 1
• The “Line of Sappey” is:
– A. 2 cm below the umbilicus
– B. In the axilla
– C. Supraclavicular
– D. 2 cm above the umbilicus
Answer 1
• D. 2 cm above the umbilicus
Line of Sappey
• Is the line 2 cm above the umbilicus to L 2-3
• Generally, melanoma drains to axillary nodes
above this line or to ilioinguinal nodes below
this line
Question 2
• 2 weeks s/p Whipple, a 62 yo female develops
fever, chills, and hypotension. Her blood
cultures grow E. coli. What is the key stimulant
involved in this condition?
– A. TGF- beta
– B. Lipid A
– C. IL-2
– D. Protein C
Answer 2
• B. Lipid A is a part of the LPS complex found in
E. coli sepsis
• It is responsible for the release of TNF- alpha
involved in inflammation and sepsis
Question 3
• Pyriodostigmine is:
– A. Cholinesterase inhibitor
– B. Succinylcholine inhibitor
– C. Steroid inhibitor
– D. Beta blocker
Answer 3
• A. Pyridostigmine is a cholinesterase inhibitor
• It increases the amount of ACH and is used to
treat myasthenia gravis
Question 4
• Trimethoprim inhibits:
– A. Purine synthesis
– B. Small subunit of ribosome
– C. Large unit of ribosome
– D. DNA gyrase
Answer 4
• A. Trimethoprim inhibits purine synthesis by
inhibiting dihydrofolate reductase
Question 5
• Cyclosporine binds to:
– A. IL-2 receptor
– B. Cyclophilin
– C. INF-gamma
– D. FK binding protein
Answer 5
• B. Cyclophilin
• Cyclosporine binds to the cyclophilin protein
and forms a complex that binds to calcineurin.
• It inhibits IL-2 transcription and prevents T-cell
activation
• Zenepax is an antibody against IL-2
• Tacrolimus binds to FK binding protein
Question 6
• Oxygen is bound more tightly to hemoglobin
when there is a:
– A. Increase in CO2
– B. Increase in temperature
– C. Decrease in H+ ion concentration
– D. Increase in levels of 2,3 DPG
Answer 6
• C. Decrease in H+ ion concentration, or more
alkalotic condition, will cause oxygen to be more
tightly bound to Hgb.
Question 7
• The organism found specifically in human bites
is:
– A. Pasteurella multocida
– B. Staph aureus
– C. Eikinella
– D. Strep pyogenes
Answer 7
• C. Eikinella is an organism found only in
human bites.
• Pasteurella multocida is the most common
agent in cat and dog bites
Question 8
• Which of the following is the most common
organism responsible for spontaneous
bacterial peritonitis?
– A. Staph aureus
– B. E. coli
– C. Strep pyogenes
– D. Bacteroides fragilis
Answer 8
• B. E. coli is the most common causative
organism of SBP.
Question 9
• BRCA 1 is associated with a:
– A. 40% lifetime risk of ovarian cancer
– B. 30% lifetime risk of ovarian cancer
– C. 20% lifetime risk of ovarian cancer
– D. 10% lifetime risk of ovarian cancer
Answer 9
• BRCA 1 gene together with family history has a
60% lifetime risk of breast cancer as well as a
40% risk of ovarian cancer and 1% risk of male
breast cancer
• BRCA 2 is a/w a 10% risk of ovarian cancer and
10% of male breast cancer
Question 10
• The brachial plexus is located:
– A. Posterior to the middle scalene muscle
– B. Anterior to the middle scalene muscle
– C. Anterior to the anterior scalene muscle
– D. Posterior to the posterior scalene muscle
Answer 10
• A. B. Anterior to the middle scalene muscle
Scalene Anatomy
Question 11
• A 72 yo man with a history of TIA’s is found to
have 80% stenosis of the left carotid on
ultrasound. The most appropriate next step is:
– A. Repeat ultrasound in 3 months
– B. Initiate plavix and aspirin
– C. Start aspirin only
– D. Recommend carotid endarterectomy
Answer 11
• D. Recommend carotid endarterectomy
• Patients with symptomatic carotid disease and
stenosis greater than 60% should have CEA
• Asymptomatic patients with more than 70%
stenosis should be considered for CEA
Question 12
• The left gastric artery is a branch off of the:
– A. Celiac artery
– B. Splenic artery
– C. Common hepatic artery
– D. SMA
Answer 12
• A. The celiac trunk
Blood supply of stomach
Question 14
• A broncholith, or calcified granuloma, may be
found in which of the following conditions?
– A. Actinomycosis
– B. Coccidiomycosis
– C. Blastomycosis
– D. Histoplasmosis
Answer 14
• D. Histoplasmosis is associated with calcified
granulomas on presentation
Question 15
• A 67 yo male with a 4 cm mass in the left lower
lung is shown to be adenocarcinoma on biops6y.
Mediastinoscopy also shows paratracheal lymph
node involvement on the left side. The most
appropriate management plan at this time would
be:
– A. Chemoradiation
– B. Left LL resection and mediastinal LN dissection
– C. Left pneumonectomy and mediastinal LND
– D. Left LL resection only
Answer 15
• A. Chemoradiation. Being that the patient has
+ paratracheal nodes, the disease is
unresectable.
Question 16
• A 79 yo male from a nursing home undergoes a lap
chole and requires increased narcotics to control
his pain. 4 days later he is brought back because of
abdominal distention and worsening pain diffuse.
Films demonstrate a distended colon extending
through the cecum. Which of the following should
be done next?
– A. Colonoscopy
– B. Cecostomy
– C. Serial abdominal exams
– D. Total abdominal colectomy
Answer 16
• A. Colonoscopy should be attempted first
given that the patient most likely has Ogilves
associated with recent opiate use. Treatment
should include decompressive cscope,
discontinuing drugs that inhibit intestinal
motility, and checking electrolytes.
Question 17
• 1 hour after CEA, a 66 yo male is found to be
substantially hoarse. Which of the following is
the most appropriate step in management?
– A. Observation
– B. MRI of head and neck
– C. OR for re-exploration
– D. Angiography
Answer 17
• C. The patient should be taken back to the OR
as hoarseness immediately after surgery is
suggestive of a nerve injury
Question 18
• A 66 yo male undergoes an Ivor-lewis
esophagectomy for a distal esophageal
adenocarcinoma. POD #1 he becomes febrile,
tachycardic, and has a WBC of 22. You obtain a
contrast study which shows an anastomotic leak.
Which of the following is the next best step?
– A. Ask for an esophageal stent to be placed
– B. Re-explore
– C. Place a chest tube and keep NPO
– D. Place a chest tube and esophageal stent
Answer 18
• B. A leak on POD #1 would best be treated by
reexploration
Question 19
• A 56 yo female has crampy RUQ and epigastric
abdominal pain that started about 3 days ago.
Films demonstrate multiple air-fluid levels in the
small bowel with distention. You also note
pneumobilia. Which of the following should you do
next?
– A. Start broad spectrum antibiotics
– B. Request percutaneous cholecystotomy tube placement
– C. Order ERCP
– D. Plan for exploratory laparotomy
Answer 19
• D. This patient presents with what is most
likely gallstone ileus.
• Pneumobilia, crampy abdominal pain, and
bowel obstruction are characteristic
• It is most appropriate to explore the patient
and remove the stone
Question 20
• A 52 yo male undergoes embolectomy for a clot to
the right lower extremity. 4 hours post-op his leg
becomes swollen and painful. It feels tight to
palpation. Which of the following is the next best
step?
– A. Take back to the OR for angiography and possible graft
revision
– B. Perform a femoral-tibial bypass
– C. Initiate a heparin drip
– D. Perform a 4 compartment fasciotomy
Answer 20
• D. This patient is having a reperfusion injury
resulting in compartment syndrome so he
needs a fasciotomy
Question 21
• A left to right cardiac shunt is seen in which of
the following conditions?
– A. Atrial septal defect
– B. Tetralogy of Fallot
– C. Truncus arteriosus
– D. Tricuspid atresia
Answer 21
• A. Atrial septal defect is associated with left to
right shunt and in most cases closes
spontaneously by age 1.
• Right to left shunt is associated with :
– TOF
– Truncus arteriosus
– Tricuspid atresia
Question 22
• During a laparoscopic cholecystectomy you
perform a cholangiogram and believe that you
have clipped across the distal hepatic duct.
The next most appropriate step is:
– A. Open end-to-end anastomosis
– B. Choledochojejunostomy
– C. Hepaticojejunostomy
– D. Whipple procedure
Answer 22
• C. Hepaticojejunostomy
• An end-to-end anastomosis in this setting can
lead to strictures
Question 23
• The most common symptom after a pulmonary
embolism is:
– A. Cough
– B. Hemoptysis
– C. Pleural pain
– D. Dyspnea
Answer 23
• D. Dyspnea is the most common symptom associated
with pulmonary embolism
Question 24
• Epiphrenic diverticula are:
– A. Located in the proximal esophagus
– B. Are true diverticula
– C. Commonly are asymptomatic
– D. Are associated with achlasia
Answer 24
• D. Epiphrenic diverticula are usually asymptomatic
and located in the distal 1/3 of the esophagus.
• They are commonly associated with achlasia
Question 25
• Maltose contains which of the following?
– A. Galactose
– B. Fructose
– C. Glucose
– D. Cellulose
Answer 25
• C. Maltose contains glucose
Question 26
• A 30 yo presents with a BP of 220/110 mmHg
and complaining of headaches. The most
appropriate test to begin his evaluation should
be:
– A. Check a renin level
– B. MIBG scan
– C. 24 hour urine VMA and metanephrine
– D. TSH
Answer 26
• C. 24 hour urine VMA and metanephrine
• This patient may have a pheochromocytoma and the
best test is urine CMA and metanephrine
• MIBG is the most sensitive test for localizing a pheo.
Question 27
• Cortisol releasing hormone is released from
the :
– A Pituitary
– B. Thyroid
– C. Hypothalamus
– D. Adrenal
Answer 27
• C. The hypothalamus releases GhRH, GnRH,
CRH, and TRH
Question 28
• Which of the following genes is associated with
Von Hippel-lindau disease?
– A. APC
– B. VHL
– C. BRCA 2
– D. P53
Answer 28
• B. VHL gene
• VHL is a tumor suppressor gene involved in
Von Hippel-Lindau disease.
• This disease is a/w pheochromocytoma, renal
angioma, renal cell carcinoma, and
hemangioblastoma
Question 28
• The first line of therapy for ITP is:
– A. Gamma globulin
– B. Splenectomy
– C. Platelet transfusion
– D. Steroids
Answer 28
• D. Steroids are considered as primary therapy
for ITP
Question 29
• Which of the following is true of nitric oxide
(NO)?
– A. It increases cGMP level
– B. It is released from lymphocytes
– C. It is released from platelets
– D. Glutamine is its precursor
Answer 29
• A. NO is produced in endothelial cells and its
release causes increase of cGMP levels.
• This subsequently leads to vasodilation
• Arginine is the precursor to NO
Question 30
• Cantrell’s pentalogy does NOT involve which of
the following?
– A. Pericardial defects
– B. Diaphragmatic defects
– C. Sternal defects
– D. Liver defects
Answer 30
• D. Liver defects.
• Cantrell’s pentaology involves omphalocele,
diaphragmatic, sternal, pericardial, and cardiac
defects
Question 31
• Regarding hormone receptors and breast
cancer, which of the following is true?
– A. Both estrogen (ER) and progesterone (PR)
positive tumors have a worse prognosis
– B. ER tumors have a worse prognosis
– C. HER tumors have a poorer prognosis
– D. Progesterone receptor positive tumors have a
poorer prognosis than PR negative tumors
Answer 31
• C. HER receptor tumors have a poorer
prognosis
• ER and PR positivity improves prognosis
Question 32
• A 49 yo male vomits multiple times after drinking
heavily. Following this he develops severe chest
pain and presents to the ER. You obtain a chest XR
which shows a left sided pleural effusion and PTX.
Which of the following is the most likely location of
the perforation?
– A. Right posterior middle esophagus
– B. Left posterior middle esophagus
– C. Left posterior upper esophagus
– D. Left posterior lower esophagus
Answer 32
• D. This patient likely has a Boerhaave’s rupture
of the esophagus
• The most common location is the posterior left
aspect of the lower esophagus
Question 33
• A 65 yo man presents with an anal mass. You
biopsy the lesion and pathology shows that it is
a squamous cell carcinoma. Which of the
following is the next most appropriate step?
– A. Abdominoperineal resection
– B. Low anterior resection
– C. Chemoradiation
– D. Wilde excision
Answer 33
• C. Chemoradiation is the best first treatment
for SCC of the anus
Question 34
• A 55 yo man with epigastric pain that is not
responsive to PPI’s undergoes an EGD. A mass
is noted in the stomach and biopsied. The
biopsy comes back as lymphoproliferative
tissue. The next most appropriate therapy is:
– A. Chemoradiation
– B. Radiation only
– C. Amoxicillin, tetracycline, and PPI therapy
– D. Total gastrectomy
Answer 34
• C. The appropriate treatment of MALT
(mucosa associated lymphoproliferative tissue)
is eradication of H. Pylori
Question 35
• A 63 yo man presents with abdominal pain in
the RUQ and nausea. The patient is 5 days
post-op from laparoscopic cholecystectomy.
The test you should first perform should be:
– A. Abdominal CT
– B. ERCP
– C. Abdominal MRI
– D. RUQ ultrasound
Answer 35
• D. RUQ ultrasound should be done to evaluate
pain s/p lap chole
Question 36
• A 50 year old male undergoes a left
pneumonectomy for a malignant mass. On post-op
day #5, he develops fever and serosanguinous
sputum. A chest xray shows air-fluid levels on the
left. The most appropriate step is:
– A. Chest CT
– B. Repeat the chest xray
– C. ECHO
– D. Perform bronchoscopy
Answer 36
• D. This patient most likely has a
bronchopleural fistula.
• Bronchoscopy is the most important diagnostic
test
Question 37
• The MELD score for ESLD does NOT include:
– A. INR
– B. Serum creatnine
– C. Total bilirubin
– D. Albumin
Answer 37
• D. The MELD score consists of:
– Bilirubin
– INR
– Creatnine
• The maximum score is 40
Question 38
• Catecholamines are synthesized from which of
the following substances?
– A. Arginine
– B. Glutamine
– C. Tyrosine
– D. Tryptophan
Answer 38
• C. Tyrosine is the substrate for catecholamines
Question 39
• Hesselbach’s triangle involves all of the
following structures except:
– A. The inguinal ligament
– B. The lacunar ligament
– C. Rectus muscle
– D. Inferior epigastric artery
Answer 39
• B. Hesselbach’s triangle involves the rectus
muscle, inferior epigastric artery, and the
inguinal ligament
Question 40
• A 55 yo female received 7 days of ampicillin for a
UTI. Six days after she started taking the
medication she presents with diarrhea which tests
positive for C. dif. Which of the following should
you do next?
– A. Observe the patient
– B. Give 7 days of PO clindamycin
– C. Give PO flagyl for 10-14 days
– D. Admit the patient for hydration and IV gentamycin
Answer 40
• C. Treatment of C. dif is flagyl 250-500 mg
orally 4 times a day for 10 days.
• Vancomycin is also effective in treating C. dif.
Question 41
• A 30 yo male presents with RLQ pain suspicious for
appendicitis. You perform laparoscopy and note
the he has terminal ileitis but the cecum and
appendix appear normal. Which of the following
should you do next?
– A. Close the patient
– B. Perform a right hemicolectomy
– C. Perform a segmental ileal resection
– D. Perform an appendectomy
Answer 41
• D. This patient with terminal ileitis that does
not involve the cecum should still undergo
appendectomy.
• Only leave the appendix when the cecum is
involved
Question 42
• A 24 yo man feels a sudden and sharp pain in his
lower calf after playing basketball. Examination
reveals diffuse ecchymosis over his calf and weak
plantar and dorsiflexion of the ankle. Squeezing
the calf does not produce ankle plantar flexion.
The patient most likely injured the:
– A. Posterior tibial tendon
– B. Achilles tendon
– C. Gastrocnemius
– D. Plantaris tendon
Answer 42
• Achilles tendon
• The patient most likely has an achilles tendon
injury.
• Squeezing the calf muscles and producing no
ankle flexion is characteristic for Achilles
tendon injury- this is called the Thompson test.
Question 43
• A 32 yo female presents with an abdominal
bulge slightly to the right of the midline. You
obtain a CT scan which reveals a Spigelian
hernia. Which of the following is true about
this condition?
– A. It very rarely incarcerates
– B. It is through the linea semilunaris
– C. Operative repair is not indicated
– D. It is located inferior to the inguinal ligament
Answer 43
• B. It is through the linea semilunaris
• Spigelian hernias have a high rate of incarceration
• When diagnosed, repair is indicated
Question 44
• A preciously healthy 12 yo boy is brought to the ED
for severe testicular pain that started abruptly
about 5 hours ago. He is afebrile, stable, and his UA
is negative. His left testicle is swollen and TTP with
no cremasteric reflex noted. The next step should
be:
– A. Take to the OR for exploration
– B. Admit for pain control and observation
– B. Send him home with anti-inflammatories and ice pack
– D. Perform an ultrasound
Answer 44
• A. The findings listed are consistent with
testicular torsion
• He should immediately undergo exploration- 6
hours is the window after which the risk of
irreversible damage becomes high
• An ultrasound can delay the procedure
Question 45
• Indications for hiatal hernia repair include:
– A. Regurgitation and aspiration in a patient with a
UGI that demonstrates reflux that is well-controlled
with medical therapy
– B. A patient who has required medical therapy for
more than 5 years
– C. A patient with low grade Barrett’s esophagus
– D. A patient with high grade Barrett’s esophagus
Answer 45
• Indications for hiatal hernia repair include high
grade Barrett’s, regurgitation and aspiration
not controlled with medical therapy, and a
patient that does not want to be treated
medically
Question 46
• A 66 yo man presents with a 9 month history
of dysphagia to solids and food regurgitation.
He denies any abdominal pain or weight loss.
The best test to start his workup should be:
– A. Upper endoscopy
– B. Barium esophagogram
– C. CT of the chest
– D. Esophageal manometry
Answer 46
• B. Barium esophaogram could delineate the
presence of the mass as well as a Zenker’s
diverticulum
Question 47
• A 65 yo man undergoes a low anterior resection for
rectal cancer. 7 hours post-op he develops a fever
of 104 and increased drainage is noted from the
inferior aspect of his incision. The most
appropriate step in management is:
– A. Change the dressing every 4 hours
– B. Change the dressing and start broad spectrum
antibiotics
– B. Take to the OR for debridement
– D. Transfer to the ICU for fluids, IV abx, and monitoring
Answer 47
• C. This patient has an early invasive wound
infection and needs to be taken back for
emergent debridement
• Clostridium perfringes or beta-hemolytic strep
group A are the usual pathogens
• Penicillin is the treatment of choice for both
pathogens
Question 48
• A 70 yo female with diabetes and cor pulmonale is
POD # 14 from a laparoscopic colon resection. She
does not have any complaints though you notice
she now has some blisters over her lower
extremities and 3+ pitting edema of both legs. Her
condition is most likely due to:
– A. Beta hemolytic strep infection
– B. Lymphedema
– C. Superficial venous thrombosis
– D. Peripheral vascular disease
Answer 48
• B. Lymphedema
• Her condition is most likely due to lymphedema from
recent immobilization
• Beta hemolytic strep infection is usually unilateral
• Superficial vein thrombosis usually develops in
varicose veins and is symptomatic (tender, painful,
erythematous)
• Treatment of lymphedema involves diuresis and
compression of lower extremities with ace wraps
Question 49
• An 80 yo woman develops dark and purple
lesions on her right arm. 10 years ago she
underwent a mastectomy with ALND. Her
condition is most likely due to :
– A. Melanoma
– B. Sarcoma
– C. Sarcoidosis
– D. Fibromatosis
Answer 49
• B. Sarcoma.
• This patient most likely has
lymphangiosarcoma from chronic lymphedema
after axillary dissection
• Stewart-Treves syndrome
Question 50
• Thromboxane is involved in:
– A. Increased platelet aggregation and vasodilatation
– B. Increased platelet aggregation and
vasoconstriction
– C. Decreased platelet aggregation and vasodilatation
– D. Decreased platelet aggregation and
vasoconstriction
Answer 50
• B. Thromboxane causes platelet aggregation
and vasoconstriction by increasing calcium
within the platelet
Question 51
• The first branch of the internal carotid artery
is:
– A. The superior thyroid
– B. Inferior thyroid
– B. The ophthalamic
– D. Vertebral artery
Answer 51
• C. The first branch of the INTERNAL carotid is
the ophthalamic artery
Question 52
• Hereditary spherocytosis involves defects in:
– A. Fibrin
– B. Spectrin
– C. Collagen
– D. Plasmin
Answer 52
• B. Hereditary spherocytosis involves defect in
a spectrin protein
Question 53
• A recurrent papillary or follicular thyroid
cancer can be detected with:
– A. Rise in TSH level
– B. Rise in TRH level
– C. Transferrin
– D. Rise in thyroglobulin
Answer 53
• D. Serum thyroglobulin is the best test to
detect recurrent papillary or follicular thyroid
cancer.
Question 54
• Which of the following is true about lobular
breast cancers?
– A. They are often multicentric
– B. They have calcifications on MMG
– C. They make up about 30% of all breast cancers
– D. There is no increased risk for bilateral
involvement
Answer 54
• A. Lobular breast cancers are multicentric,
bilateral, and do not have calcifications.
• They consist of about 10% of all breast cancers
Question 55
• Somatostatinoma can present with all of the
following except:
– A. Diabetes
– B. Necrolytic migratory erythema
– C. Gallstones
– D. Steatorrhea
Answer 55
• B. Somatostatinoma can present with
diabetes, gallstones, and steatorrhea.
• Glucagonoma can present with necrolytic
migratory erythema
Question 56
• Hereditary non-polyposis colon cancer
syndrome is associated with:
– A. Ovarian and breast cancer
– B. Thousands of polyps
– C. Loss of p53 gene
– D. Lung cancer
Answer 56
• A. Pts with HNPCC have predisposition to
ovarian, breast, and colon cancer.
• HNPCC does not present with thousands of
polyps
• It is present:
– In 3 primary relatives over 2 generations
– In at least one person who was less than 50 yrs old
at the time of diagnosis
Question 57
• A 69 yo patient with a history of pancreatitis
presents with severe upper GI bleeding from
gastric varices. On an EGD, you do not see any
esophageal varices. The best treatment would
be:
– A. TIPS
– B. Splenectomy
– C. Splenorenal shunt
– D. Portocaval shunt
Answer 57
• B. Bleeding from gastric varices without
esophageal varices is most likely a/w a
thrombosed splenic vein.
• Treatment is splenectomy
Question 58
• Where would be the most common place to
find an accessory spleen?
– A. The splenic hilum
– B. Aortic bifurcation
– C. Gallbladder fossa
– D. GE junction
Answer 58
• A. The splenic hilum would be the most likely
place to find an accessory spleen
Question 59
• A 66 yo patient with chronic abdominal pain, a
history of chronic pancreatitis, and a dilated
pancreatic duct may benefit from:
– A. A Whipple procedure
– B. A lateral pancreaticojejunosotmy
– C. Billroth 2 procedure
– D. Percutaneous drainage
Answer 59
• B. Patients with chronic pancreatitis and
chronic abdominal pain may benefit from the
Peustow procedure
Question 60
• A 68 yo male is 5 days s/p LAR when he develops
fever, chills, and a WBC of 19,000 with diffuse
abdominal tenderness and feculent brown drainage
of ~ 1200 cc/day from his wound. Which of the
following is the next best step?
– A. OR for takedown of the anastomosis
– B. Percutaneous drainage
– C. Re-exploration and repair of the anastomosis
– D. APR
Answer 60
• A. The next best step would be to re-explore,
being it is only POD #5. The best bet is to
perform a Hartmann’s and colostomy
Question 61
• A 77yo nursing home pt presents with
abdominal pain and films reveal a sigmoid
volvulus. Her vitals are stable and she does not
have peritonitis. The next best step is:
– A. Perform serial exams
– B. Perform colonoscopy
– C. LAR
– D. APR
Answer 61
• B. Colonoscopy can usually decompress
patients with sigmoid volvulus. This is an
appropriate initial step in a stable patient; a
rectal tube should be left in place.
Question 62
• A 33 yo man with a supracondylar humeral
fracture has his arm reduced and a sling is
placed. 2 hours later he develops severe
forearm pain and swelling. You cannot detect
distal pulses. The most appropriate step is to:
– A. Open reduction and internal fixation
– B. Fasciotomy
– C. Angiogram with stent placement
– D. Observation and ice packs
Answer 62
• B. This patient has a compartment syndrome
so fasciotomies are most appropriate
Question 63
• A 55 yo alcoholic presents to the ER after a large
episode of emesis and now has severe substernal
pain. The pain started about 8 hours ago.
Gastrograffin study shows perforation of the distal
esophagus with drainage into the mediastinum.
Which of the following is the most appropriate
treatment?
– A. Esophagectomy
– B. Emergent primary repair
– C. Esophagectomy and placement of a FCJ
– D. Nonoperative mgmt with NNGT and TPN
Answer 63
• B. Primary repair of esophageal rupture is
acceptable within the first 24 hours- the
survival rate is around 90%
• A left thoracotomy is performed for ruptures of
the distal esophagus
• In highly unstable patients, diversion with an
esophagostomy may be safest.
Question 64
• Charcot’s Triad includes all of the following
except:
– A. RUQ pain
– B. Tremor
– C. Fever
– D. Jaundice
Answer 64
• B. Tremor is not a part of Charcot’s triad
Question 65
• A 78 yo patient following a right
hemicolectomy for stage III adenocarcinoma
should have the following?
– A. Chemotherapy and radiation
– B. Radiation only
– C. Q 3 labs, CEA, and imaging
– D. Chemotherapy and follow up
Answer 65
• D. Stage 3 colon cancer should have
chemotherapy
Question 66
• After esophagectomy, the primary blood
supply to the stomach is:
– A. Gastroduodenal artery
– B. Right gastroepiploic artery
– C. Left gastric artery
– D. Left gastroepiploic artery
Answer 66
• B. The right GE artery is what provides the
gastric conduit
Question 67
• Coarctaction of the aorta is?
– A. Proximal to the left subclavian artery
– B. Distal to the left subclavian artery
– C. Is a/w Down’s syndrome
– D. Is located in ascending aorta
Answer 67
• B. Distal to the left subclavian artery
• Coarctation of the aorta, or aortic coarctation,
is a congenital condition whereby the aorta
narrows in the area where the ducts arteriosus
(ligamentum arteriosum after regression)
inserts
Question 68
• A newborn fails to pass meconium during the
first 24 hours and develops abdominal
distention. An x-ray of the abdomen shows
dilated colon. The most appropriate next step
is:
– A. Perform a BE
– B. Observe and administer stool softeners
– C. Order an upper GI with SBFT
– D. Perform a rectal biopsy
Answer 68
• D. This patient may have Hirschsprung’s,
which is characterized by an absence of
ganglion cells in the myenteric plexus
• The best diagnostic test is a rectal biopsy
Question 69
• The most common blood transfusion reaction
is:
– A. Transfusion related acute lung injury
– B. Febrile nonhemolytic transfusion reaction
– C. Allergic reaction
– D. Acute hemolytic reaction
Answer 69
• B. Febrile nonhemolytic transfusion reaction
is the most common
Question 70
• A 45 o develops dizziness and decreased
respiratory rate after he is given IV morphine.
Which drug is most effective in treating this
condition?
– A. Edrophonium
– B. Flumazenil
– D. Neostigmine
– D. Narcan
Answer 70
• D. Overdose of narcan can present with slow
breathing, dizziness, and pinpoint pupils.
• Narcan is the best drug to treat narcotic
overdose
Question 71
• A 6 yo boy presents with a midline neck mass
that is moving when he swallows. You should
recommend:
– A. Resection of the thyroglossal duct cyst
– B. Resection on the cyst and LND
– C. Removal of the mass and resection of the hyoid
bone
– D. Removal of the mass and total thyroidectomy
Answer 71
• C. Removal of the mass and resection of the
hyoid bone is the appropriate treatment for
thyroglossal duct cysts
• This is called the Sistrunk procedure
Question 72
• Gain of function of the following gene is
implicated in the development of colon cancer:
– A. P53
– B. K-ras
– C. DCC
– D. APC
Answer 72
• B. K-ras
• Loss of function of p53, DCC, and APC is
implicated in colorectal cancer development
• Gain of function of pro-oncogene k-ras is
involved in the development of colon cancer
Question 73
• A 25 yo male is s/w GSW to the chest and
abdomen. He is in the intensive care unit and
has required multiple units of blood products.
You would expect increased levels of all of the
following hormones EXCEPT in this patient?
– A. ADH
– B. TSH
– C. Growth hormone
– D. ACTH
Answer 73
• B. TSH. Tissue injury is associated with
stimulation of ADH, growth hormone, and
ACTH production
Question 74
• The most common organism associated with
ventilator associated pneumonia is:
– A. Pseudomonas
– B. S. aureus
– C. E. coli
– D. Strep pneumonia
Answer 74
• B. Staph aureus is the most common organism
a/w VAP
Question 75
• A 67 yo patient with hyperkalemia and new
onset of arrhythmia should first receive:
– A. Kaexylate
– B. Calcium gluconate
– C. Insulin
– D. Insulin and Glucose
Answer 75
• B. Calcium gluconate is the first choice in a
patient with hyperkalemia and arrhytmias to
stabilize cell membranes
Question 76
• The cricothyroid muscle is innervated by:
– A. Superior laryngeal nerve
– B. Recurrent laryngeal nerve
– C. Glossopharyngeal nerve
– D. Vagus nerve
Answer 76
• A. The cricothyroid is innervated by the SLN
Question 77
• Contraindications to lumpectomy and radiation
are all of the following EXCEPT:
– A. Previous radiotherapy
– B. A large tumor
– C. Multiple lesions
– D. Pregnancy
Answer 77
• B. A large tumor.
• Contraindications to lumpectomy and XRT-
pregnancy, multiple lesions, and previous
radiation
Question 78
• A 66 yo male with lung cancer presents with
swollen face and arms. These symptoms are
most likely due to:
– A. SVC invasion
– B. Invasion of the brachial plexus
– C. Invasion of the sympathetic nerves
– D. Paraneoplastic syndrome
Answer 78
• A. This is most likely due to tumor invasion
into the SVC
• Treatment of SVC invasion due to cancer is
radiation
Question 1
• Which of the following statements about
primary sclerosis cholangitis is correct?
– A. Is frequently associated with high fever and chills
– B. Course is usually rapidly progressive to liver
failure
– C. Can usually be managed successfully by anti-
inflammatories and corticosteroids
– D. Is commonly associated with inflammatory bowel
disease
Answer 1
• D. PSC is associated w/ IBD.
• Review of PSC:
– Most commonly affects men in 4-5th decade
– A/w retroperitoneal fibrosis, Riedel’s thyroiditis,
pancreatitis, inflammatory bowel (more so in UC),
and DM
– Sx- fatigue, fluctuating jaundice, pruritis, weight
loss, RUQ pain
– ERCP shows multiple dilations and strictures
(beaded appearance)
Answer 1
• Pts have antimitochondrial antibodies
• Bacterial cholangitis unusual unless biliary tract has
been manipulated
• Does not get better after colon resection for UC
• Leads to portal hypertension and hepatic failure
(progressive fibrosis of intra and extrahepatic ducts)
• Can have isolated intra or hepatic duct fibrosis
• Complications- cirrhosis, cholangiocarcinoma
Answer 1
• Tx- transplant needed long-term for most people
– PTC drainage may be temporizing
– Choledochojejunostomy may be effective in some
– Balloon dilatation of strictures can provide symptomatic
relief
– Cholestyramine can decrease pruritis by decreasing bile
acids
– Irodeoxycholic acid can decrease symptoms by decreasing
bile acids as well as improve liver enzymes
Question 2
• Which of the following regarding splenic
trauma is true?
– A. Most children with blunt splenic trauma require
splenectomy
– B. Patients may return to full-contact activity in 3
weeks
– C. Active blushes and pseudoaneuryms are usually
managed non-surgically
– D. Splenic salvage is associated with increased
transfusion.
Answer 2
• D is correct. Review of splenic trauma->
• Pts are considered fully healed in 6 weeks
• Postsplenectomy sepsis is most common in pts 1-5 yrs and is
more likely within the first 2 years after splenectomy
• Immunizations are required after trauma splenectomy
• Threshold for splenectomy in children is much higher; they
rarely undergo trauma splenectomy
• With conservative mgmt pt should be on bed rest for 5 days
Answer 2
• Indications for operative mgmt for blunt spleen
injuries:
– Pt becomes unstable despite aggressive
resuscitation, including at least 2 units of blood
– Should be highly considered if the pt requires more
than 2 units of RBC to keep HCT at 25
– Active blush on CT or pseudoaneurysm
Question 3
• Which of the following proto-oncogenes is
associated with a G protein defect?
– A. Myc
– B. Ras
– C. Src
– D. Sis
– E. Erb B
Answer 3
B. Ras is associated with a G-protein defect

Ras G-protien defect

Src Tyrosine kinase defect


Sis Platelet-derived growth factor
receptor defect
Erb B Epidermal growth factor
receptor defect
myc Transcription factors
Question 4
• Which of the following antibodies is the most
abundant in the body?
– A. IgM
– B. IgG
– C. IgA
– D. IgD
– E. IgE
Answer 4
• B. IgG is the most abundant
• IgM- initial antibody made after antigen
exposure. Is the largest antibody with 5
domains and 10 binding sites
• IgG- most abundant antibody. Responsible for
secondary immune response. Can cross the
placenta and provides protection in the
newborn period
Answer 4
• IgA- found in secretions, in Peyer’s patches of
the bowel, and in breast milk. Helps prevent
microbial adherence and invasion in the gut
• IgD- membrane-bound receptor on B cells-
serves as an antigen receptor
• IgE- Allergic reactions, parasite infections
Question 5
• Regarding microsomal drug metabolism, which
of the following is true?
– A. Phase II reactions describe demethylation,
oxidation, reduction, and hyrolysis reactions
– B. Cimetidine is a P-450 inducer
– C. Warfarin is a P-450 inhibitor
– D. The P-450 system transforms aromatic
hydrocarbons into carcinogens
Answer 5
• D. The P-450 system does transform aromatic
hydrocarbons into carcinogens.
• To review;
– Microsomal drug reaction takes place in the hepatic cell
endoplasmic reticulum via the P-450 system
– Phase I reactions describe demethylation, oxidation,
reduction, and hyrolysis reactions
– Phase II reactions describe those where glucoronic acid
(#1) and sulfides become attached to form a water-soluble
metabolite that is often inactive & ready for excretion.
Answer 5
• Inhibitors of P-450
– Allopurinol, Amiodarone
– Cimetidine, Cipro
– Disulfram, Erythromycin, Flagyl
– Isoniazid, ketaconzaole, MAOI’s, verapamil
• Inducers of P-450
– Cruciform vegetables, cigarette smoke, insecticides, ETOH
– Dilantin, phenobarbial
– Theophylline, warfarin
Question 6
• A schistosomiasis abscess is best treated by:
– A. Percutaneous drainage and antibiotics
– B. Flagyl
– C. Levofloxacin
– D. Praziquantel
Answer 6
• D. The treatment of a schistosomiasis liver
abscess is praziquantel. These cysts only need
percutaneous drainage for superinfection.
Surgery is indicated only for complications such
as bleeding.
• These patients may also get esophageal varices
Question 7
• An amoebic abscess is best treated by:
– A. Percutaneous drainage and antibiotics
– B. Flagyl
– C. Levofloxacin
– D. Praziquantel
Answer 7
• B. An amoebic abscess is best treated with
flagyl. Percutaneous drainage is needed only
for super-infection. Surgery is only for
complications such as bleeding.
Question 8
• A pyogenic abscess is best treated by:
– A. Percutaneous drainage and antibiotics
– B. Flagyl
– C. Levofloxacin
– D. Praziquantel
Answer 8
• A. A pyogenic abscess is best treated by broad
spectrum antibiotics and percutaneous
drainage. GNR’s are the most common
organisms in these cysts. You should also cover
for anaerobes
• Liver abscesses can arise weeks after an
episode of diverticulitis, appendicitis, or other
abdominal infectious processes.
Question 9
• All of the following are true about bladder cancer
except;
– A. The most common type is transitional cell carcinoma
– B. Schistosomiasis infection is a/w squamous cell cancer
of the bladder
– C. Transitional cell cancer limited to the submucosa can
be treated with intra-vesicle BCG or trans-urethral
resection
– D. Transitional cell cancer with muscle invasion (T2) can
be treated with wedge resection
Answer 9
• D. Transitional cell cancer with muscle invasion
requires total cystectomy and ileal conduit
formation
Question 10
• The most common type of ovarian tumor is:
– A. Epithelial
– B. Germ cell
– C. Sex-cord
– D. Lymphoma
Answer 10
• A. The most common type of ovarian cancer is
epithelial
Question 11
• The following structures are found in the
anterior compartment of the leg:
– A. Posterior tibial artery, peroneal artery, tibial
nerve
– B. Anterior tibial artery and superficial peroneal
nerve
– C. Sural nerve, gastrocnemius, soleus, and plantaris
muscles
– D. Anterior tibial artery, deep peroneal nerve
Answer 11
• D. The anterior compartment of the leg
contains:
– Anterior tibial artery and deep peroneal nerve
– Muscles- anterior tibialis, extensor hallicus longus,
extensor digitorum longus, and communis.
• The superficial posterior compartment
includes:
– Sural nerve
– Muscles- gastrocnemius, soleus, plantaris
Answer 11
• The lateral compartment includes:
– Superficial peroneal nerve
– Muscles- peroneal muscles
• The deep posterior compartment includes:
– Posterior tibial artery, peroneal artery, and tibial
nerve
– Muscles- flexor hallicus longus, flexor digitorum
longus, posterior tibialis
Question 12
• You are asked to see a 1-day old infant with Down’s
syndrome due to feeding intolerance. In addition to the
diagnosis of Down’s the records indicate that
polyhydramnois was noted during the pregnancy. The
mother describes the baby’s emesis as green. Which of
the following condition does this child most likely have?
– A. Tracheoesophageal fistula
– B. Intussesception
– C. Duodenal atresia
– D. Pyloric stenosis

Answer 12
• C. Duodenal atresia.
– #1 cause of duodenal obstruction in newborns
– Usually presents within 1st week
– Usually distal to ampulla of Vater
– Bilious emesis, feeding intolerance
– Associated with cardiac, renal, GI anomalies as well as
polyhydramnois in the mother
– 20% have Down’s syndrome
– “Double-bubble” sign on plain film
– Treatment- resuscitation, duodenoduodenostomy or
duodenojejunostomy
Question 13
• A mother brings her 1-month old to you because of a
sudden onset of bilious emesis. You examine the
patient then order an upper GI which reveals that the
duodenum does not cross the midline. Given the
most likely diagnosis, what will ultimately be required
to correct this problem?
– A. Pyloromyotomy
– B. Air-contrast enema
– C. Duodenojejunostomy
– D. Resect Ladd’s band, counterclockwise rotation of bowel,
place cecum in LLQ (cecopexy), and perform an
appendectomy
Answer 13
• D. This patient most likely has malrotation so the
appropriate procedure is resection of Ladd’s bands,
cecopexy, appendectomy, and counterclockwise
rotation of bowel.
• Characteristics of malrotation
– Sudden onset of bilious vomiting
– Ladd’s bands cause duodenal obstruction, coming out of
the retroperitoneum near the Ligament of Treitz
– Failure of normal counterclockwise 270 degree rotation
Answer 13
• Volvulus is associated with compromise of the
SMA which can lead to intestinal infarction
• 90% present by 1 yr, 75% present within the 1st
month of age
• Any child with bilious vomiting needs an upper GI
to rule out malrotation
• On upper GI the duodenum does not cross the
midline
Question 14
• A 48 yo male that drinks a case of beer daily had nausea
and vomiting at home about 5 hours before he presents
to the ER due to chest pain. Vitals- BP 100/50, HR 110;
you subsequently bolus crystalloid. A chest xray shows
a left effusion and pneumothorax. What is the most
sensitive test to diagnose this pt’s condition?
– A. Abdominal CT
– B. Chest CT
– C. Angiogram
– D. Gastrograffin swallow followed by thin barium swallow
Answer 14
• D. Although a CT of the chest & abdomen with
oral and IC contrast is reasonable, a swallow is
the most sensitive test to diagnose
Boerhaave’s syndrome
Question 15
• The most common location for Boerhaave’s to
occur is:
– A. The left posterior lower esophagus
– B. The left posterior middle esophagus
– C. The upper stomach
– D. The left posterior upper esophagus
Answer 15
• A. The most common location for such a
rupture is the left posterior lower esophagus.
They usually will perforate into the left chest.
Question 16
• You decide to explore this pt. What is the best
operative approach?
– A. Right thoracotomy
– B. Left thoracotomy
– C. Thoracoabdominal incision
– D. Laparotomy
Answer 16
• B. The best approach to reach a perforation in
the left lower esophagus is a left thoracotomy.

• If the lesion were in the middle or upper 1/3


you would go through the right chest
Question 17
• You open the pt and find gastric contents
coming out through a hole in his esophagus.
You wash it out and then FIRST:
– A. Place a chest tube and close
– B. Perform a myotomy
– C. Full thickness bites where you see fluid coming
out
– D. Esophagectomy
Answer 17
• B. Perform a myotomy. You want to see the extent
of the injured mucosa which often extends farther
than the muscle injury.
• Esophagectomy isn’t yet indicated in this pt
because it occurred 5 hours ago and gross
mediastinitis is unlikely
• Esophagectomy is indicated:
– When the esophagus is too badly damaged
– When there is severe mediastinitis and the esophagus
can’t be salvaged
– More likely to be required in delayed presentations
Question 18
• All of the following are true about nitrous
oxide (NO) except:
– A. Increases cAMP
– B. Precursor is arginine
– C. Released primarily from vascular endothelium
– D. Receptor is guanyl cyclase
Answer 18
• A. Arginine is the precursor to nitrous oxide.
NO acts on guanyl cyclase to increase cGMP
and cause vasodilation
Question 19
• All of the following are true of the hepatic
acute phase response except:
– A. IL-6 is the most potent stimulus
– B. Albumin is decreased
– C. C-reactive protein is increased
– D. Transferrin in increased
Answer 19
• D. is false; transferrin and albumin are
decreased in the acute phase response.
• Proteins that are increased include C-reactive
protein, amyloid A & P, fibrinogen,
haptoglobin, ceruloplasmin, alpha-1
antitrypsin, C3 (complement), and alpha-1
antichymotrypsin
Question 20
• Which correctly describes the zones of the
retroperitoneum?
– A. Zone 1 includes the pelvis and is associated with pelvic
fractures
– B. Zone 2 includes the central retroperitoneum
– C. If you find a hematoma in zone 2 when exploring the
patient for blunt trauma, you do not have to explore it if it
remains stable throughout the operation
– D. Zone 3 injuries, whether they are blunt or penetrating,
always need to be explored
Answer 20
•C. If exploring a patient for blunt injury and you
encounter a stable hematoma in zone 2, you do
not have to explore it.

Zone Location Associated injuries

1 Central Pancreaticoduoden
retroperitoneum al injuries, injuries
to major abdominal
vasculature
2 Flank, perinephric Injuries to GU tract
area or colon
3 Pelvis Pelvic fractures
Answer 20
• In general, penetrating injuries to any of the 3
zones need to be explored
• Blunt zone 2 injuries that are stable can be
worked up radiographically to determine the
location of the injury. If they are found
intraoperatively and noted to be expanding,
they need to be explored.
Answer 20
• Blunt zone 3 usually can be managed by pelvic
fixation.
– Seventy percent of pelvic bleeding is from pelvic veins
– Restoring the pelvis to its normal configuration cause the
hematoma to compress the veins and stop venous
bleeding
– If the patient continues to be hemodynamically unstable,
an arteriogram with selective embolization of the bleeding
vessel coming from the internal iliac artery should be
performed
Question 1
• Which of the following is true regarding the efficacy of
prophylactic antibiotics following a GSW through the colon?
– A. Giving a single dose of abx 2 hrs preoperatively increases SSI in
these patients
– B. Infection rates are significantly lower when antibiotics are given for
a complete 5 day course
– C. There is no statistical difference when antibiotics are given within 2
hours of initial skin incision and when given within the first 3 hours
when the skin incision is made
– D. There is no evidence supporting continuing abx beyond 24 hours
even when the bowel is perforated
– E. The bullet is sterilized coming out of the handgun and the intra-
abdominal organs themselves are the major source of wound
infections in these patients
Answer 1
• D. The bullet is not sterilized coming out of the barrel
• The clothing draws the skin flora into the wound and is the
major source of wound infection
• There have been studies comparing length of abx therapy
and no proven benefit has been demonstrated by
extending therapy beyond 24 hours if no abscess is present
and the source of bowel contamination is removed
• There is a difference in SSI when an appropriate fose of abx
is given within 2 hours before the skin incision
Question 2
• Which of the following is the most important
prognostic factor in patients with malignant
melanoma?
– A. Surface size
– B. Tumor grade
– C. Depth
– D. SLN biopsy status
– E. Margins of resection
Answer 2
• D. The development of SLN Bx began an era where
micromets could be identified and staged more accurately,
allowing appropriate patients to receive total LN dissection
and treated with adjuvant therapy
• For patients with stage 1 and 2, SNLB status has become
the most important prognostic factor
• There is a survival benefit when patients with nodal
involvement receive high-dose interferon alfa 2-b adjuvant
• Thickness does parallel the percent positivity of SLN (3.6%
for lesions less than 0.75 mm to 44% in tumors greater
than 4 mm)
Question 3
• Which of the following is NOT a component of
informed consent?
– A. Disclosure
– B. Comprehension
– C. Competency
– D. Voluntariness
– E. Description of basic procedural standards of care
Answer 3
• E is not a part of informed consent
• Disclosure by the physician, patient
comprehension, competency, and
voluntariness all are.
Question 4
• Which of the following is true about the
secretory diarrhea seen in gastrinoma
patients?
– A. It partially is due to enterocyte damage from the
excessive acid
– B. It can be improved by NG suctioning
– C. It may be the only complaint in around 20% of
patients presenting with gastrinoma
– D. A and C only
– E. All of the above are true
Answer 4
• E. All of the above are true about gastrinoma
Question 5
• Which of the following forms does fat take to
enter the lymphatic system and eventually
reach the liver?
– A. Free fatty acids
– B. Chylomicrons
– C. Triglycerides
– D. Monoglycerides
Answer 5
• B. Chylomicrons
Etiology of Steatorrhea in Gastrinoma

• Fat digestion begins in the small bowel and begins with


the emulsification of fat by bile and lecithin, which are
partly soluble in water and partly in lipid
• Larger fat globules get broken down into smaller
globules, increasing the total surface area of fat
– This allows better access for pancreatic lipase, which digests TGs into Fatty acids
and 2-MG’s
• Bile salts separate fat lobules until they create micelles
– These carry the FA’s into the intestinal brush border for absorption
Etiology of Steatorrhea in Gastrinoma

• Once inside cell, the FAs and MGs reform TGs


which combine with cholesterol,
phospholipids, and apoprotiens to form
chylomicrons
• Chylomicrons then enter the lymphatic system
and eventually get transferred to the liver
Etiology of Steatorrhea in Gastrinoma
• Gastrinoma is the second most common islet cell
tumor of the pancreas
• Is the most common symptomatic endocrine tumor
of pancreas
• Hallmark of Zollinger-Ellison Syndrome
• 25% of gastrinoma patients have MEN-1 syndrome
• Main sx caused by increased gastric acid secretion-
abdominal pain most common (75%)
• Diarrhea in 2/3 and may be only sx in 20%
Etiology of Steatorrhea in Gastrinoma
• Increasing amount of gastric acid secretion
decreased the pH in duodenum after stomach
emptying
• Decreased duodenal pH leads to breakdown of
pancreatic lipases- this leads to malabsorption and
steatorrhea
• This also damages enterocytes and insolubilizes bile
salts
• Unlike other forms of secretory diarrhea, secretory
diarrhea from gastrinoma can be treated by NG
suctioning
Question 6
• A 63 yo female with a longstanding history of Type 2
diabetes and noncompliance with medication presents
with some bony deformity of her right foot and a clean
~4 cm plantar ulcer. The pulses are good and there is
no evidence of cellulitis. Which of the following is the
most appropriate initial treatment for this patient?
– A. Ulcer debridement and skin grafting
– B. Bony reconstruction and ulcer debridement
– C. Modified fitted boots
– D. Below knee amputation with early prosthetic fitting
Answer 6
• C. A removable boot or appropriate shoe with
monitoring of the wound is most appropriate
Initial Treatment of Plantar Foot Ulcer in
Diabetic
• Diabetes is the most common cause of neuropathy
leading to loss of protective sensation on the plantar
aspect of the foot
• Bony deformity can also occur as a result of neuropathy
• Combination of lack of protective sensation and
deformity leads to pressure ulceration and infection
• Osteomyelitis can develop requiring bone resection or
amputation
Initial Treatment of Plantar Foot Ulcer in
Diabetic
• Gold standard to treat diabetic foot ulcers- provide
appropriate pressure relief
• Can be via a total contact cast or remoavable cast
boot
• Further treatment o more advanced wounds
includes debridement with removal of all necrotic
tissue and to create a plantar grade foot
• Surgery may be necessary in those whose deformity
doesn’t respond to shoe modification
• Surgical goals- resect bony prominences and infected
bone, restore bone architecture
Question 7
• A 38 yo female underwent a laparoscopic Nissen
fundoplication and hiatal hernia repair 2 days ago that
was technically difficult. She now is mildly tachycardic
and complains of shortness of breath. Chest x-ray
shows an effusion on the left and a small amount of
free air under the diaphragm. Her SBP is 140/90 mmHg
and UOP is good. Which study is best to determine the
site of leak?
– A. MRI of chest and abdomen
– B. Water-soluble contrast study
– C. EGD
– D. EUS
Answer 7
• B. Imaging with a water soluble contrast agent
is most appropriate
Dx Test for Pleural Effusion after Nissen
• Feared complication after laparoscopic Nissen- esophageal or
gastric perforation
• Causes:
– Traction applied to esophagus or stomach
– Inadvertent cautery burns during dissection
• Leak will manifest during the first 48 hours
• Peritoneal signs will be noted if spillage limited to abdomen
• SOB and pleural effusion if spillage extends to chest
• Site of leak should be confirmed by a water-soluble contrast
study
• Perforation best handled by laparotomy though may be fixed
laparoscopically if discovered during initial operation
Question 8
• Which of the following patients with an AAA
should be offered elective repair?
– A. A 45 yo male with a 4 cm aneurysm that was 3.8
cm 6 months ago
– B. A 59 yo female with a 5.1 cm asymptomatic
aneurysm
– C. A 61 yo male with a 4.8 cm aneurysm that was
4.0 cm 6 months ago
– D. All of the above should be offered elective repair
Answer 8
• C is most appropriate
Indication for Surgical Treatment of
Asymptomatic Aneurysm
• Ruptured AAA is the 10th leading cause of death in men in the USA
• Risk of rupture increases with increasing diameter
• Annual risk of rupture of 4-5 cm lesion is 1%
• 5-6 cm- 2-5%
• 3-10 cm- 3-10%; 5 year rupture is nearly 50%
• Most are asymptomatic and are found incidentally
• Elective repair should be offered when risk of mortality is less than
5%
• Size less than 5.5cm- follow with biannual CT or U/S
• Pts with aneurysms larger than 5.5 cm or which have growth over
0.5 cm in 6 months should be offered elective repair
Question 1
• Of the following, which is the most critical
component of neovascularization in tumor
metastases?
– A. HER receptor
– B. VEGF receptor
– C. Neu receptor
– D. FGF receptor
Answer 1
• B. One of the most critical elements in the
neovascularization of metastases is the VEGF
(vascular endothelial growth factor) receptor
• Many new chemo regimens are targeting this
receptor
Question 2
• Which of the following lab values is most
consisted with primary hyperparathyroidism?
– A. Cl 105, phos, 3, elevated renal cAMP
– B. Cl 105, phos 4, elevated renal cAMP
– C. Cl 107, phos 3, and decreased renal cAMP
– D. Cl 106, phos 4, decreased renal cAMP
Answer 2
• A. Primary hyperparathyroidism is associated
with: an
– Elevated PTH
– Elevated Ca
– Elevated renal cAMP (effect of PTH on the PTH
receptor in the kidney)
– A Chloride / Phos ratio greater than 33
• The half-life of PTH is 18 minutes
Question 3
• Massive bleeding 7 days after a tracheostomy
is most likely due to:
– A. Tracheo-carotid fistula
– B. Tracheo-inominate fistula
– C. Tracheo-aortic fistula
– D. Tracheo-jugular fistula
Answer 3
• B. The most common cause of massive
bleeding following a tracheostomy is a tracheo-
innominate fistula
• Place finger through the tracheostomy site to
try to press the innominate artery against the
sternum
• If diagnosed, go to OR for median sternotomy
Answer 3
• Ligate and divide the innominate artery (some
say place a graft but this is a high infection risk
for this complication)
• Ligation of the innominate artery proximal to
the takeoff of the right subclavian usually
doesn’t result in any neurologic dysfunction
due to collateral flow
Question 4
• You start coumadin on a patient who just got
diagnosed with a PE. Three days later he starts to
slough skin from his arms and legs. All of the
following are true of this pt’s condition except:
– A. This likely could have been prevented if you started
heparin before coumadin
– B. Pts with protein kinase C deficiency are more susceptible
to this problem
– C. The skin sloughing is caused by skin necrosis
– D. This is most likely due to undiagnosed hemophilia A
Answer 4
• D. Warfarin-induced skin necrosis occurs in patients
started on coumadin without first being given heparin. It
results from a relatively hypercoagulable state that occurs
in some people due to the relatively short half-life of
protein c & S.
• Protein C & S decrease after coumadin administration
before the other factors start to decrease, leading to a
relative hypercoagulable state
• Pts with protein C deficiency are at increased risk for
having this problem, which is prevented by starting heparin
before coumadin
Question 5
• The most common benign tumor of the parotid
is:
• The most common tumor to involve bilateral
parotids glands at the time of presentation is:
– A. Mucoepidermoid ca
– B. Adenoid cystic carcinoma
– C. Pleomorphic adenoma
– D. Warthin’s tumor
Answer 5
• C. The most common benign tumor is a
pleomorphic adenoma
• D. The tumor most likely to involve bilateral
parotids at the same time a Warthin’s tumor
Question 6
• All of the following are true about pts that
have undescended testicles except:
– A. You can wait until they are 2 yrs old to treat
– B. There is a higher risk of testicular cancer in these
pts
– C. The cancer risk is lower if both testicles are
brought into the scrotum early
– D. The tumor they are most likely to get is a
seminoma
Answer 6
• C. The risk of developing cancer is the same even if
the testicles are returned to the scrotum early.
However, returning them to the scrotum facilitates
examination.
• The treatment when diagnosed is orchiopexy
through an inguinal incision.
– If you can’t bring the testicles down close and wait 6
months before trying again.
– If you still can’t get them down, divide spermatic vessels
Answer 6
• The other points are true
– You can wait until they are 2 yrs old to treat
– There is a higher risk of testicular cancer in these pts
– The tumor they are most likely to get is a seminoma
• If undescended bilaterally get chromosomal studies
• Prune belly syndrome- hypoplasia of the abdominal
wall, urinary tract abnormalities with a dilated
urinary system, and bilateral cryptorchidism
Question 7
• Antibiotics can be subdivided into
bacteriostatic and bacteriocidal. Each of the
following is bacteriostatic except:
– A. Bactrim
– B. Tetracycline
– C. Erythromycin
– D. Gentamicin
Answer 7
• D. Bacteriostatic agents include
chloramphenicol, tetracycline, clindamycin,
erythromycin, and bactrim
Question 8
• Which of the following is not true about the
treatment of anal margin squamous cell cancers?
– A. The anal margin is defined as those below the dentate
line
– B. They tend to have a better prognosis than anal canal
lesions
– C. Wide local excision is appropriate for lesions less than 3
cm
– D. Metastases first tend to go to para-aortic nodes
Answer 8
• D. Mets tend to go to the inguinal nodes first.
• Squamous cell cancers of the anus are often
ulcerating
• You can perform WLE with a 0.5 cm margin for
lesions less than 3 cm
• The margin describes lesions below the dentate
line and do have a better prognosis than canal
lesions
Answer 8
• An APR may be required for large lesions if the
sphincter is involved.
• If the nodes are clinically positive, an inguinal
dissection should be performed.
• Men have a better prognosis
Question 9
• Which of the following are true about LeFort
fractures?
– A. All 3 types can be managed with intramaxillary
fixation only
– B. Type 3 involves the lateral orbital walls
– C. Type 2 involves the lateral orbital walls
– D. They generally are managed nonoperatively
Answer 9
• B. Type 3 involves the lateral orbital walls.
Please refer to the following chart and
pictures.
Answer 9

Type Description Treatment

1 Maxillary fracture straight Reduction, stabilization,


across intramaxillary fixation, +/-
circumzygomatic and
orbital rim suspension
wires
2 Lateral to nasal bone, Same as type 1
under eyes, down toward
maxilla
3 Lateral orbital walls Suspension wiring to
stable frontal bone, may
need external fixation
LeFort 1
LeFort 2
LeFort 3
Question 10
• Sludging in the gallbladder and cholestatic
jaundice is most characteristic of which
antibiotic?
– A. Quiniolones
– B. Bactrim
– C. Erythromycin
– D. Ceftriaxone
Answer 10
• D. Sludging in the gallbladder and cholestatic
jaundice are complications of ceftriaxone
Question 11
• The most common infection in pts with large
(>35%) severe burns is:
– A. Pneumonia
– B. UTI
– C. Wound sepsis
– D. Liver abscess
Answer 11
• A. Although UTI is the most common infection in
surgery pts, pneumonia is the most common
infection in people with severe burns.
• Inhalation injury, decreased immunity, fluid
resuscitation leading to pulmonary edema, and
requirement for mechanical ventilation all lead to
increased risk for pneumonia
• Up to 60-70% of pts with large burns get
pneumonia
Question 12
• Post-op day 1 after a right upper lobectomy
your pt develops a 102 F fever. The most likely
source of the fever is:
– A. PMN’s
– B. Platelets
– C. Macrophages
– D. Lymphocytes
Answer 12
• C. Alveolar macrophages are activated with
atelectasis and cause fever
Question 13
• The most important prognostic factor in pts
with sarcomas without metastases is:
• The most important prognostic factor in pts
with breast cancer without metastases is:
– A. Node status
– B. Size of tumor
– C. Tumor grade
– D. Gender of pt
Answer 13
• C and A.
• Sarcomas the most important factor is tumor
grade. These tumors rarely go to lymph nodes
• Nodal status is most important in breast cancer
(whether there are + nodes, how many, and
location- axillary, IMA)
Question 14
• Of the anesthetics listed below, which of the
following is most likely to cause an allergic
reaction?
– A. Lidocaine
– B. Bupivicaine
– C. Mepivicaine
– D. Procaine
Answer 14
• D. Amide type anesthetics (all have an I in the
1st part of their name) such as lidocaine,
bupivicaine, and mepicivaine rarely cause
allergic reactions
• Ester type anesthetics such as procaine,
cocaine, and tetracaine are most likely to cause
allergic reactions because of their PABA
analogue
Question 15
• The most common organism involved in
cholangitis is:
– A. S. aureus
– B. Streptococcus
– C. Bacteroides fragilis
– D. E. coli
Answer 15
• D. E. coli is the most common organism
involved in cholangitis
Question 16
• The most common cause of cholangitis is:
– A. Gallstones
– B. Malignancy
– B. Benign biliary strictures
– C. Iatrogenic injury
Answer 16
• A. The most common cause of cholangitis in
gallstones
Question 1
• All of the following are true of atrial natriuretic
peptide EXCEPT:
– A. Released from atria when stretched
– B. Causes natiuresis and diuresis
– C. Is an inherent method of removing excess
volume in patients with CHF
– D. Is a steroid hormone
Answer 1
• D. ANP is released when the atrium are
stretched. The peptide inhibits Na absorption
and water absorption at the collecting ducts.
Question 2
• The ventilation strategy that has been most
effective in pts with ARDS in terms of
morbidity and mortality is:
– A. Volume control
– B. Pressure control
– C. Permissive hypoxia
– D. Strict control of CO2 into normal ranges
Answer 2
• B. Pressure control ventilation to avoid
barotrauma and permissive hypercapnia
(allowing an elevated CO2) while correcting
the pH and HCO3- has been the most effective
mode of ventilating pts with ARDS.
• Other maneuvers, such as prone and supine
positioning, have been used.
Question 3
• A 4 yr old girl is brought to the ER after being bitten
by a classmate. The bite is on her forearm and
there is not sign of devitalized tissue. The would is
irrigated and closed. Cultures of the wound will
most likely grow:
– A. S. aureus
– B. Eikenella corrodens
– C. Fusobacterium species
– D. Prevotella species
– E. Pasteurella multocida
Answer 3
• A. S. aureus. The most common bacteria found in human
bites are S. aureus, S. epidermidis, alpha and beta strep,
Corynebacterium sp, and Eikenella corrodens
• Occlusional human bites- teeth puncture
– Infection risk similar to dog and cat bites
– Hand wounds- higher infx risk
• Clenched fist- Fist hits teeth and causes injury
– More dangerous; can lead to septic arthritis and osteomyelitis
Answer 3
• E. corrodens infects up to 25% of closed fist injuries
and can lead to chronic, indolent infections
• Human bites worse than dog or cat
• Tx human bites- wash w/ iodine, elevate if possible,
give broad spectrum abx if on the hand or those at
higher risk; tetanus
• Tx animal bites- most are dog; tx with washout,
broad spec abx. Primary closure OK if not on hand,
less than 12 hrs old, and not a cat bite
– Most common bacteria- Pasturella multocida
Question 4
• 4 yrs after surgery for a GSW to the right thigh, a 26
yo man is referred to you for a thrill and bruit over
the area discovered on a routine exam. Which of
the following might you also find in this pt?
– A. Cardiac output of 3.2 L/ min
– B. HR of 55 bpm
– C. MVO2 of 55%
– D. BP of 160/90 mmHg
– E. Left ventricular hypertrophy
Answer 4
• E. AV fistulas can occur following penetrating trauma to the
abdomen or the extremities.
• They are usually asymptomatic, especially in young pts, but
can lead to high output cardiac failure due to shunting or
distal ischemia due to steal.
• Left-to-right shunting will caused elevated mixed venous O2
and lead to high output failure (LVH, tachycardia, low BP)
• Branham-Nicoldoni sign- compress the artery proximal to
the AVF- causes reflex bradycardia and increased blood
pressure
• Prompt surgical tx (open or endovascular) is necessary
Question 5
• Which of the following is characteristic of
Merkel Cell cancer?
– A. Slow growing, well-defined cutaneous lesion
– B. Early distant mets
– C. Locally aggressive with low chance of distal
spread
– D. Frequently cured with WLE alone
– E. Similar to squamous cell ca histologically
Answer 5
• B. Early distant mets. Merkel cell tumors are most
similar to basal cell lesions. Both are aggressive
and may have distant mets to the bone, nodes, and
viscera
• Difficult to treat though need to be removed with
2-3 cm margins, elective regional
lymphadenectomy, SNL bx, radiation
• Radiation can be used as adjuvant therapy for both
local and regional control and when surgery is not
an option
• Chemotherapy is under investigation
Question 6
• The mechanism of 5-FU is to:
– A. Inhibit thymidalate synthesis
– B. DNA alkylation
– C. Inhibit microtubule formation
– D. Stabilize microtubules
Answer 6
• A. 5-FU inhibits thymidalate synthesis.
Leucovorin increases the toxicity of 5-FU
Question 7
• Histamine release is characteristic of:
– A. Demerol
– B. Fentanyl
– C. Sufentanil
– D. Morphine
Answer 7
• D. Morphine has a characteristic histamine
release which can cause hypotension
Question 8
• 4 days after an Ivor-Lewis esophagectomy you
start enteral feeds through a j-tube. The long
chain fatty acids contained in tube feeds:
– A. Enter the circulation via the portal vein
– B. Enter the circulation via lymphatics
– C. Are only synthesized in the body
– D. Are not found in chylomicrons
Answer 8
• B. Long chain fatty acids enter the body
through terminal lacteals (absorption through
the lymphatic system) either as free long chain
fatty acids or as chylomicrons
Question 9
• An overdose of fentanyl is treated with:
• An overdose of ativan is treated with:
– A. Flumazenil
– B. Narcan
– C. Neostigmine
– D. Edrophonium
Answer 9
• B and A. All narcotic agents can be treated
with Narcan when overdoses.
• Overdoses of benzodiazapenes can be treated
with flumazenil
Question 10
• During the resection of a pelvic tumor, the left
ureter is inadvertently transected below the
pelvic brim. Which of the following do you
choose for immediate repair of this problem?
– A. Primary repair
– B. Primary repair with stent
– C. Diversion with ureteroenterostomy
– D. Ureterocystostomy
– E. Delayed repair and perc drain the urinoma
Answer 10
• D. Ureterocystostomy
• When the ureter is accidentally ligated and the injury
missed, the pt develops hydronephrosis and urosepsis. This
can lead to pressure buildup and urine leak
• For minor injuries where there is a partial laceration, the
ureter can be repaired over a stent
• For complete transections, primary closure should be
attempted
• Below the pelvic brim, ureteroureterostomy can be difficult
so a ureterocystostomy is the procedure of choice
Question 11
• Which of the following is a characteristic of a
cutaneous lymphatic malformation?
– A. Bluish mass with overlying telangiectasias
– B. Pulsatile ballottable mass
– C. Cystic mass with overlying vesicles
– D. Firm, nodular mass
– E. Irregular mass fixed to the underlying tissues
Answer 11
• C. Cystic mass with overlying vesicles
• Cutaneous lymphatic malformations often
occur as cystic masses with overlying vesicles.
• A lymphatic mass wound not be associated
with telangiectasias.
• They are also not nodular or pulsatile and they
are not be fixed to underlying tissue
Question 12
• A 22 yo man suffers a severe pelvic fracture
and has hematuria. You get a retrograde
cysturethrogram and see an extraperitoneal
bladder rupture. The most appropriate
therapy is:
– A. Foley x 7 days
– B. Exploratory laparotomy
– C. Nothing
– D. Cystectomy
Answer 12
• A. The most appropriate therapy for an
extraperitoneal bladder rupture is urinary
catheter drainage for several days
• The most appropriate therapy for an
intraperitoneal bladder rupture is ex lap and
primary repair.
Question 13
• A 27 you man is in the ICU and is s/p splenectomy
for MVA 6 hrs ago. The patient had a prolonged
transport and received 20 units of blood prior to
arrival. Currently his peak airway pressures are 65,
his abdomen is distended, he is not making urine,
and his bladder pressure is 40. The most
appropriate action is:
– A. Increase PEEP
– B. Volume resuscitation
– C. Decompressive laparotomy
– D. CT scan
Answer 13
• C. This is a classic description of compartment
syndrome.
• A bladder pressure above 25, decreased UOP
(compression of the IVC causes decreased
cardiac output) and elevated ventilation
pressures are part of the syndrome
• Treat with decompressive laparotomy
Question 14
• When performing an abdominal perineal
resection, the vessels located in the lateral
stalks are:
– A. Middle rectal arteries
– B. Superior rectal arteries
– C. Superior sigmoidal arteries
– D. Inferior rectal arteries
Answer 14
• A. The middle rectal arteries are located in the
lateral stalks.
Question 15
• A newborn fails to pass meconium in the 1st 24
hours and develops abdominal distention. Plain
films show a distended colon. The child’s anus is in
the correct location but on rectal exam there is an
explosive release of watery stool. The next most
appropriate step is:
– A. Upper GI
– B. Barium enema
– C. Enteroclysis
– D. Rectal biopsy
Answer 15
• D. The scenario is most consistent with
Hirschsprung’s disease. The diagnosis is made
with rectal biopsy which shows an absence of
ganglion cells in the myenteric plexus
Question 16
• A 65 yo woman has stage 2 squamous cell
cancer of the vulva. What is the most
appropriate therapy?
– A. Radiation
– B. Chemotherapy
– C. Resection of the labia
– D. Bilateral labial resection
Answer 16
• D. Stage 1 vulvar cancer is limited to 1 labia and is
less than 2 cm
• Stage 2 is > 2 cm
• Stage 3 involves nodes or invasive disease beyond
the labia
• Treatment of stage 1- remove labia
• Stage 2- bilateral labial resin
• Stage 3- Wide en bloc resection and nodal
dissection
Question 17
• The most common cause of persistent
hyperparathyroidism after surgery is:
– A. Parathyroid cancer
– B. Parathyroid hyperplasia
– C. Missed adenoma during original operation
– D. New adenoma
Answer 17
• C. The most common cause of persistent
hyperparathyroidism is a missed adenoma.
Approximately 5% of patients having surgery
for an adenoma have multiple adenomas.
Question 18
• A 20 yo man has a BP of 240/120 mmHg. He
occasionally gets headaches when he works out.
Given the most likely do, which of the following
tests is the best for confirming it?
– A. 24 hr urine VMA and metanephrines
– B. Urine cortisol
– C. Urine aldosterone
– D. Serum epinephrine and norepinephrine levels
Answer 18
• A. The best test for pheochromocytoma is a 24
hr urine VMA and metanephrine collection
Question 19
• The test you order for the last pt is suggestive
of pheo. You order a CT but can’t find the
mass. The most sensitive test for finding the
pheochromocytoma is:
– A. MIBG
– B. MRI
– C. PET
– D. Angiogram
Answer 19
• A. MIBG scan (I-131-metaiodobenzylguanide),
a norepi analogue, is the best test
Question 20
• MIBG scan in the last pt shows a tumor in the
right adrenal. You want to get the patient
ready for surgery so you prescribe:
– A. Propanolol
– B. Phenoxybenzamine
– C. Dilaudid for headaches
– D. ASA
Answer 20
• B. The 1st drug you should prescribe is an alpha
blocker. Then you want to hydrate them as
much as possible. You may have to gradually
increase the alpha blocker until they have slight
orthostatic hypotension. This should be done 4
weeks before surgery.
• Do not prescribe beta blockers as the first drug
as it can lead to hypertensive crisis from
unopposed alpha stimulation
Question 1
• You are evaluating abdominal imaging on a 32 yo
female patient that is s/p high speed MVA. A 3-cm
lesion is noted in the periphery of the right lobe of
the liver that is homogenous with a central stellate
scar. Which of the following liver lesions is this
most likely to be?
– A. Hepatocellular carcinoma
– B. Hepatic adenoma
– C. Focal nodular hyperplasia
– D. Amoebic abscess
Question 2
• Which of the following should you recommend
for this patient?
– A. Wedge resection
– B. Right lobectomy
– C. Radiofrequency ablation
– D. Observation if asymptomatic
– E. None of the above
Answer 1
• C. Focal nodular hyperplasia- central stellate
scar in a homogenous lesion
Answer 2
• D. Observation is appropriate management in
asymptomatic FNH
Focal Nodular Hyperplasia
• FNH is a benign liver tumor that can be managed w/
observation if asymptomatic
• Resection indicated if pt has RUQ pain/ fullness, if
there is concern about the diagnosis after
observation, or if lesion has bled
• Second most common benign liver tumor
• Women; 20-40’s
• A/w OCP use
Focal Nodular Hyperplasia
• Usually found incidentally
• Unlikely to cause symptoms
• Conservative mgmt appropriate unless patient
is symptomatic- unlikely to rupture
• Radiographic features:
– Has Kupffer cells so will take up sulfur colloid
– MRI/ CT scan will show a hypervascular tumor
Question 3
• A 35 yo woman presents with right upper quadrant
pain and undergoes a CT A/P. Her PMH is negative
though she does take birth control pills. She has a 3
cm in the right lobe of the liver that appears to be
hypervascular but does not have uptake on
subsequent sulfur colloid scan. Which of the
following tumors is most likely?
– A. Hepatic cyst
– B. Hepatic adenoma
– C. Hepatocellular carcinoma
– D. Metastatic disease
Answer 3
• B. This hypervascular tumor in the right lobe
with no uptake of sulfur colloid is most likely
an adenoma.
Question 4
• Given the above scenario, which of the
following should you recommend for this
patient?
– A. Observation
– B. Radiofrequency ablation
– C. Resection
– D. Multiagent chemotherapy
Answer 4
• C. Given that you suspect a symptomatic
adenoma, resection is indicated
Hepatic Adenoma
• Higher risk- women, steroid or OCP use, type 1
collagen storage disease
• 80% are symptomatic; 10-20% with risk of
bleed from rupture
• Can become malignant
• Most common in right lobe
• Sx- pain, increased LFT, decreased BP (rupture),
palpable mass
Hepatic Adenoma
• Dx- no Kupffer cells so no uptake on sulfur
colloid scan
• MRI- hypervascular tumor with peripheral
blood supply
• Treatment-
– Asymptomatic- stop OCP- no further tx if regresses.
If no regression, pt needs resection
– Symptomatic- resection for bleeding & malignancy
risk; if multiple and not resectable-> embolize
Question 5
• A 35 yo female is found to have a 3 cm lesion in the right liver
found on CT s/p workup from an MVE. She is found to have a
3 cm tumor in the right lobe of her liver that appears
hypervascular with significant peripheral to central
enhancement. She otherwise reports feeling fine and does
not have any history of abdominal pain. Which of the
following is most likely to be true?
– A. This is the most common benign hepatic tumor
– B. Rarely it may be associated with a consumptive coagulopathy
– C. You should not biopsy this tumor as there is a risk for rupture
– D. A and B
– E. All of the above are true
Answer 5
• E. This most likely is a hepatic hemangioma so
all of the above are true
Hepatic Hemagiomas
• Most common benign hepatic tumor
• Rare risk of rupture, most asymptomatic
• More common in women
• Avoid biopsy b/c of hemorrhage risk
• Dx- MRI and CT- peripheral to central enhancement that
appears hypervascular
• Treatment-
– Conservative if asymptomatic
– Symptomatic- resection +/- embolization
– XRT and steroids if unresectable
• Rare complications- consumptive coagulopathy (Kasabach-
Merritt syndrome) and CHF- more common in children
Question 6
• A 25 yo female is found to have a 3-cm lesion in her
liver found incidentally on CT consistent with a cyst.
Which of the following is true about this lesion?
– A. This is an acquired lesion that is most likely secondary
to a history of abdominal trauma
– B. They should be resected because there is a 10%
incidence of malignant degeneration
– C. They occasionally can become infected but often can
be treated by percutaneous drainage
– D. All of the above
Answer 6
• C. A hepatic cyst is usually benign and does
not require treatment.
Solitary Cysts
• Congenital; more common in women and in
right lobe
• Resection needed if infected and can’t be
treated percutaneously
• Complications rare; most can be left alone
• Walls have characteristic blue hue
Question 7
• A 50 yo male that underwent a Billroth II gastric resection for
gastric outlet obstruction 1 month ago presents for follow-up.
He states that he experiences nausea, occasional nonbilious
emesis, and dizziness about 2 hours after he eats. Which of
the following is most likely?
– A. If you were to check his glucose when the symptoms occur, it
would most likely be high
– B. This usually does not improve with conservative treatment so you
should anticipate converting him to a Roux-en-Y
– C. Octreotide may be helpful
– D. This is most likely due to retained antrum so you will need to
consider repeat resection
Answer 7
• C. This describes late dumping syndrome and
may respond to octreotide
Dumping Syndrome
• Gastrointestinal and vasomotor symptoms
• Thought to be due to unmetered entry of ingested food
into SB
• May be seen after vagotomy & resection or division of
pyloric sphincter
• Early sx- immediately after meal from hyperosmotic
load into the small bowel
– Nausea, epigastric discomfort, borborygmi, palpitations, dizziness
• Late- 1-3 hrs after meal
– Reactive hypoglycemia in addition to sx of early dumping
Dumping Syndrome
• Usually responsive to dietary modifications (small, low-
fat diet, limit liquids with meals)
• Octreotide- can improve symptoms
– 50-100 mg before meal
– Inhibit release of vasoactive peptides from gut
– Decrease peak insulin levels and slow intestinal transit
– Prevents changes in pulse, systolic BP, and RBC volume during early dumping and
blood glc levels during late dumping
Question 8
• A 50 yo male that underwent a subtotal gastrectomy with BII
reconstruction for a T2 adenocarcinoma 1 month ago presents for
follow-up. He experiences frequent epigastric pain after meals with
subsequent bilious vomiting that does not relieve the pain. He has
lost about 15 pounds though otherwise has been stable. Which of
the following is false?
– A. He will likely respond well with dietary modification and bile chelators
– B. This usually does not improve with conservative treatment so you should
anticipate converting him to a Roux-en-Y
– C. This is most likely due to retained antrum so you will need to consider
repeat resection
– D. All of the above
– E. A and C only
Answer 8
• B. This is consistent with alkaline reflux which
is less likely to respond to conservative
treatment and will ultimately need conversion
to a Roux-en-Y
Alkaline Reflux Gastritis
• Postprandial epigastric pain, nausea, vomiting of bile,
evidence of bile reflux into stomach with gastritis
• Persist in 1-2% of patients who have had vagotomy/
drainage or resection
• Differential dx- recurrent ulceration, biliary or pancreatic
disease, afferent loop obstruction, esophagitis
• Gastric acid analysis- basal hypochlorhydria with little
increase w/ pentagastrin administration
• Obtain serum gastrum measurements to r/o ZE syndrome
and retained antrum
Alkaline Reflux Gastritis
• Endoscopy- red, friable mucosa, edema, glandular
atrophy, intestinal metaplasia
• Antacids, H2 lockers, bile acid chelators, dietary
manipulation not definitively beneficial
• Reoperation to divert contents from gastric mucosa
– Convert to Roux-en-Y GJ with 50-60 cm intestinal limb
– Very effective in eliminating bilious vomiting
– 20-30% still have pain or persistent gastric emptying
dysfunction
Question 9
• You perform a subtotal gastrectomy with B2 reconstruction on
a 62 yo man with an antral GIST tumor. On postoperative day
1 he is experiencing severe abdominal pain and nonbilious
emesis. You obtain imaging which is concerning for a
markedly dilated bowel limb proximal to the anastomosis.
Which of the following is true?
– A. This is most likely secondary to anastomtic edema so you should
place an NGT and start octreotride
– B. You would be less likely to have this complication if you did an
antecolic gastrojejunostomy
– C. This requires urgent operative intervention
– D. In the chronic form (from partial obstruction) patients may develop
microcytic anemia
– E. B, C, and D
Answer 9
• C. This picture is concerning for afferent limb
obstruction- the patient needs intervention as
he is high risk for duodenal stump blowout.
Afferent Limb Syndrome
• Partial or complete obstruction of the afferent limb
proximal to the GJ anastomosis
• Afferent limb- duodenal remnant and segment of
jejunum proximal to gastrojejunostomy
• Jejunal limb is subject to adhesions, herniaton,
obstruction, volvulus
• Rare (~1%) after Billroth II
• Can occur immediately or remotely
– Acute- operative emergency; secretions remained trapped in limb so vomiting is
nonbilious
– Chronic- from partial obstruction- relief of postprandial discomfort with
projectile bilious vomiting
Afferent Limb Syndrome
• One of main causes of duodenal stump blowout
• Prolonged stasis in chronic disease can lead to bacterial
deconjugation of bile salts-> steatorrhea, malnutrition, Vit
B12 deficiency-> megaloblastic anemia
• Pts with increased risk of developing syndrome
– Jejunal portion of afferent limb longer than 30 cm
– Antecolic GJ
– Improper closure of mesocolic defects
• Surgical intervention usually necessary
– Some attempting endoscopic therapy, though not routine
Question 10
• You are performing a diagnostic laparoscopy in a 28
yo female that you suspect has appendicitis. In
addition to an inflamed appendix, you find a 4 cm
right ovarian cystic mass. Which of the following
describes the most appropriate management?
– A. Aspirate the cyst and send the fluid for cytology
– B. Remove her appendix and perform a right
salpingoophorectomy
– C. Remove her appendix and recommend to her post-
operatively that she gets follow-up for an ovarian cyst
– D. Perform a TAH/BSO
Answer 10
• C. In a cyst on a woman of childbearing years
and unknown desire to have children, the best
step would be to perform an appendectomy
and monitor the cyst postoperatively
Question 11
• A 55 yo female is undergoing diagnostic
laparoscopy for suspected appendicitis. You find an
inflamed appendix but also note a 6 cm cystic
appearing right ovarian mass. The next best step
should be:
– A. Inform her next of kin and obtain consent for a BSO/
hysterectomy
– B. Inform her next of kin, perform an appendectomy, and
aspirate the lesion to send for cytology
– C. Inform her next of kin, perform an appendectomy and
unilateral salpingoophorectomy, await pathology
– D. None of the above
Answer 11
• C. Given this patient has a relatively large
cystic lesion and is most likely done with
childbearing, removing the affected tube and
ovary is an appropriate first step. She may
need further surgical treatment based on the
pathology.
Ovarian Mass- Benign
• May be solid or cystic
• Functional process or neoplasia
• Size, age of patient, tumor characteristics factor in on necessity of
surgical removal
• 90% tumors in women younger than 30 benign
• 80% benign in 30-50 yrs
• 50% malignant in pts over 50
• Follicular cysts- failure of a developing follicle to develop or regress.
Usually less than 8 cm
• Corpus luteal cysts- occur from hemorrhage into CL; blood-filled
yellow/ granular. Usually regress in 4-8 weeks
• Thecal luteal cyst- may be present in pregnancy with high circulating
gonadotrophins
• Chocolate cysts- pelvic endometriosis
Treatment Ovarian Mass- Incidental
• In reproductive age patient:
• Solid, unilateral- ovarian resection and frozen
section
– Usually will be benign stromal neoplasm (fibroma)
– If lesion comes back as a germ-cell malignancy-
• TAH/ BSO, omentectomy, staging bx/ LND
• Can leave uterus and contralateral adenxa behind & perform a
retroperitoneal LND, omentectomy, staging biopsy
– If bilateral dysgerminoma found (rare)-
• Resect both if pt has no interest in fertility
• Resect only larger and treat smaller with chemo if fertility
interest + or unknown
Treatment Ovarian Mass- Incidental
• Cyst-
– Usually benign and asymptomatic
– Most are serous cystadenomas
– Tx- premenopausal, less than 5 cm- observe. Cystectomy
or oophorectomy for cysts > 5cm, those with solid
elements, or postmenopausal women
• Endometriosis- common nonmalignant cause of
pelvic pain
– Treat conservatively
– If diagnosis questionable, can biopsy- try to preserve
ovarian tissue if possible
Question 12
• You are performing a laparoscopic ventral hernia repair on a
patient when the overhead light falls down, landing on and
breaking the patient’s left arm. Which of the following
describe an appropriate JCAHO response to this event?
– A. Conducting a root cause analysis
– B. Ensure that the chief of staff and OR supervisor are immediately
involved
– C. This is not considered a sentinel event because it is not due to your
medical error
– D. Development of an improvement plan and implement the
improvements
– E. A and D only
– F. A, B, and D
Answer 12
• E. Conducting a root cause analysis and
developing an improvement plan to reduce
risks are correct
JCAHO Requirement of a Sentinel Event

• Sentinel event- an unexpected occurrence that results in an


unexpected death or serious injury
• Called sentinel b/c when they occur they signal the need for
immediate investigation and response
• Not necessarily synonymous with medical errors- not all are due to
medical errors and not all med errors are sentinel events
• Accredited organizations are expected to identify and respond to all
sentinel events. An appropriate response required by JCAHO
includes:
– Conducting a root cause analysis
– Developing an action plan designed to implement improvements to reduce
risk
– Implementing the improvements
– Monitoring the effectiveness of those improvements
Question 13
• You are attempting to perform a parathyroidectomy for
hyperparathyroidism due to suspected adenoma and are using
intraoperative PTH levels. You find a small, normal-appearing
right upper, lower, and left upper gland. You cannot identify
the left lower gland. Which of the following choices is most
likely correct?
– A. The patient probably only has 3 glands so the hyperparathyroidism
was probably misdiagnosed as an adenoma and is most likely
secondary to hyperplasia
– B. Upper glands are more likely to be in unusual position than lower
glands
– C. This gland may be found in the thymus
– D. You should close at this point and repeat sestamibi postoperatively
Answer 13
• C. Around 15% of inferior parathyroids are
found in the thymus
Treatment for Missing Parathyroids
• Explore the sites where parathyroids are usually located
near the posterolateral surface of the thyroid gland
• Most inferior glands are within 1 cm from where the
inferior thyroid artery crosses the RLN
– When a lower gland is missing, it may be found in the thymus (15% of inferior
parathyroid)
• Normal superior glands are more consistent in position-
80% are near posterior aspect of upper & middle lobes
at level of cricoid cartilage
– 1% of normal upper glands may be found in paraesophageal or retroesophageal
space
Question 14
• A 60 yo man presents with dysphagia and pain after
swallowing. A barium swallow shows a smooth,
solitary mass in the lower 1/3 of the esophagus.
Which of the following is not true about this
condition?
– A. This tumor has a high malignant potential
– B. It should be removed by esophageal resection
– C. It may be adequately treated by enucleation
– D. A and B are not true
– E. All of the above are not true
Answer 14
• D. A and B are not true. This tumor is most
likely a benign esophageal leiomyoma and can
be adequately treated with enucleation.
Benign Tumors of Esophagus
• Benign tumors and cysts of the esophagus are uncommon
• Can be intramural or intraluminal
• Leiomyomas- 50% of benign tumors
– Usually solitary in lower 1/3
– Dysphagia and pain most common sx
– Dx- barium swallow
– Biopsying may scar tissue planes, complicating removal
– Can be removed by simple enucleation
• Esophageal cysts
– Enteric or bronchogenic cysts
– Usually intramural in the middle or lower 1/3
– Treat by enucleation
Question 1
•A 62 yo man presents with ischemic lesions in his right foot.
You work him up and discover a popliteal aneurysm. Which
of the following is not true about this condition?
– A. There is a significant chance he has a contralateral aneurysm
– B. It is an uncommon peripheral aneurysm
– C. Asymptomatic lesions must be repaired
– D. Around 1/3 or them also have AAA
Answer 1
• B. It is an uncommon peripheral aneurysm is
false- it is the most common
Popliteal Artery Aneurysm
• Most common peripheral aneurysm (≈ 70%)
• 50% are bilateral
• 30% also have AAA
• 20 – 30% of limb loss with distal emboli
• Elective repair of all, regardless of size
• Management options:
– Medial exploration-> proximal/distal ligation & bypass
– Acute thrombosis- preoperative thrombolytics
– Endovascular repair (not yet accepted)
Question 2
• A 65 yo man presents with progressive dysphagia.
You perform EGD which reveals a GE junction mass.
Which of the following would be an indication to
perform neoadjuvant treatment?
– A. T1 adenocarcinoma with perineural invasion
– B. T2 lesion with well-differentiated pathology
– C. 2 cm LN near tumor on EUS
– D. All of the above are indications for neoadjuvant
treatment with GE jxn tumors
Answer 2
• C. 2 cm LN near tumor on EUS
Indication Neoadj. Tx in GE Jxn Cancer

• Preoperative chemo/XRT should be done for


any:
– T3/4 tumor
– N + disease
• T1 or T2 generally surgery first.
• Staging for these tumors should also include
PET/CT.
Question 3
• You perform an EGD on a patient well-known to you
with Barrett’s for screening. He has been well-
controlled symptomatically with omeprazole. You
biopsy the GE jxn and final pathology returns as
Barrett’s with high grade dysplasia. Which of the
following should you recommend?
– A. Nissen Fundoplication
– B. Chemoradiation
– C. Esophagectomy
– D. None of the above
Answer 3
• C. Esophagectomy should be recommended
Barrett’s & High Grade Dysplasia

• Esophagectomy is considered the standard


• Have 10% incidence of associated malignancy
with high-grade dysplasia
• For non-high grade Barrett’s, endoscopic
surveillance and treatment of reflux (surgical
or medical) is appropriate
Question 4
• Which of the following is a contraindication for
sentinel lymph node biopsy in breast cancer?
– A. History of mastectomy
– B. Invasive ductal cancer with poor histological
features
– C. A patient with high grade DCIS
– D. A T4 tumor with extension into the axilla
Answer 4
• D. A very large tumor extending into the axilla
would not undergo SLN biopsy
Indications for SLN Bx in Breast Cancer

• For our purposes, SLN indicated for any invasive


cancer except T4
• For test purposes, any patient with clinically
proven nodes is not a SLN candidate (need an
ALND)
• Mastectomy is not a contraindication for SLN
• On the test, DCIS is not an indication unless it is
high grade disease or the patient is otherwise
undergoing a mastectomy
Question 5
• A 52 yo female comes into the office noting easily
expressible bloody nipple discharge on the left side.
Exam demonstrates blood and mammogram is
otherwise negative. Which of the following is most
likely to be true?
– A. This is likely an intraductal papilloma and there is a
50% chance of this being malignant
– B. It should be treated with mastectomy
– C. Localization with major duct excision Is usually curative
– D. Patients with this condition have an underlying DCIS in
2/3 of cases
Answer 5
• C. Localization with major duct excision Is usually
curative
Intraductal Papilloma of Breast
• Most commonly present with bleeding/bloody
nipple discharge.
• Generally resect via major duct excision or
needle localization if seen on imaging
• This is a benign condition- very rare to be
malignant or premalignant
Question 6
• You are examining a 67 yo male in whom you
performed a cholecystectomy on 2 weeks ago. His
final pathology reveled a focus of adenocarcinoma in
the lamina propria of the gallbladder but no invasion
into the muscularis. Which of the following should
you recommend?
– A. Resection of segment 4-5 of the liver
– B. Resection of segment 4-5 of the liver, CBD resection
with hepaticojejunostomy and regional LND
– C. Observation only
– D. Chemoradiation
Answer 6
• C. In this lesion that does not invade the
muscularis, cholecystectomy is adequate
Gallbladder Cancer
• If T1a only (does not invade muscle layer,
confined only to lamina propria), then
• cholecystectomy alone appropriate
• Anything more advanced->
– Resection of gallbladder & liver fossa (wedge
resection segment 4/5 liver)
– Regional node dissection
• Consider resection of CBD if cystic duct
involved
Question 7
• You are performing a cholecystectomy on a 35 yo female with
acute cholecystitis. It is a difficult dissection so you decide to
perform an intraoperative cholangiogram. It appears that you
have inserted the catheter through the common bile duct
itself. Which of the following is the most appropriate thing to
do next?
– A. Remove the catheter, continue with cholecystectomy, ask GI to
perform ERCP and stenting postoperatively
– B. Perform a choledochoduodeonostomy if the lesion is large
– C. Repair the lesion over a T-tube if the injury is small
– D. Perform a hepaticojejunostomy regardless of the size of the injury
Answer 7
• C. Repair the lesion over a T-tube if the injury is small
Management of CBD Injury
• Appeared on 2006 & 2007 tests with high % of
incorrect answers
• Minor injury recognized intra-operatively- repair
primarily, usually over a T-tube or stent
• Major injury recognized intra-operatively or injury
recognized post-op usually will require roux-en-y
hepaticojejunostomy
• Possible incorrect option may be
hepatico/choledochoduodenostomy
– Would not be a good choice in a nondilated duct
Risk For OPSI
• Remember most children with sickle cell
anemia auto-splenectomize by early age.
• Children younger than <4yrs are at highest risk
post-splenectomy
• Splenectomy done for thalassemia carries an
increased risk
Question 8
• Which of the following is the fuel source for
coloncytes and has been used for enemas in
those with ulcerative proctitis?
– A. Lysine
– B. Nitric oxide
– C. Butyrate
– D. Prostaglandin E
Answer 8
• C. Butyrate
Fuel Source Coloncyte
• Butyrate is the preferential source of fuel for
colonocytes
• It is usually derived from bacterial
fermentation with the colon
Question 9
• Which of the following is the most common
cause of delayed infection (several months out)
of a AAA graft?
– A. Salmonella
– B. Strep pneumoniae
– C. Staph aureus
– D. Staph epidermidis
Answer 9
• D. Staph epidermidis
AAA Infections
• 40% of aneurysmal infections caused by
salmonella
• Staph aureus and gram negative bacteria are
most common organisms in early graft
infection
• Staph epidermidis is more chronic
– Usually presents > 4 months; 2 years is average time
of presentation
Question 10
• A 35 yo male that had short bowel syndrome
secondary to multiple resections from Crohns has
been on TPN for the last 3 years. You note that he
has wounds that are very slow to heal, a periorbital
rash, and some darkening of his skin creases.
Which of the following vitamin deficiencies is this
most likely from?
– A. Vitamin K
– B. Vitamin B 12
– C. Copper
– D. Zinc
Answer 10
• D. Zinc
Zinc Deficiency in TPN
• If they ask a question relating to TPN and
vitamin deficiency, zinc is almost always the
answer (usually appears every other year)
• Zinc deficiency is associated with periorbital
rash, darkened skin creases, neuritis and
chronic, non-healing wounds
Question 11
• A 35 yo male that you are evaluating for inguinal
hernia repair informs you that he has a family
history of “getting dangerously overheated” when
undergoing anesthesia. Which of the following is
not true about this condition?
– A. Hyperthermia is usually the first symptom
– B. It can be triggered by halogenated inhalational agents
– C. Dantrolene should be used to treat
– D. Rise in CO2 is usually the first symptom
Answer 11
• Hyperthermia is usually the first symptom
Malignant Hyperthermia
• Can be genetically transferred
• Triggered by halogenated inhalational agents
(1 in 250,000)
• Can also be triggered by succinylcholine (1 in
60,000)
• Earliest sign is rise in CO2
• Hyperthermia is a relatively late finding
• Treat with dantrolene
Question 12
• Which of the following are not diagnostic criteria
for HNPCC?
– A. 3 or more family members with a HNPCC associated
cancer, one of whom is 1st degree relative of others
– B. 2 generations affected
– C. At least 1 diagnosed prior to age 50
– D. Multiple cancers in at least one affected person
Answer 12
• D. Multiple cancers in at least one affected
person
Characteristic Lynch Syndrome
• HNPCC is autosomal dominant with primary feature of
right-sided colon cancers without polyposis
• Altered genes are MLH1, MSH2, MSH6 and PMS2 (DNA
mismatch repair)
• In women, other primary risk is endometrial cancer
• Other at risk organs include stomach, small bowel, urinary
tract, ovary, pancreas and brain
• Amsterdam Criteria of diagnosis of HNPCC in a family:
– 3 or more family members with HNPCC associated cancer, one of
whom is 1st degree relative of others
– 2 generations
– At least 1 diagnosed prior to age 50
Question 13
• You are taking care of a patient that sustained 25%
TBSA 1 week ago. His wounds have mostly been
treated with debridement and topicals and he has
been otherwise stable. He has recently developed
a metabolic acidosis after you added a new topical
to his wound care regimen. Which medication is
the most likely causative agent?
– A. Acticoat
– B. Silver nitrate
– C. Mafenide
– D. Polymixin B
Answer 13
• C. Mafenide
Burn Topicals
• Silver nitrate:
– Broad spectrum, painless, cheap
– Poor eschar penetration, may cause electrolyte imbalance
• Silver sulfadiazine:
– Painless, no electrolyte abnormalities, no occlusive dressing required
– Little eschar penetration; misses Pseudomonas, idiosyncratic neutropenia
• Mafenide:
– Penetrates eschars, broad spectrum (but misses staph)
– Pain and burning on application
– 7% have allergic reaction
– May cause metabolic acidosis
– Agent of choice in contaminated burns; water‐soluble
Question 14
• Which of the following is NOT a negative
prognostic factor in thyroid cancer?
– A. Age over 50
– B. Poorly differentiated histology
– C. Female gender
– D. Large lesion
Answer 14
• C. Female gender is not a poorer prognostic
factor
Prognostic Factor Thyroid Cancer
• Remember the acronym AGES:
• Age (women < 50, men < 40)
• Grade (poorly differentiated = bad)
• Extent of disease ( extracapsular extension or
regional LN disease = bad)
• Size ( > 4cm = bad)
• Sex (women better than men)
Question 15
• A 52 yo female underwent a total thyroidectomy for a
MNG and is found to have a 3 cm nodule with papillary
cancer on final pathology. She currently is on Cytomel
(T3) and reports feeling fairly well. You recommend
that she undergoes I-131 treatment. Which of the
following is true about management of this condition?
– A. Patients can receive synthroid (T4) up to 1 week prior to I-131 treatment
– B. Most T3 supplements need to be held for at least 3 weeks prior to I-131
– C. Both of the above are true
– D. Neither of the above are true
Answer 15
• B. Most T3 supplements need to be held for at least 3
weeks prior to I-131
Preparation for I-131 scan
• Patient has to be hypothyroid for the scan
• Need to stop thyroid hormone before scan
– Synthroid/levothyroxine (T4) stop 6 weeks prior
– Cytomel/liothyronine (T3) stop 3 weeks prior
Question 16
• A 31 yo female who had a successful cadaveric
renal transplant 6 months ago presents with rising
creatnine. Biopsies are performed which confirm
rejection and BK virus infection is suspected to be a
factor. Which of the following is true about this
condition?
– A. Immunosuppression should be increased
– B. It is an uncommon virus in the general population
– C. It will respond well to Valtrex administration
– D. You should decrease her MMF dosage
Answer 16
• D. You should decrease her MMF dosage
BK Virus and Nephropathy
• BK Virus is an important factor associated with
graft nephropathy
• Prevalent in 90% of population and results in
nephropathy in 1 – 8% of transplant recipients
• No adequate anti‐viral treatment
• Treatment- Must ↓ immunosuppression, in
particular MMF
Question 17
• A 46 yo male s/p OLTX 9 years ago presents with
fever, night sweats, and diffuse lymphadenopathy.
Biopsy of an axillary node confirms lymphoma.
Which of the following would you expect to be
true?
– A. It will most likely be a Hodgkin’s-type disease
– B. He will likely respond well to chemotherapy
– C. EBV is associated with this condition
– D. It is more common in renal recipients than in
heart/lung recipients
Answer 17
• C. EBV is associated with this condition
Post-Transplant Lymphoma
• Lymphoma is 10 – 100 x more common in transplant
patients than general population
• Ranges from 1% incidence in kidney; 4 – 5% in heart/lung
patients
• Especially seen in CNS
• Usually NHL B‐cell lymphoma related to malignant
transformation of EBV
• Treatment- Reduce or withdraw immunosuppression
• High dose acyclovir may be effective
• Conventional chemotherapy generally not effective
Question 18
• Which of the following factors are not included
in the calculation of MELD score?
– A. Bilirubin
– B. INR
– C. Serum creatnine
– D. Albumin
Answer 18
• D. Albumin
MELD Score
• Model for End‐stage Liver Disease criteria for liver
failure
• Score = 3.8 * [bilirubin (mg/dL)] + 9.6 * [Cr (mg/dL)]
+ 11.2 * [INR]
• Formula predicts the risk of death in 3 months
• Average score for most patients being transplanted
currently is 15
• Additional points given for tumors suspected or
confirmed to be HCC
Question 19
• Which of the following types of collagen is
most abundant in normal skin?
– A. Type 1
– B. Type 2
– C. Type 3
– D. Type 4
Answer 19
• A. Type 1
Question 20
• Which of the following types of collagen is
likely low in Ehler-Danlos syndrome?
– A. Type 1
– B. Type 2
– C. Type 3
– D. Type 4
Answer 20
• C. Type 3
Question 21
• About what concentration of microogranisms
does it take to impede wound healing?
– A. 1,000 organisms/ cm2
– B. 10,000
– C. 100,000 (10^5)
– D. 1,000,000
Answer 21
• C. 100,000 (10^5)
Wound Strength/ Collagen
• Early tensile strength due to fibrin; late due to collagen
cross‐linking
• Tensile strength is never equal to pre‐wound
• 100,000 organisms/cm2 is enough to retard wound healing
• Ultimately the ratio of type I:III collagen is 8:1 (i.e. that of
normal skin)
• I- Most abundant, found in scar
• II- In cartilage
• III- In wound healing (low in Ehler-Danlos)
• IV- In basement membrane
• V - Found in cornea
Question 22
• Which of the following is not true about
propothyouracil (PTU)?
– A. It can be used in pregnancy
– B. It prevents production of thyroid hormone but
not peripheral conversion of T3 to T4
– C. It can cause agranulocytosis
– D. It prevents production of thyroid hormone as
well as peripheral conversion
Answer 22
• B is false. It prevents production of thyroid
hormone as well as peripheral conversion of T3
to T4
Question 23
• Which of the following patients with
hyperthyroidism do not have a significant
indication to perform thyroid resection?
– A. A 30 yo woman that is 20 weeks pregnant and
not fully responsive to PTU
– B. A 65 yo male with Plummer’s disease
– C. A 47 yo female with a large goiter that is
beginning to cause compressive symptoms
– D. A 61 yo male that has been treated with
methimazole for 1 week with minimal result
Answer 23
• D. A 61 yo male that has been treated with
methimazole for 1 week with minimal result
Indications for Thyroidectomy in
Hyperthyroidsim
• Most patients with hyperthyroidism are managed with anti-thyroid meds:
– PTU prevents production of thyroid hormone by gland and prevents peripheral
conversion of T4 to T3; most serious side effect is agranulocytosis
– Methimazole only prevents production of thyroid hormone (which has a high relapse
rate). Should be continued for at least 6 months before trying to stop
– Radioactive iodine
• Surgery usually not the tx of choice, but is always an option.
• Indications for surgery over other options:
– 1. Pregnant patient- Beta-block and then operate
– 2. Failed medical treatment
– 3. Hyperthyroidism secondary to an autonomously functioning thyroid nodule
(Plummer’s syndrome)- best tx is surgery and not RAI.
– 4. Large goiter with compressive symptoms
Question 24
• A 35 yo male with a 10 year history of Crohn’s
presents with nausea and vomiting after meals with
increasing frequency. Upper GI and EGD reveal a
stricture in the 3rd part of his duodenum that
appears to be at least 4 cm in length. Which of the
following is the best option?
– A. Whipple procedure
– B. Strictureoplasty
– C. Balloon dilation with stents if not effective
– D. Gastrojejunostomy
Answer 24
• D. Gastrojejunostomy
Duodenal Obstruction in Crohn’s
• Assuming failed medical mgmt, gastrojejunostomy is
frequently indicated
• Unlike rest of small bowel, strictureoplasty is likely
difficult in this location unless very short segment
• Resection usually not possible short of Whipple
• If stricture in a distal location, duodenojejunostomy
(side-to-side) might be best option
Question 25
• A 42 yo man that has lived overseas, including in
different parts of Asia and Australia presents with
fever and RUQ pain. CT scan reveals a lesion
consistent with an echinococcal abscess in the right
lobe of the liver. Which of the following should be
recommended to treat this patient?
– A. Right hepatectomy
– B. RFA
– C. 6 weeks of IV Flagyl and Cefepime
– D. Removal of the cyst and administration of
mebendazole
Answer 25
• D. Removal of the cyst and administration of
mebendazole
Treatment Echinococcal Abscesses

• Diagnosis by indirect hemagglutination and


ELISA
• Treatment is surgical with complete
cystectomy and avoiding spillage
• Antiparisitic rx is with mebendazole
Question 26
• A 72 yo man is following up 1 month s/p
parotidectomy for malignant disease. He notes
sweating in association with eating and the thought
of food. Which of the following is true about this
syndrome?
– A. It is likely due to dysfunction of the facial nerve
– B. It is usually a sign of recurrent malignancy
– C. Surgical treatment is usually necessary
– D. Antiperspirants should be recommended
Answer 26
• D. Antiperspirants should be recommended
Treatment Frey’s Syndrome
• Post-gustatory sweating (Frey’s syndrome) is
associated with parotidectomy
• Patients get perspiration/flushing overlying site
of parotid gland
• Auriculotemporal nerve is the culprit.
• Symptomatic treatment only- usually managed
with a roll-on anti-perspirant.
Question 27
• A 18 yo football player is brought in after a tackle
where he describes his leg “being crushed.” He is
tender over the lateral knee and leg and you note
dorsiflexion of the foot is decreased. X-ray shows
fracture of the fibular head. Which nerve is most
likely injured?
– A. Femoral
– B. Posterior tibial
– C. Common peroneal
– D. Sciatic
Answer 27
• C. Common peroneal
Paralysis Common Peroneal Nerve
• Nerve lies laterally below the knee and wraps
around the head of the fibula
• Can be injured with trauma to fibular head or
from compression of the lateral aspect of the
knee joint
• Result is foot drop with diminished dorsiflexion
of the ankle.
Question 28
• Which of the following is not a strong
preoperative predictor of cardiac
complications?
– A. JVD and gallop in a patient with CHF
– B. MI within the last 6 months
– C. Frequent PVC’s
– D. Need for multiple agents to control hypertension
Answer 28
• D. Need for multiple agents to control hypertension
Preop Predictors Cardiac Complications

• On test almost every year and widely missed


• The Goldman Index is the best recognized attempt at
correlating cardiac sxs with periop complications
– It assigns a numeric grade to multiple risk factors.
– They range from 3-11 points, depending on the symptom.
• The worst prognostic sxs are:
– 11- uncompensated CHF (evidenced by elevated CVP/JVD/S3 gallop)
– 10- recent MI (within 6 months)
– 7- > 5 PVC/min on EKG
– 7- non-sinus rhythm or PAC’s on EKG
Question 29
• A 31 yo woman underwent a CT scan s/p MVA and
incidentally is found to have a 4 cm lesion in segment 7
of her liver. The area appears to have a central stellate
scar and the rest of the hepatic parenchyma appears
normal. Which of the following is true about this
condition?
– A. It has a 20% chance of developing malignancy so should be resected
– B. It has a fairly high incidence of bleeding and rupture
– C. If a sulfur colloid scan is performed, it would appear cold
– D. It may be observed unless it is symptomatic
Answer 29
• D. It may be observed unless it is symptomatic
Focal Nodular Hyperplasia

• Typically < 5cm


• Usually incidental and asymptomatic
• Hallmark characteristic is central stellate scar
seen on imaging
• Have intact Kupffer cells so will be hot on
sulfur colloid scan
• No rx needed unless symptomatic; resection
then is indicated
Question 30
• A 40 yo female undergoes a sterotactic breast
biopsy and you are reviewing her pathology results.
Which of the following is associated with an
increased risk of breast cancer in either breast but
is not cancerous itself?
– A. Atypical ductal hyperplasia (ADH)
– B. Atypical lobular hyperplasia (ALH)
– C. LCIS
– D. All of the above
Answer 30
• D. All of the above
Question 31
• The biopsy result in this patient is consistent
with atypical ductal hyperplasia. There are no
other lesions or concerning areas in either
breast. Which of the following should be
recommended to this patient?
– A. Wire guided lumpectomy
– B. Wire guided lumpectomy and SLN biopsy
– C. Chemoradiation
– D. Observation with possible tamoxifen treatment
Answer 31
• D. Observation with possible tamoxifen treatment
Benign Proliferative Diseases with Breast Cancer
Risk
• 3 benign proliferative lesions that increase risk of
developing breast cancer but aren’t themselves pre-
malignant:
– Atypical ductal hyperplasia (ADH)
– Atypical lobular hyperplasia (ALH)
– LCIS
• Any of these found on needle biopsy warrant excision
because of risk of assoc cancer
• Risk is bilateral, not just in ipsilateral breast
• However, purpose of excision is to have adequate sample
and once excised, don’t need negative margins
• Consider chemoprevention with Tamoxifen
Question 32
• A 32 yo female is found to have a right hepatic 4 cm
hemangioma found incidentally when undergoing
CT scan for trauma. Which of the following is not
true regarding hemagioma in adults?
– A. May benefit from daily aspirin therapy- has been
shown to lead to reduction in size
– B. This patient may be observed
– C. This lesion is almost always benign
– D. If symptomatic, either enucleation or segmental
resection is appropriate
Answer 32
• A. May benefit from daily aspirin therapy- has been
shown to lead to reduction in size
Question 33
• A 4 year old patient is found to have
thrombocytopenia and shortness of breath. During
workup, a giant hepatic hemangioma is found.
Which of the following is true about this condition?
– A. Large hemangiomas may be observed if asymptomatic
– B. This lesion can lead to AV shunting and heart failure
– C. Thrombocytosis is the most common finding
– D. This lesion may involute with prostaglandin therapy
Answer 33
• B. This lesion can lead to AV shunting and heart
failure
Treatment of Liver Hemangioma
• On test 3 years in a row
• Common and usually incidental
• Giant hemangioma = > 10cm
• Diag test of choice =MRI
– Therefore if CT suggestive, use MRI to confirm.
• Treat only for symptoms
• If incidental, leave alone
• Surgical mgmt can be either enucleation or anatomic resection.
– If both are choice, go with enucleation.
• No need/benefit for non-surgical therapy.
• Giant hemangioma in pediatric patient may present with Kasabach-Merit
syndrome:
– Hepatic sequestration and thrombocytopenia
– Can also lead to AV shunting and heart failure
– Large hemangiomas in kids should be resected when found.
Question 34
• Which of the following correctly describes a T3
colon tumor?
– A. Invasion into adjacent organs/ structures
– B. Lesion involving mucosa/submucosa but not
through
– C. Lesion invading into but not through muscularis
propria
– D. Lesion invading through muscularis propria
Answer 34
• D describes T3- invasion through muscularis
propria
T3 Characteristics for Colon Cancer

Memorize T staging:
• T1- mucosa/submucosa
• T2- into but not through muscularis propria
• T3- through muscularis propria
• T4- invades other organs/structures
Question 35
• A 65 yo male presents with nonbilious emesis. He
has a of H. pylori for which he declined treatment
for 6 years ago. His films are suggestive of gastric
outlet obstruction. You suspect MALT lymphoma
based on EGD. Which of the following should you
recommend?
– A. Treatment of H. pylori
– B. Radiation only
– C. Gastrectomy
– D. Chemoradiation
Answer 35
• C. Gastrectomy is appropriate with
obstruction. Otherwise gastric MALT should be
treated by eradicating H. pylori
Treatment MALT of Stomach
• These are treated initially with eradication of
H. pylori
• If that fails-> chemo and/or XRT
• If that fails, then resect.
• Exception-> those presenting with symptoms
of gastric outlet obstruction, then surgery best
initial treatment
Question 36
• Which of the following myeloproliferative
disorders may benefit from splenectomy?
– A. AML
– B. CLL
– C. Myelofibrosis
– D. T-cell lymphoma of HIV
Answer 36
• C. Myelofibrosis
Myeloproliferative Disease Benefitting from
Splenectomy
• Myelofibrosis may benefit- patients can get
extramedullary hematopoeisis in spleen
• This condition is also associated with increased
incidence of splenic/portal vein thrombosis
postoperatively
Question 37
• A 57 yo male presents with chest pain 2 hours after
vomiting up a large meal where he feels he drank
too much wine. His workup in the ER is significant
for a left pleural effusion. Which of the following
should you do next to confirm the diagnosis?
– A. MRI
– B. EGD
– C. Esophagogram
– D. Left thoracotomy
Answer 37
• C. It is appropriate to confirm perforation with
an esophogram
Question 39
• You confirm perforation and proceed to the
operating room. Which side would you
perform the thoracotomy?
– A. Left
– B. Right
Answer 39
• A. Left
Question 40
• It is now ~ 3.5 hours after he developed symptoms. You
identify a lower esophageal perforation and that there
is fairly little contamination. The tissue appears
otherwise healthy and the patient is stable. Which of
the following is the appropriate next step?
– A. Esophagectomy and anastomosis
– B. Creation of a spit fistula, flap coverage of perforation
– C. Primary repair with placement of a G-tube and drains
– D. Mediastinal washout with planned return to OR in 24 hours
Answer 41
• C. Primary repair with placement of a G-tube and
drains
Boerhaave’s Perforation
• Classic history- chest pain, fever, respiratory distress after
emesis resulting from large meal
• Diagnosis usually suspected by left pleural effusion on CXR
• Confirmatory test- esophagram with H2O soluble contrast.
• If that isn’t a choice, then choose orally contrasted CT
• Treatment-
– If early, primary repair(left thoractomy) +/- g-tube and j-tube.
– If late, then consider spit fistula and g-tube/j-tube and delayed
restoration of GI continuity
– If severely ill, mediastinal washout/drainage alone can be done
initially with more definitive surgery after recovery.
Question 42
• A 21 yo female presents with abdominal pain and
subsequent workup reveals numerous
hamartomatous polyps throughout her bowel as
well as mucocutaneous hyperpigmentaiton. Which
of the following is true about this condition?
– A. Prophylactic colectomy should be recommended
– B. The polyps are usually adenomatous though have a low
malignant potential
– C. The oral lesions have a high malignant potential
– D. This is associated with sex cord tumors
Answer 42
• D. This is associated with sex cord tumors
Peutz-Jeghers Syndrome
• Autosomal dominant disease associated with colonic
polyps
• Thought to primarily represent hamartomas and
mucocutaneous hyperpigmentation (most often oral)
• No significant increased risk of cancer in these
polyps, so prophylactic colectomy not warranted,
just endoscopic removal.
• Syndrome is associated with unusual sex cord tumors
in women and Sertoli cell tumors in men
Question 43
• Which of the following deficiencies is
associated with glucose intolerance?
– A. Zinc
– B. Phosporous
– C. Biotin
– D. Chromium
– E. Vitamin A
Answer 43
• D. Chromium
Question 44
• Which of the following deficiencies is
associated with cardiomyopathy?
– A. Zinc
– B. Selenium
– C. Biotin
– D. Chromium
– E. Vitamin A
Answer 44
• B. Selenium
Vitamin Deficiencies
• Phosphorus: weakness, paresthesias
• Zinc: perioral rash, alopecia, poor wound healing, impaired
immunity, change in taste
• Copper: anemia, neutropenia, pancytopenia
• Iron: anemia
• Linoleic acid: dermatitis, alopecia, blurred vision, paresthesias
• Selenium: cardiomyopathy, weakness, alopecia
• Vitamin A: night blindness, skin keratosis
• Chromium: glucose intolerance (relative diabetes), peripheral
neuropathy
• Biotin: alopecia, neuritis
Question 1
• A 20 yo man who recently underwent an ex lap for
a bowel obstruction has had NGT output of over
1000 ml per day for 3 days. Which metabolic
derangement is most likely to be true?
– A. Hypochloremic, hypokalemic metabolic alkalosis
– B. Hyperchloremic, hypokalemic metabolic alkalosis
– C. Hypochloremic, hyperkalemic metabolic alkalosis
– D. Hyperchloremic, hyperkalemic metabolic alkalosis
Answer 1
• A. This person is losing water and HCl via the NG tube.
• To compensate for the water loss, the kidney reabsorbs Na in
exchange for potassium, leading to hypokalemia
• The kidney also recognizes that is losing K so it exchanges K for
H, which results in a paradoxical aciduria
• The end result is hypochloremic, hypokalemic metabolic
alkalosis with paradoxical aciduria.
• Tx- volume resuscitation with K+ containing solutions. This can
be D10 NS with 10 meq/L KCL in children or D5 ½ NS with 20
meq/L KCL in adults
Question 2 (4 part question)
• Sensitivity of a test reflects:
– A. Ability to detect disease
– B. Ability to say no disease is present
– C. Rejecting the null hypothesis incorrectly
– D. Accepting the null hypothesis when it is false
Question 3
• Specificity of a test reflects:
– A. Ability to detect disease
– B. Ability to say no disease is present
– C. Rejecting the null hypothesis incorrectly
– D. Accepting the null hypothesis when it is false
Question 4
• Type 1 error reflects:
– A. Ability to detect disease
– B. Ability to say no disease is present
– C. Rejecting the null hypothesis incorrectly
– D. Accepting the null hypothesis when it is false
Question 5
• Type II error reflects:
– A. Ability to detect disease
– B. Ability to say no disease is present
– C. Rejecting the null hypothesis incorrectly
– D. Accepting the null hypothesis when it is false
Answers 2-5
• 2- a; 3-b; 4-c; 5-d
• Sensitivity reflects ability to detect disease.
– Is equal to true positives / (true positives + false negatives)
– With high sensitivity a negative result means the pt is very unlikely to have the
disease
• Specificity reflects ability to state that no disease is
present
– Is equal to true negatives / (true negative + false positive)
– With high specificity a positive result means that the patient is very likely to
have the disease
Answers 2-5
• Type 1 errors reject the null hypothesis
incorrectly (thinking there is a correlation
when there really isn’t one)
• Type II errors accept the null hypothesis
incorrectly (you didn’t find a correlation but
one actually exists
• The most common reason for a type II error is
low sample size
Question 6
• Spontaneous closure is least likely in fistulae
originating from:
– A. Colon
– B. Esophagus
– C. Pancreas
– D. Stomach
– E. Small intestine
Answer 6
• D. Fistulae involving the stomach are the least
likely to close
Question 7
• Biliary-enteric fistula most commonly connect
the:
– A. Gallbladder and ileum
– B. Gallbladder and duodenum
– C. Common bile duct and jejunum
– D. Gallbladder and jejunum
– E. Common bile duct and ileum
Answer 7
• B. Biliary-enteric fistula most commonly
connect the gallbladder and duodenum. This is
usually caused by severe cholecystitis with
abscess and/or perforation and subsequent
erosion into the duodenal wall
• A large stone may erode into the duodenum
and subsequently cause gallstone ileus
Question 8
• Meckel’s diverticulum:
– A. Is a false diverticulum
– B. Is asymptomatic in most cases
– C. Commonly presents as gastrointestinal bleeding
in adults
– D. Commonly presents with intestinal obstruction in
children
– E. Is found in approximately 5-10% of the
population
Answer 8
• B. Meckel’s diverticulum are asymptomatic in most cases.
• They are true congenital diverticula
• Vestigal remnant of omphalomesenteric duct and most frequent
malformation of the GI tract
• 2% of population have them; males more likely to be symptomatic
• Rule of 2's:
– 2% (of the population)
– 2 feet (from the ileocecal valve)
– 2 inches (in length)
– 2% are symptomatic
– 2 types of common ectopic tissue (gastric and pancreatic)
– Most common age at clinical presentation is 2
– Males are 2 times as likely to be affected.
• It can also be present as an indirect hernia, typically on the right side,
where it is known as a Hernia of Littre
Question 9
• The most common site of gastrointestinal
lymphoma is :
– A. Small intestine
– B. Stomach
– C. Colon
– D. Duodenum
– E. Appendix
Answer 9
• B. The stomach is the most common site of
gastrointestinal lymphomas
Question 10
• Rightward shift of oxyhemoglobin dissociation
curve occurs with:
– A. Hypothermia
– B. Acidosis
– C. Decrease in 2,3-diphosphoglycerate (DPG)
– D. Hypocapnia
– E. Methemoglobinemia
Answer 10
• B. Acidosis causes a right shift of the
oxyhemoglobin dissociation curve. All of the
other listed factors shift it to the left.
Question 11
• Phosphorous:
– A. Is a major extracellular anion
– B. Is passively absorbed from the GI tract
– C. Deficiency may result in insulin resistance
– D. Deficiency is rare in hospitalized patients
Answer 11
• C. A patient that is suffering from
hypophosphatemia may demonstrate insulin
resistance.
Question 12
• Hemangioma of the liver:
– A. Is the most common benign hepatic tumor
– B. Is diagnosed with percutaneous needle biopsy
– C. Is associated with alpha fetoprotein level
– D. Should be resected as soon as diagnosed
Answer 12
• A. Hemangiomas are the most common benign tumor of the
liver.
• They are more common in women
• Rupture is rare. They are usually asymptomatic
• Avoid biopsy-> this risks hemorrhage
• Peripheral to central enhancement on CT
• Can operate or embolize if symptomatic
• Steroids can be used to treat masses that are unresectable
• Kasabach-Merritt syndrome- rare complication of
hemangioma.
– Mass causes consumptive coagulopathy and CHF
– This is more common in children
Question 13
• Leiomyoma of the esophagus:
– A. Commonly presents with dysphagia
– B. Is more common in females
– C. Is usually multiple
– D. Is usually diagnosed with endoscopic biopsy
– E. Is usually located in the lower 1/3 of the
esophagus
Answer 13
• E. Leiomyomas are usually located in the distal 1/3
of the esophagus
• They are the most common benign tumor of the
esophagus
• Dx- endoscopy, esophogram to r/o cancer
• Symptoms- dysphagia, pain
• Do not biopsy- scarring can make subsequent
resection more difficult
• Operate when they are >5cm or symptomatic.
Enucleation via thoracotomy is appropriate
Question 14
• The best operative approach to a choledochal
cyst is:
– A. Cystoduodenostomy
– B. Cystojejunostomy
– C. Roux-en-Y cystojejunostomy
– D. Cyst excision and hepaticojejunostomy
Answer 14
• D. Cyst excision with reconstruction via a
hepaticojejunostomy is the preferred treatment.
They need to be resected because of malignant
potential (15%).
• They are common in females and Asians
• 90% are extrahepatic
• Most are type 1- fusiform dilation of the CBD
• Possibly caused by angle of insertion of the duct
which leads to reflux of pancreatic enzymes during
development
Answer 14
Question 15
• Li-Fraumeni syndrome shows increased
incidence of:
– A. Colon cancer
– B. Ovarian cancer
– C. Lung cancer
– D. Breast cancer
– E. Pancreatic cancer
Answer 15
• D. Breast cancer
• Li-Fraumeni syndrome is due to a defect in the
p53 gene
• Patients are at risk for:
– Childhood sarcomas
– Breast cancer
– Brain tumors
– Leukemia
– Adrenal cancer
Question 16
• During cell cycle, DNA replication occurs in:
– A. G1 phase
– B. G2 phase
– C. S phase
– D. M phase
Answer 16
• C. DNA replication occurs during the S
(synthetic) phase
Question 17
• Small bowel obstruction in an elderly female
without external hernia or previous surgery is
most likely caused by:
– A. Small bowel neoplasm
– B. Volvulus
– C. Gallstone ileus
– D. Abdominal abscess
– E. Obturator hernia
Answer 17
• C. Gallstone ileus would be the most likely
cause of SBO in an elderly patient without a
clear hernia
Question 18
• Malignant small bowel neoplasms most
commonly present with:
– A. Weight loss
– B. Abdominal pain
– C. GI bleeding
– D. Jaundice
– E. Bowel perforation
Answer 18
• A. Of the choices, weight loss is the most
common presenting symptom of small bowel
malignancies.
• Small bowel adenocarcinomas are rare
• High proportion are in the duodenum
• When diagnosed, they need to be resected
with lymphadenectomy
Question 19
• A 50 yo man undergoes LAR followed by
chemo-XRT. 6 weeks after this is complete he
has severe proctitis and bleeding per rectum.
He has required several transfusions for this
problem. The best therapy for this pt is:
– A. Angio-embolization of the rectal arteries
– B. Abdominoperineal resection
– C. Formalin fixation of the rectum
– D. Antibiotics
Answer 19
• C. Radiation proctitis leading to severe
bleeding is best treated by formalin fixation of
the rectum
Question 20
• You perform laparoscopy for presumed
appendicitis on a 25 yo man. He has terminal
ileitis but the cecum looks normal. You should:
– A. Perform an appendectomy
– B. Close
– C. Place a drain
– D. Perform ileal resection
Answer 20
• A. Pts with presumed appendicitis but instead
are found to have terminal ileitis should
undergo appendectomy so that confusion of
ileitis and appendicitis will not occur in the
future.
• If the cecum is involved with the inflammation,
then leave the appendix- if you perform an
appendectomy in this situation you have a high
chance for a leak
Question 1
• The microtubule organizing center of the cell is
called a:
– A. Centrosome
– B. Lysosome
– C. Codon
– D. Kinetochore
– E. Leucine zipper
Answer 1
• A. The centrosome is the microtubule
organizing center.
• Other cell organelles and structures:
• Rough endoplasmic reticulum- synthesizes
proteins that are exported (increased in
pancreatic acinar cells)
• Smooth endoplasmic reticulum- lipid/ steroid
synthesis, detoxifies drugs (increased in liver
and adrenal cortex)
Answer 1
• Golgi apparatus- modifies proteins with
carbohydrates; proteins are then transported
to the cellular membrane, are secreted, or are
targeted to lysosomes
• Lysosomes- have digestive enzymes that
degrade engulfed particles and worn out
organelles
• Phagosome- Engulfed large particle; these fuse
with lysosomes
Answer 1
• Endosome- engulfed small particles; these fuse
with lysosome
• Protein kinase C- activated by calcium and
diacylglycerol (DAG)
– Phosphorylates other enzymes and proteins
• Protein kinase A- activated by cAMP
– Phosphorylates other enzymes and proteins
• Myosin- thick filaments, use ATP to slide along
actin to cause muscle contraction
Answer 1
• Actin- thin filaments that interact with myosin above
• Intermediate filaments- keratin (hair/ nails), desmin
(muscle), vimentin (fibroblasts)
• Microtubules form specialized cellular structures such
as cilia, neuronal axons, and mitotic spindles.
– Also involved in the transport of organelles in the cell (form a latticework inside
the cell)
– Centriole- a specialized microtubule involved in cell division (form spindle fibers
which pull chromosome apart)
Question 2
• Treatment for intractable abdominal pain from
chronic pancreatitis with a normal pancreatic
duct is usually:
– A. Percutaneous drainage
– B. Resection
– C. Lateral pancreatico-jejunostomy
– D. Nonoperative management
Answer 2
• B. Resection may be indicated in pts with
chronic intractable abdominal pain due to
pancreatitis
Question 3
• Treatment for intractable abdominal pain in a
patient with chronic pancreatitis and a dilated
pancreatic duct is usually:
– A. Percutaneous drainage
– B. Resection
– C. Lateral pancreatico-jejunostomy
– D. Nonoperative management
Answer 3
• C. Patients with intractable pain due to chronic
pancreatitis and a dilated duct may benefit
from lateral pancreaticojejunostomy (Puestow
procedure ) if the duct is greater than 8 mm
Question 4
• You operate on a 25 yo man with presumed
appendicitis, send a frozen section because
there is a mass at the tip of the appendix, &
path comes back as a 2.5 cm carcinoid. The
most appropriate step in his management is:
– A. Right hemicolectomy
– B. Close
– C. Post-op XRT
– D. Chemotherapy post-op
Answer 4
• A. Appendectomy is appropriate management
for carcinoid localized to the appendix, as long
as they are less than 2 cm, not at the base, and
there is no evidence for metastatic disease.
• If the above criteria are not met, perform a
right hemicolectomy
Question 5
• A newborn is found to be in severe respiratory
distress immediately following birth. CXR shows
loops of bowel filling the left chest. All of the
following are true except:
– A. Pts with this problem have about a 50% overall survival
– B. The incidence of this problem is higher on the left than
the right
– C. Both lungs are often dysfunctional
– D. Pts with this problem require immediate repair after
birth
Answer 5
• D. The above scenario is consistent with a
congenital diaphragmatic hernia. This is
associated with a 50% survival overall.
• It is more common on the left (80%) and both
lungs are usually dysfunctional- one from
compression by the bowel and the dysfunction
in the contralateral lung is not completely
understood
Answer 5
• Pulmonary hypertenstion is frequent in this
population
• Current treatment of choice in these children is
stabilization with high frequency ventilation,
ECMO, and/or inhaled NO before repair.
• Delayed repair after the patient is better able to
tolerate surgery is thought to improve survival
Question 6
• 57 yo man has a 3 cm mass in the RLL on chest CT.
Biopsy shows the mass is an adenocarcinoma. You
perform mediastinoscopy and a right paratracheal
lymph node is positive for cancer. The next best
step is:
– A. Right lower lobe resection only
– B. Right lower love resection and mediastinal lymph node
dissection
– C. Pneumonectomy and mediastinal lymph node
dissection
– D. Chemo & radiation
Answer 6
• D. Positive paratracheal nodes identified on
mediastinoscopy are considered N2 disease
and the pt is unresectable. Therefore, chemo/
XRT is the best answer.
• Some protocols would have the pt undergo
chemo/ XRT and if the pt had a great response,
resection of the primary along with
mediastinal lymph node dissection would be
offered.
Question 7
• A 50 yo woman comes in with a chief
complaint of a nodule in her neck. You get a
TSH and a T3, both of which are normal. The
most appropriate management is:
– A. Thyroid lobectomy
– B. Total thyroidectomy
– C. Ultrasound and FNA
– D. Neck MRI
Answer 7
• C. After getting baseline TFT’s you should get
an ultrasound and an FNA
Question 8
• Ultrasound of the nodule reveals a 1.2 cm
mass. The pathology comes back as follicular
cells. The next most appropriate step is:
– A. Thyroid lobectomy
– B. Nothing
– C. Neck MRI
– D. Neck CT
Answer 8
• A. Follicular cells on FNA end up being a
follicular cell cancer in 10% of cases. Therefore
you need to do a lobectomy to get a definitive
diagnosis
Question 9
• You take the above patient to the OR for a
lobectomy. Pathology shows a 1.9 cm follicular
carcinoma. The next best step is:
– A. Completion total thyroidecotmy
– B. Close
– C. Post-op chemotherapy
– D. Post-op radiation
Answer 9
• A. Thyroid cancer with a size > 1cm requires
total thyroidecotmy
• Indications for total thyroidectomy include:
– Tumor > 1 cm
– Extra-thyroidal disease (capsular invasion, clinical or
+ nodal disease, or metastases)
– Multicentric disease
– History of XRT
Question 10
• A 10 yo boy has a cyst and a cyst tract near the
angle of his mandible. The cyst has had
recurrent infections. This cyst most likely
connects to the:
– A. External auditory canal
– B. Tonsillar pillar
– C. Nasal septum
– D. Thoracic duct
Answer 10
• A. Type 1 branchial cleft cysts extend from the
angle of the mandible to the external auditory
canal.
Question 11
• A 10 yo boy presents with a cyst in his lateral neck
medial to the anterior border of the
sternocleidomastiod. This cyst most likely connects
to the:
–A. External auditory canal
–B. Tonsillar pillar
–C. Nasal septum
–D. Thoracic duct
Answer 11
• B. Type II branchial cleft cysts extend from the
anterior border of the SCM though the carotid
bifurcation to the tonsillar pillar.
Question 12
• The monoclonal antibody that specifically
blocks CD3 molecule on T cells is:
– A. OKT 3
– B. ATGAM
– C. Thymoglobulin
– D. Zenepax
Answer 12
• A. OKT3 binds the CD3 molecule on
lymphocytes. This inhibits formation of the T-
cell receptor complex and causes opsonization
of the T cell. This drug can be used to treat
rejection that is refractory to pulse steroids.
Question 13
• The monoclonal antibody that specifically
blocks IL-2 receptors is:
–A. OKT 3
–B. ATGAM
–C. Thymoglobulin
–D. Zenepax
Answer 13
• D. Zenepax is an antibody against human IL-2
receptors. It is often used with induction
therapy immediately after transplantation
Question 14
• All of the following are true of Paget’s disease
of the breast except:
– A. It describes scaly lesions on the nipple that when
biopsied demonstrate Paget’s cells
– B. Pts with this disease have underlying DCIS or
ductal carcinoma
– C. Modified radical mastectomy is appropriate for
pts found to have carcinoma
– D. Modified radical mastectomy must be done in
these patients who are found to have DCIS
Answer 14
• D. All of the options are true of Paget’s disease
of the breast. If only DCIS is found it is
appropriate to offer simple (not modified
radical) mastectomy
Question 15
• All of the following are true of familial hypocalcemic
hypocalciuria except:
–A. Patients usually have a calcium in the 9-11 range and
decreased levels of urine calcium
–B. Treatment requires parathyroidectomy
–C. The disease is caused by a defect in the PTH receptor in
the distal convoluted tubule of the kidney that causes
increased resorption of calcium
–D. Parathyroid hormone levels in these patients is normal
Answer 15
• B is incorrect. The calcium levels in these
patients is usually not that high; they do not
require parathyroidectomy
Question 16
• A 4 yo female is brought to see you because of a
painful limp. She has no other diagnosed past
medical history though there is a strong family
history for forming blood clots. X-ray shows
flattening of the femoral head. Which of the
following is most likely the cause of her limp?
– A. Osgood-Schlatter disease
– B. Legg-Calve-Perthes disease
– C. Slipped capital femoral epiphysis
– D. Congenital dislocation of the hip
Answer 16
• C. This most likely is Legg-Calve Perthes disease.
LCP is characterized by:
– AVN of the femoral head
– Usually appears in children older than 2 yrs
– Can result from a hypercoagulable state
– 10% are bilateral
– Painful gait limp
– X-ray- flattening of the femoral head
– Tx- maintain ROM with limited exercise
– Surgery if femoral head not covered by acetabulum
– Femoral head will remodel without sequlae
Question 17
• A 12 yo male that is fairly overweight presents
with painful gait. X-ray shows widening and
irregularity of the epiphyseal plate. The most
likely diagnosis is:
– A. Osgood-Schlatter disease
– B. Legg-Calve-Perthes disease
– C. Slipped capital femoral epiphysis
– D. Congenital dislocation of the hip
Answer 17
• C. This is most likely a slipped capital femoral
epiphysis. In addition to painful gait in a 10-13
yo male and the above stated x-ray findings,,
this disease is also characterized by:
– Increased AVN risk of femoral head
– Is treated by surgical pinning
Answer 17
• Regarding the other 2 listed diseases some key
points include,
• Congenital hip dislocation:
– 1.5 per 1,000 births, more common in Caucasians
– 8 times more common females
– The left hip is twice as often involved
– 25% have bilateral involvement
– Limited abduction of flexed hip, shortening of leg
– Tx- Pavlik harness- keeps legs abducted and femoral head
in acetabulum
Answer 17
• Osgood-Schlatter disease
– Tibial tubercle apophysitis
– Traction injury from the quadriceps
– Most common in adolescents aged 13-15
– Pain in front of the knee
– Irregular shape or fragmenting of tibial tubercle seen on
x-ray
– Tx either with rest for mild symptoms or cast x 6 weeks
for more severe symptoms
Question 18
• You are performing a Whipple on a patient with a
pancreatic head mass. You attempt to pass your
finger behind the pancreas from below and get a large
amount of blood return when you remove it. You
place pressure on the neck of pancreas to tamponade
the bleeding which seems to control it. You most
likely have injured:
• A. Aorta
• B. IVC
• C. Celiac artery
• D. SMV
Answer 18
• D. The SMV lies directly behind the neck of the
pancreas and is the most likely vessel injured
when attempting to get behind the neck of the
pancreas
Question 19
• The following are indications to resect MCN’s
– A. Asymptomatic masses > 4 cm in diameter
– B. Symptomatic masses of any size
– C. Small asymptomatic masses with internal
excresences or solid components
– D. All of the above
Answer 19
• D. All of the above are indications to resect
MCNs
Question 20
All of the following are true of mucinous cystic
neoplasms except:

A.High CEA in cyst fluid


B.Most commonly seen in middle aged women
C. CA 19-9 often elevated
D. Communicate with pancreatic duct
Answer 20
• D. Mucinous cystic neoplasms do not
communicate with the pancreatic duct
Question 1
• A 15 yo boy is struck in the abdomen when he goes
over the handlebars on his bike. You get a CT abdomen
with contrast but cannot identify any abnormality. He
vomits twice the next day after clears are attempted.
He also is more distended. The most appropriate
management is:
– A. Repeat the CT
– B. Zofran
– C. Exploratory laparotomy
– D. DPL
– (Answers after ? #2)
Question 2
• You repeat the child’s CT and identify a
hematoma in the 3rd portion of the duodenum.
There is no extravasation of contrast noted.
The next best step is:
– A. Exploratory laparotomy, evacuate the clot
– B. Percutaneous evacuation of the clot
– C. NGT and TPN
– D. Whipple
Answers 1&2
• A & C. If a trauma patient has a chance in clinical status or fails
to progress, you should consider repeating imaging studies.
Pancreatic, small bowel, duodenal, an diaphragmatic injuries
can present late.

• Duodenal hematomas can occur as a delayed presentation of


trauma. The 1st step is to make the diagnosis. An UGI probably
is the best test though these injuries can also be seen on CT
with IV & PO contrast.

• Next you need to assess whether there is a contrast leak. If so,


the patient needs an operation. If not, manage w/ TPN/ NPO
for 3 weeks. Most resolve with conservative mgmt.
Question 3
• Peripheral nerves regenerate at:
– A. 0.01 mm/day
– B. 0.1 mm/day
– C. 1 mm/day
– D. 5 mm/day
Answer 3
• C. Nerves regenerate at 1 mm/day
Question 4
• The most important stimulator of TNF alpha
release is:
– A. IL-1
– B. IL-6
– C. Lipopolysaccharide
– D. Shock
Answer 4
• C. The most potent stimulator for TNF-alpha is
lipopolysaccharide
Question 5
• In patients that have undergone total
proctocolectomy for FAP the most common
cause of death is:
– A. Liver cancer
– B. Adrenal cancer
– C. Duodenal cancer
– D. Small bowel cancer
Answer 5
• C. Patients with FAP can also develop
duodenal polyps which should be followed in
removed as they have malignant potential.
Question 6
• A 55 yo woman in the ICU has a PMH of a fib & PVD. She
recently developed pneumonia and is intubated. Her
anticoagulation has been held for a week. She now is on
levophed and vasopressin from presumed sepsis. You get
called because her abdomen is now distended and there
are heme + stools. She is tender but does not have gross
peritoneal signs. Lactic acid is 6, WBC = 20, BP 100/60 &
HR= 90. The most appropriate mgmt for this patient now
is:
– A. Laparotomy
– B. Peritoneal tap
– C. CT scan
– D. Angiogram
Answer 6
• D. This pt may have mesenteric ischemia. Going to CT or
the OR is not altogether unreasonable, but since she is
stable, angiogram is the most appropriate step.
– If the angio shows only constricted vessels (NOMI or nonocclusive
mesenteric ischemia) then intra-arterial infusion of papaverine
can be started. This can improve blood flow to the bowel.
– If an embolus is found, she needs to go for an embolectomy
– If this is a thrombosis, thrombolysis with TPA can he attempted.
– In any of the above cases, if the exam becomes worrisome, go
directly to the OR
Question 7
• The angiogram ordered in the previous pt shows a
meniscus sign 5 cm down from the SMA takeoff.
The jejunal branches proximal to this are normal.
This patient most likely has
– A. NOMI
– B. SMA thrombus
– C. SMA embolus
– D. Venous thrombosis
Answer 7
• C. A meniscus sign and sparing of the proximal
jejunal branches is most consistent with SMA
embolism
Question 8
• Management of the above patient requires:
– A. Continued ICU care
– B. Direct papaverine injection
– C. SMA embolectomy, resection of necrotic bowel,
2nd look operation
– D. SMA thrombectomy, SMA bypass, resect necrotic
bowel, 2nd look tomorrow
Answer 8
• C. This pt should go to the OR for
embolectomy, resection of necrotic bowel, and
a 2nd look procedure the next day.
Question 9
• All of the following are true about ovarian cancer
except;
– A. Ovarian cancer with peritoneal mets is usually treated
primarily with total abdominal hysterectomy, bilateral
oophorectomy, resection of peritoneal mets, and
omentectomy
– B. Debulking therapy can be effective in these patients
– C. Risk factors include nulliparity, early menarche, and late
menopause
– D. Risk factors include oral contraceptive pills
Answer 9
• D. The use of OCP’s lowers the risk of ovarian
cancer The rest of the answers are true.
Ovarian cancers is one malignancy where
debulking can be effective
Question 10
• Which of the following statements about von
Willebrand factor is correct?
– A. Primarily made by the liver
– B. Stored and released by lymphocytes
– C. Critical for platelets to adhere to normal
endothelium
– D. Binds to platelet-specific receptors
– E. Catalyzes the activation of factor V.
Answer 10
• D. vWF binds to platelet specific receptors
• VW disease is the most common congenital bleeding
disorder.
• Type I and II are autosomal dominant; type III is
autosomal recessive
• vWF links GpIb receptor on platelets to collagen
• Diseased pts have a normal PT, PTT may be normal
or prolonged
• Bleeding time is prolonged
Answer 10
• Risocetin test is used for diagnosis
• Type 1 is most common (70%); symptoms are
usually mild
• Type III causes the most severe bleeding
• In Type I and III there is a reduced amount of
circulating vWF.
– Tx- cryoprecipitate, DDAVP, conjugated estrogens
• Type II there is a defect in the vWF molecule itself-
pt has it but it doesn’t work well
– Tx; cryoprecipitate
Answer 10
• Remember, cryoprecipate has the highest
concentration of vWF, VIII, and is used to treat
both VWB disease and hemophilia A (factor VIII
deficiency); it also contains fibrinogen
• FFP- has high levels of all factors including V and
VIII, protein C & S, and AT-III
• DDAVP and conjugated estrogens cause the
release of VIII and vWF from endothelium
Question 11
• Delayed type hypersensitivity as observed in the
cutaneous response to tuberculin is mediated by
which of the following?
– A. IgD to tuberculoprotein
– B. IgM antibody to cell wall antigen
– C. NK lymphocytes activated with interleukin-2 (IL-2)
– CD4+ T-lymphocytes sensitized to mycobacterial antigens
– Macrophages expressing the T-cell receptor alpha beta
Answer 11
• D. CD4+ t-lymphocytes sensitized to
mycobacterial antigens. Please review the
chart on the next slide of hypersensitivity
reactions.
Answer 11
Type Description Examples

1 Immediate hypersensitivity (allergic) Bee stings, peanuts, hay


IgE mediated; mast & basophils release fever
histamine, serotonin, &bradykinin in response
to release of protein from eosinophils, which
have IgE receptors for the antigen

2 IgG or IgM reacts w/ cell bound antigen ABO & Rh incompatibility,


Graves Disease,
Myasthenia gravis, ITP
3 Immune complex deposition Serum sickness,
Rheumatoid arthritis, SLE
4 Delayed type hypersensitivity TB skin test, contact
Question 12
• All of the following are nutritionally essential
except:
– A. Proline
– B. Leucine
– C. Tryptophan
– D. Threonine
– E. Linolenic and alpha linoleic acid
Answer 12
• A. Proline
• Protein digestion begins in the stomach with
pepsin, then trypsin, chymotrypsin, and
carboxypeptidase
• Trypsinogen released by the pancreas and is
activated by enterokinase in the duodenum
– Other pancreatic protein enzymes are then activated by
trypsin
– Trypsin can also autoactivate other trypsinogen molecules
Answer 12
• Protein is broken down to amino acids, dipeptides,
and tripeptides by proteases
• These are then absorbed by secondary active
transport and released as free amino acids into the
portal vein
• Try to limit protein intake in pts with liver and/ or
renal failure to avoid ammonia buildup and possibly
worsen encephalopathy
• Nonessential amino acids are the ones that begin
with A or G as well as proline and serine
Answer 12
• Branched chain amino acids- leucine,
isoleucine, valine (“LIV”)
• Metabolized in muscle
• Possibly important in pts with liver failure
• Are all essential amino acids
Question 13
• A 55 yo man is undergoing a right hemicolectomy
for colon cancer. When you enter the abdomen
you notice that the tumor is directly invading into
the liver about 0.5 cm based on intraop ultrasound.
The best management would be:
– A. Right hemicolectomy only and leave the liver
component
– B. En bloc resection of the right colon and the wedge of
liver with the cancer
– C. Wedge the liver lesion out only
– D. Close
Answer 13
• B. Direct invasion of another structure by
colon cancer requires en bloc resection if
possible.
Question 14
• A 35 yo woman presents with tinnitus and
hearing loss. A MRI of the head shows a tumor
at the cerebello-pontine angle. The most likely
diagnosis is:
– A. Glioma
– B. Glioma multiforme
– C. Neuroma
– D. Medulloblastoma
Answer 14
• C. Unsteadiness, tinnitus, and heraring loss are
classic sx of an acoustic neuroma. A tumor at
the cerebello-pontine angle almost ensures the
diagnosis
Question 15
• The most important step in the treatment of a
pt with Zenker’s diverticulum is:
– A. Resection of the diverticulum
– B. Division of the superior laryngeal constrictor
muscles
– C. Esophagectomy
– D. Division of the cricopharyngeus muscle
Answer 15
• D. The most important step in treating a Zenker’s
diverticulum is performing a cricopharyngomyotomy.
• The anatomic problem is the failure of the upper
esophageal sphincter to relax during swallowing
• The diverticulum is usually resected but in some
situations when the diverticulum is too hard to
removed it may be suspended upward such that it
drains into the esophagus
Question 16
• A 24 yo college pitcher comes to the emergency
room with acute pain and swelling in his right arm.
An ultrasound shows a clot in the subclavian vein.
The next most appropriate step is;
– A. Thrombolytic therapy
– B. Catheter embolectomy
– C. Resection of the 1st rib
– D. Resection of the subclavian vein and reconstruction
with a 10 mm Gore graft
Answer 16
• A. Paget von Schrotter’s disease is acute thrombosis
of the subclavian vein. Classically, it presents in
pitchers or patients who have a lot of strenuous
repetitive motion in that arm.
• The initial treatment is thrombolytic therapy to open
up the vein
• It is a little controversial what to do next. Usually the
1st rib is resected during the same hospital stay in
order to not have a repeat episode
Question 17
• A 75 yo woman falls while getting up from her
wheelchair and experiences numbness in her index
finger and thumb as well as difficulty moving her
thumb.. She has a history of rheumatoid arthritis. She
has tenderness on the dorsal aspect of her wrist and at
the base of her thumb. Her findings are suggestive of
which diagnosis?
– A. Scaphoid fracture
– B. Lunate dislocation
– C. Bennet fracture
– D. Colles fracture
– E. Smith fracture
Answer 17
• D. Colles fracture
• A Colles fx is characterized by posterior angulation
and displacemennt of the distal radius, with or
without avulsion of the ulnar styloid. There can
also be a proximal shift of the distal radial fragment
• Post menopausal women with osteoporosis are at
higher risk
• RA and osteoarthritis increase the risk of this fx and
make management more difficult
Answer 17
• “Dinner fork” appearance when viewed in
pronation
• Reduce fracture manually and place in a dorsal
plaster splint
• Follow with serial x-rays to assess reduction
• If reduction is lost after swelling subsides, may
need repair closed reduction or ORIF.
Answer 17
• The scaphoid is most commonly fractured bone in
the wrist
– Occurs when the palmar surface of the hand is abducted
and “stops’ the fall
– Symptoms- tenderness in snuffbox, tubercle of scaphoid
– Fractures at the narrowest point; if untreated, proximal
part develops avascular necrosis
– Tx- thumb spica case or screw fixation
Answer 17
• Lunate/ perilunate dislocations are rare
– Usually from high-force injury
– Significant ligamentous injury
– Dislocation compresses median nerve in carpal
tunnel
– Tx- can be open or closed reduction
Answer 17
• Smith fracture-
– Distal radius fracture with fragment angulated
anteriorly
– Unstable; requires fixation
• Bennet fracture
– Carpo-metacarpal fracture/ dislocation of the
thumb.
– Tx- reduction and internal fixation + cast for 4-6 wks
Question 18
• Which of the following about the scalenus
muscle is true?
– A. The anterior body inserts into the first rib
posterior to the subclavian vein
– B. The middle body inserts into the first rib anterior
to the brachial plexus
– C. The phrenic nerve crosses the anterior body from
the medial to lateral aspect
– D. The muscle originates from the transverse
processes of C1 to C6
Answer 18
• A. The anterior body inserts on the first rib posterior
to the subclavian vein
• The scalene muscles include the anterior, middle,
and posterior body
• They originate from the transverse processes of C2-
C6
• Anterior body inserts onto the first rib between the
subclavian vein and artery + brachial plexus
• The middle body inserts on the first rib posterior to
the subclavian artery & brachial plexus
Answer 18
• The posterior body inserts onto the second rib
posterior to the subclavian artery and brachial
plexus.
• The phrenic nerve crosses the anterior body from
lateral to medial
• Thoracic duct inserts into the junction of the jugular
and subclavian veins anterior to the anterior body
of the muscle on the left side
• Anterior body may need to be resected in the
treatment of thoracic outlet syndrome as it can
compress on local neurovascular structures
Question 19
• Which of the following is characteristic of MEN
2a?
– A. Parathyroid hyperplasia, pancreatic tumor,
pituitary tumor
– B. Parathyroid hyperplasia, medullary ca of thyroid,
pheochromocytoma
– C. Medullary ca of thyroid, pheochromocytoma,
marfanoid habitus, mucosal neuroma
– D. Von Recklinghausen's Disease
Answer 19
• B is characteristic of MEN 2a
• MEN I
– Parathyroid hyperplasia, pancreatic tumor (gastrinoma
most common) , pituitary tumor (prolactinoma most
common)
– Insulinoma most common overall endocrine tumor (not in
MEN pts, however)
– Parathyroid hyperplasia is first part to become
asymptomatic
– Pancreatic tumors lead to morbidity and mortality
– Need to correct parathyroid disease first
Answer 19
• MEN 2a
– Parathyroid hyperplasia, medullary ca of thyroid,
pheochromocytoma
– Medullary ca is first part to be symptomatic due to
diarrhea from calcitonin
– Major morbidity of syndrome is medullary ca.
– Pheo needs to be treated first, though they are
usually benign
Answer 19
• MEN 2b
– Medullary ca of thyroid, pheochromocytoma,
marfanoid habitus, mucosal neuromas
– First part to become symptomatic- medullary ca
– Major morbidity is from medullary ca
– Pheo needs to be treated first
– Medullary ca from MEN 2b is more aggressive than
2a
Question 20
• You are working a patient up for a newly diagnosed lung
mass who is otherwise asymptomatic. On CT of the
chest, you note a descending aortic dissection. Which
of the following is not true?
– A. Dissections occur in the medial layer of the vessel wall
– B. Symptomatic patients may demonstrate searing chest pain and unequal
pulses or BP of upper extremities
– C. All ascending dissections need operative management
– D. All descending dissections need operative management
Answer 20
• Dissections can be classified via the Stanford or
Debakey classification
– Stanford A- any ascending involvement; B- descending
involvement only
– Debakey-
• Type 1- Ascending and descending
• Type 2- Ascending only
• Type 3- Descending only

• Most dissections start in the ascending aorta


Answer 20
• Can mimic MI
• Sx- searing chest pain, may have unequal pulses in
upper extremities
• 95% have severe hypertension
• Other risk factors- Marfan’s, previous coarcation
repair, atherosclerosis, infection, syphilis
• CXR may show wide mediastinum
• Dx with CT chest + contrast
• Dissection occurs in media layer of vessel
Answer 20
• 70% have aortic insufficiency (aortic valve cusp
sheared off)
• Coronary arteries can become occluded
• Cardiac failure from aortic insufficiency or
tamponade usually causes death
• If possible, tx medically with Nipride and beta
blockers
• Postop complications- MI, renal failure, paraplegia
– Paraplegia is caused by ischemia due to intercostal artery
occlusion and artery of Adamkiewicz
Answer 20
• Surgery aims to obliterate false lumen and
place graft
– Operate on all ascending dissections
– Operate on descending dissections with visceral,
renal, or leg ishcemia, persistent pain, or large size
(vessel dilatation)
– Need to follow these pts lifetime with serial CT’s-
30% eventually develop aneurysm that will require
surgery
Question 1
• Which of the following about coagulation is
true?
– A: PT is not affected by liver synthetic function.
– B. PTT would be abnormal in factor VII deficiency
– C. PTT would be unaffected by factor VII deficiency
– D. PTT does not measure fibrinogen
Answer 1
• C. PTT would be unaffected by factor VII deficiency.
• PT measures II, V, VII, X, and fibrinogen. It is the
best test for liver synthetic function
• PTT measures most factors including fibrinogen
except VII and XIII so it does not pick up factor VII
deficiency
• PTT of 60-90 is desirable for anticoagulation
• ACT= activated clotting time; need value of 150-200
for routine anticoagulation
Question 2
• A patient that you are treating for a lower GI bleed develops fever
soon after blood transfusion is initiated. He is otherwise stable. All
of the following are true of this pt’s most likely condition except:
– A. It is usually caused by a recipient antibody to the donor’s WBC
– B. You should treat this by discontinuing the transfusion and giving
the patient blood that is filtered for WBCs in the future
– C. It is usually due to IgG against IgA in an IgA deficient recipient
– D. It is the most common transfusion reaction
Answer 2
• C is false
• This most likely is a febrile nonhemolytic
transfusion reaction, which is the most
common reaction
– Usually due to recipient Ab to donor WBC
– Appropriate treatment includes discontinuing the
transfusion and giving the pt blood filtered for WBC
in the future
Answer 2
• Review of other transfusion reactions:
• TRALI (transfusion related lung injury)
– Rare; caused by antibodies to the recipient’s WBC
– Causes clots in pulmonary capillaries
• Anaphylaxis- Bronchospasm, urticaria, hypotension
– Caused by IgG against IgA in IgA-deficient recipient
– Tx- fluids, lasix, benadryl, pressors, epinephrine, steroids
• Urticaria- Usually nonhemolytic
– Reaction against plasma proteins or IgA in transfused blood
– Tx- benadryl, supportive
Question 3
• You perform EGD on a 50 yo and find a small mass in the
stomach. It comes back as a GIST so you decide to operate on
him. In the OR you find a small mass in the liver that comes
back as metastatic disease. The next best step is:
– A. Radiation
– B. 5 FU and cisplatin
– C. Gleevec
– D. Surveillance
Answer 3
• C. Gleevec, a tyrosine kinase inhibitor, has
been found to be extremely effective for
metastatic GIST
Question 4
• A 65 yo man undergoes an Ivor-Lewis
esophagectomy. On POD 6 he develops a 103
fever and a WBC of 18. You start IVF and send
off cultures. The next best step is:
– A. Chest CT
– B. CT abdomen
– C. Gastrograffin followed by thin barium swallow
– D. Chest MRI
Answer 4
• C. One of the most feared complications of
esophagectomy is the development of a leak.
This is usually manifested by fevers and an
elevated WBC. Severe leaks can result in
tachycardia and hypotension
• The diagnostic study of choice is a gastrograffin
followed by a thin barium swallow.
Question 5
• The barium swallow reveals a non-contained
leak at the level of the anastomosis. The next
best step is:
– A. Chest tube
– B. Re-exploration
– C. Esophageal stent
– D. Upper endoscopy
Answer 5
• B. The patient needs exploration to find the
source of the leak
Question 6
• You re-explore the above pt and find a pinhole leak
at the top of your suture line. The tissue otherwise
looks viable. The next best step in mgmt is:
– A. Take the anastomosis down and perform
esophagostomy
– B. Repair the defect and place an intercostal muscle flap
– C. Lay a drain over the hole only
– D. Place an esophageal stent
Answer 6
• B. Finding just a tiny hole in the esophagus can be
repaired though you should re-enforce the repair
with a pleural, pericardial fat, or muscle flap to cover
the area. You should also place drains.
• Keep the pt NPO x 1 week & repeat the swallow. Use
J-tube feeds until then
• Esophagostomy for diversion can be considered
though this is pretty aggressive for this problem
Question 7
• Instead of finding a tiny hole, you find that the
anastomosed stomach is necrotic. You then
should:
– A. Take the conduit down and perform
esophagostomy
– B. Repair the defect and cover with an intercostal
flap
– C. Lay a drain over the hole
– D. Place an esophageal stent
Answer 7
• A. In this scenario, you need to take down the
conduit, resect the necrotic portion of
stomach, place drains, and perform an
esophagostomy
• In 3 months you may consider reconnecting
the pt, likely with a colonic interposition
Question 8
• The most common syndrome following
vagotomy and pyloroplasty is:
– A. Diarrhea
– B. Dumping syndrome
– C. Gallstones
– D. Alkaline reflux gastritis
Answer 8
• B. Diarrhea is the most common problem
following vagotomy. Also true of post
vagotomy diarrhea:
– Occurs because of sustained postprandial organized
MMC’s
– Results in nonconjugated bile salts in colon
– Tx- cholestyramine, octreotide
– Surgery usually not necessary though reversed
jejunal interposition graft can be considered
Question 9
• A 60 yo patient that underwent antrectomy with
Billroth 2 reconstruction presents with epigastric
pain, nausea, and vomiting that often occurs after
meals. The discomfort is not relieved by vomiting.
The most likely diagnosis is:
– A. Symptomatic cholelithiasis
– B. Gastroparesis
– C. Dumping syndrome
– D. Alkaline reflux gastritis
Answer 9
• D. This most likely represents alkaline reflux
gastritis. This is characterized by:
– Postprandial epigastric pain, n/v
– Pain not relieved by vomiting
– Diagnose with EGD- will see bile in the stomach
– Tx- PPI, cholestyramine, reglan
– If medical mgmt not effective, consider conversion
to Roux-en-Y with afferent limb 60 cm distal to the
original gastrojejunostomy
Question 10
• 3 months after Billroth 1 for ulcer disease your patient notes
feeling dizzy and faint after eating as well as frequent diarrhea.
All of the following are true of the most likely condition except:
– A. Surgery is rarely necessary though if needed, options
include conversion to Roux-en-Y or a procedure that will
increase the gastric reservoir or emptying time
– B. You should first try small meals high in protein but low in
fat and carbs
– C. Octreotride is not usually effective
– D. Most cases improve medically
Answer 10
– C is not true. All of the other statements are
applicable to dumping syndrome.
• Surgery is rarely necessary though if needed, options
include conversion to Roux-en-Y or a procedure that will
increase the gastric reservoir (jejunal pouch) or emptying
time (reversed jejunal loop)
• You should first try small meals high in protein but low in fat
and carbs
• Octreotride is usually effective (take before meals)
• Most cases improve medically
Answer 10
• Dumping is due to the rapid entry of carbs into the
small bowel
• There are 2 parts
– Hyperosmotic load in bowel causes fluid shift ->
hypotension, diarrhea, dizziness
– Insulin release (2nd phase) can cause hypoglycemia (rarely
occurs)
• In addition to the other recs, pts should:
– Avoid lying down after a meal
– Avoid liquids with meals
Question 11
• The most commonly injured nerve during CEA
is:
– A. Vagus
– B. Hypoglossal
– C. Lingual
– D. Glossopharyngeal
Answer 11
• A. The most commonly injured nerve is the
vagus.
• Because the recurrent laryngeal nerve has not
yet branched from the vagus at the level where
you are operating while doing a CEA, injury to
the vagus nerve in the neck places the pt at
risk for hoarseness.
Question 12
• Which organ does not contain lymphatics?
– A. Lung
– B. Liver
– C. Spleen
– D. Muscle
Answer 12
• D. Organs that do not contain lymphatics
include brain, tendon, muscle, bone, cartilage,
and cornea
Question 13
• Myasthenia gravis affects:
– A. Beta adrenergic receptors
– B. Alpha adrenergic receptors
– C. Acetylcholine receptors
– D. Dopamine receptors
Answer 13
• C. Myasthenia gravis involves formation of
antibodies to acetylcholine receptors which
impairs binding of acetylcholine
Question 14
• A 56 yo woman presents with a weepy, scaly
lesion on her right nipple. You perform an H &
P and next:
– A. Bilateral mammogram
– B. Mammogram of the affected side
– C. Core needle biopsy
– D. FNA
Answer 14
• A. The 1st step in any pt over the age of 30
with a suspected breast problem is to get a
bilateral mammogram
Question 15
• The above pt’s MMG is normal. You should
next perform:
– A. Modified radical mastectomy
– B. Simple mastectomy
– C. Wedge biopsy
– D. Radiation
Answer 15
• C. This could be Paget’s or inflammatory
breast cancer. You need to get a skin biopsy
with some breast tissue first
Question 16
• The biopsy shows Paget’s and DCIS. You should
now offer her:
– A. MRM
– B. Simple mastectomy, including the nipple-areolar
complex
– C. Lumpectomy and axillary LND
– D. Radiation
Answer 16
• B. The appropriate treatment of Paget’s with
underlying DCIS is simple mastectomy
including the nipple-areolar complex
• Note that simple mastectomy usually means
you leave the nipple but in Paget’s you need to
take it
Question 17
• You perform a simple mastectomy and
immediately send the entire specimen to
pathology. The pathologist calls and says there
is a 1 cm ductal carcinoma with at least a 1cm
clear margin. You next should:
– A. Close
– B. Sentinel node biopsy
– C. Axillary node dissection
– D. Chest wall resection
Answer 17
• C. You need to perform a formal axillary
dissection. Since the breast tissue is already
gone sentinel node bx is no longer an option
Question 18
• A 70 yo man presents for elective umbilical
hernia repair. He has a history of ascites
secondary to cirrhosis. Should he be offered
repair?
– A. Yes, if it is significantly affecting his lifestyle
– B. Yes, if he is leaking ascites from his hernia
– C. Yes, if he is on the liver transplant list
– D. No, he should not be offered repair
Answer 18
• B: Yes, if he is leaking ascites.
• The repair of umbilical hernias in cirrhotics is
associated with a high rate of recurrence
secondary to the ascites and nutritional
deficiencies, resulting in muscular wasting and
fascial thinning.
• ~20% of cirrhotics develop an abdominal wall
hernia.
Answer 18
• Surgery on cirrhotics is complicated by risk of
hemorrhage from variceal disruption, peritonitis,
ascitic leak, and hepatic decompensation.
• Minimize ascites pre-op via salt restriction,
diuretics, and possible paracentesis. TIPS also can
be considered.
• Without adequate control of the ascites,
recurrence can be expected to be up to 73%
Question 19
• At reoperation for a recurrence of an inguinal
hernia following a laparoscopic repair, the
most common location for the recurrent hernia
is:
– A. Lateral
– B. Medial
– C. Anterior
– D. Posterior
Answer 19
• B. The most common location for breakdown
of a laparoscopic hernia repair is the medial
portion of the mesh.
• This is usually because the mesh is too small or
that it was not appropriately attached
medially.
Question 20
• Each drug is matched with a characteristic. Which of the following is
incorrect?
– A. Etomidate- causes fewer hemodynamic changes than most other anesthetics
and is fast-acting
– B. Ketamine- often used in children, can cause hallucinations, and does not
cause respiratory depression
– C. Propofol- can cause respiratory depression but provides both analgesia and
amnesia
– D. Sodium thiopental (barbiturate) is fast acting but can decrease cerebral blood
flow and blood pressure
Answer 20
• C. Propofol is not an analgesic.
– It does, however, have a rapid distribution
– It can cause hypotension and respiratory depression
– Is broken down in the liver.
– It should not be used in pts with egg allergies
• Etomidate
– Causes fewer hemodynamic changes than most other anesthetics and is fast-
acting
– Continuous infusions can lead to adrenocortical suppression
1. The lesion pictured in the xray may be associated with
dysphagia, regurgitation of food, and bad breath. All of
the following techniques may be used for treatment
EXCEPT

a. Diverticulectomy and myotomy

•b. Myotomy alone


•c. Diverticulectomy alone
•d. Internal pharyngoesophageal myotomy
•e. Diverticulopexy with or without myotomy
Answer 1
• C- Zenker’s pharyngoesophageal diverticulum
• These are a pulsion/ false diverticulum
• Treatment is indicated in symptomatic patients-
perform diverticulectomy and cricopharyngeal
myotomy to prevent recurrence
• Diverticulopexy is acceptable in elderly/high-risk
patients
• L cervical incision

• Retract SCM/carotid sheath laterally,

• thyroid/trachea medially

• Divide inferior thyroid artery

• 40-Fr bougie in esophagus, dissect pouch at

base

• extramucosal esophagomyotomy in both

directions (7-10 cm from base of pouch)

• excise diverticulum with stapler if > 2 cm


Answer 1
Other esophageal diverticula:
• Midesophageal
• Traction
• True diverticulum
• caused by external inflammation (e.g. TB) that pulls out esophagus
• new evidence suggest also may be from pulsion
• more common on right
• Epiphrenic
• distal esophagus
• Pulsion
• False
• more common on right
• Tx if symptomatic or > 3 cm
• repair via L thoracotomy, diverticulectomy, esophagomyotomy opposite side
Question 2
A 70-year-old man with a history of chronic obstructive pulmonary
disease present to the ED with diffuse abdominal pain and
distention. Labs show a WBC of 14,000 with a left shift of 90%. CT
confirms a small amount of pneumoperitoneum and mild small
bowel dilation. At laparotomy, diffuse small bowel diverticulosis of
the proximal jejunum is noted. The surgical procedure of choice
would be:

a. irrigation of peritoneal cavity and drainage


b. jejunal resection and anastomosis of the perforated segment
c. jejunal resection and diverting ostomy
d. jejunal resection of the entire segment of diverticulosis
e. closure of perforated diverticulum
Answer 2
• B. Resection and anastomosis of the
perforated segment.
• SB diverticula are usually asymptomatic
• Tx for perforation: resect perforated segment
and anastomosis
• If perforation cannot be found, no resection
• Resection of all diverticula not indicated
Question 3
• A 30 yo woman presents 7 days s/p lap chole
with nausea, RUQ pain, and malaise. The most
appropriate next step is:
– A. RUQ ultrasound
– B. Change pain meds
– C. ERCP
– D. Antibiotics
Answer 3
• A. Pain and nausea after a lap chole needs to
be worked up. Start by sending labs, including
LFT’s, then obtain a scan.
• The author of this question recommended u/s
though CT is acceptable & what we usually do
Question 4
• An 8 x 8 cm fluid collection is found in the
gallbladder fossa of this patient. The next step
should be:
– A. Antibiotics
– B. Re-exploration
– C. ERCP
– D. Percutaneous drainage
Answer 4
• D. Place a drain and see what you get
Question 5
• The drainage is bilious fluid. You should next
order:
– A. ERCP
– B. Exploratory laparotomy
– Abdominal CT
– D. Broad spectrum antibiotics
Answer 5
• A. Bile should make you think of a cystic duct
stump leak, biliary injury, or leak from Duct of
Luscka. This can often be treated with ERCP.
Question 6
• ERCP shows free extravasation of contrast
coming from the cystic duct stump remnant.
The next most appropriate step is;
– A. Exploratory laparotomy
– B. Broad spectrum abx
– C. PTC tube
– D. ERCP, sphincterotomy, stent
Answer 6
• D. Most likely, a clip fell off the cystic duct. 95% of
the time this can be effectively treated with
sphincterotomy and a temporary stent
• The cystic duct remnant eventually scars down
• This also effectively treats a duct of luschka leak
• More serious injuries may require reoperation
though this should be done 8 weeks
postoperatively
Question 7
• A 50 yo post-menopausal woman is diagnosed with
ovarian cancer. You find that she had disease in
both ovaries though all of the peritoneal &
diaphragmatic biopsies as well as peritoneal
washings are negative. What stage is this patient’s
cancer?
– A. I
– B. II
– C. III
– D. IV
Answer 7
• A. Ovarian cancer limited to both ovaries is
stage I
Question 8
• The above patient requires:
– A. Oophrectomy and partial oophrectomy
– B. Bilateral oophrectomy
– C. Bilateral oophrectomy and salpingectomy
– D. Total abdominal hysterectomy and bilateral
oophorectomy
Answer 8
• D. Even though the disease is stage 1, total
abdominal hysterectomy and bilateral
oophrectomy is indicated
Question 9
• A 35 yo man is POD#6 from a cadaveric kidney
transplant develops a rise in his creatnine. The
most appropriate step is:
– A. Emergent re-operation
– B. Angiography
– C. OKT 3
– D. Ultrasound
Answer 9
• D. The 1st step in the work-up of a kidney
transplant pt with an elevated creatnine or
decreased UOP should be an ultrasound.
• Ultrasound may assess vascular supply, looks for
ureteral compression, and can identify fluid
collections consistent with either urine leaks,
lymphoceles, hematomas, or seromas.
• A biopsy can be done at the same time
• Some people empirically treat with steroids while
waiting for the results of the biopsy
Question 10
• In the previous pt, the ultrasound shows flow
acceleration and narrowing at the level of the
arterial anastomosis. The next step should be:
– A. Emergent reoperation
– B. Angiography
– C. OKT 3
– D. Additional biopsy
Answer 10
• B. Angiogram with angioplasty and stent
placement is the treatment of choice for a tight
arterial anastomosis following kidney
transplantation
Question 11
• Instead of the above, ultrasound is normal.
The next most appropriate step is;
– A. Emergent re-operation
– B. Angiography
– C. OKT 3
– D. Biopsy
Answer 11
• D. If there is no obvious mechanical problem
with the graft you should obtain a biopsy
Question 12
• Biopsy shows acute tubulitis. This is consistent
with:
– A. Acute rejection
– B. UTI
– C. Chronic rejection
– D. Renal vein thrombosis
Answer 12
• A. Acute tubulitis is consistent with rejection.
A more severe reaction would involve
vasculitis.
• This pt should be started on pulse dose
steroids.
• Creatnine should be followed and the kidney
should be rebiopsied in 5-7 days
Question 13
• After being treated with pulse-dose steroids for a
week, you re-biopsy the kidney and there is no
longer any signs of acute rejection although there is
a large fluid collection anterior to the kidney. You
send the fluid off and the creatnine is 20 (serum is
0/8.) The next step should be:
– A. Explant the kidney
– B. Try to repair the cysto-ureteral anastomosis
– C. Place a stent and percutaneous drainage
– D. Nothing
Answer 14
• C. The most appropriate treatment for a urine
leak in most cases is stent placement across the
ureteral anastomosis and percutaneous drainage
of the fluid collection.

• Trying to redo the anastomosis is usually


unsuccessful though it may be appropriate if the
anastomosis fell apart very early, like POD 1
Question 15
• Which of the following can cause prolonged
ventilatory requirements?
– A. Hypokalemia
– B. Hypomagnesemia
– C. Hyponatremia
– D. Hypophosphatemia
Answer 15
• D. Hypophosphatemia can result in prolonged
ventilation due to relative ATP insufficiency (need
phosphate to convert ADP to ATP).

• K and Mg are important ions involved in gut motility


(these should be replaced in pts with a prolonged
ileus.)

• Deficiencies in K and Mg can also contribute to heart


arrhythmias such as atrial fibrillation
Question 16
• Which of the following may be associated with
increased gastric acid and increased gastrin
levels?
– A. Gastric cancer
– B. People on long-term PPI
– C. Patients with pernicious anemia
– D. A patient with retained antrum after partial
gastrectomy for ulcer disease
Answer 16
• D. Patients with retained antrum may have high gastrin
and high acid levels

• Other diseases that may lead to high gastrin and high acid-
ZES, antral cell hyperplasia, renal failure, gastric outlet
obstruction, short bowel syndrome

• Disease that may cause high gastrin and normal to low


acid- Pernicious anemia, cancer, chronic gastritis,
postvagotomy pts, people on medical acid suppression
Question 17
• Which of the following is not true about
lobular carcinoma in situ (LCIS)?
– A. 40% get cancer in either breast
– B. It is considered a marker for breast cancer- in
itself it is not premalignant
– C. It has no calcifications on MMG and is not
palpable
– D. If you find this when you are resecting a lesion
for other lesions, you need to go back and get
negative margins
Answer 17
• D. If LCIS is found you do not need to go back and get
negative margins. A, B, and C are true.
• Other facts about LCIS:
– Primarily found in premenopausal women; usually an incidental finding
– Pts with it that ultimately develop breast cancer get ductal cancer (70%)
– Multifocal disease is common
– 5% risk of having a synchronous cancer when this is diagnosed
– Treatment options include monitoring, tamoxifen, and bilateral subcutaneous
mastectomy without axillary dissection
Question 18
• Which of the following statements is not true?
– A. Macula densa senses low Na/Cl and produces renin
– B. Renin converts angiotensin 1 to angiotensin II
– C. Angiotensin I is converted to angiotensin II in the
lung by ACE
– D. Angiotensin II is a vasoconstrictor that causes an
increased aldosterone level -> sodium is therefore
retained and K/H is lost in the urine
Answer 18
• B is incorrect.
• Macula densa senses low Na/CL -> renin is then
produced
• Renin converts angiotensiongen to angiotensin I
• Angiotensin I gets converted to AT II in the lung by
angiotensin converting enzyme (ACE)
• AT II causes vasoconstriction and aldosterone
release
• Aldosterone leads to sodium uptake as well as loss
of H & K
Question 19
• What should the rate of maintenance fluid be
on a 50 kg boy?
– A. 120 cc/hr
– B. 90 cc/hr
– C. 60 cc/hr
– D. 30 cc/hr
Answer 19
• B. For the 1st 10 kg, the rate is 4 cc/kg/hr
• For the next 10 kg, the rate is 2 cc/kg/hr
• For anything after that, the rate is 1 cc/kg/hr
Question 20
• Erythema multiforme is most likely with which
of the following antibiotics?
– A. Quinolones
– B. Bactrim
– C. Erythromycin
– D. Ceftriaxone
Answer 20
• B. Bactrim has many side effects associated
with it. These include allergic reactions, renal
damage, erythema multiforme, and hemolysis
in G6PD deficient pts
Question 21
Of the following, which would be the best initial treatment
of most patients with bleeding esophageal varices?

a. Intravenous pitressin and nitroglycerin


b. Placement of a Sengstaken-Blakemore tube
c. Emergency esophagoscopy with sclerotherapy or variceal
ligation
d. Transjugular intrahepatic portosystemic shunt (TIPS)
e. Intravenous octreotide
Answer E

- Pitressin induces cardiac ischemia, which can be offset by nitro

- Octreotide is safer, acts by decreasing splanchnic blood flow

50 ug bolus, then 50 ug/h for 48-72 hours

The modified Sengstaken-Blakemore tube.


Note the accessory nasogastric (N-G) tube
for suctioning of secretions above the
esophageal balloon and the two clamps, one
secured with tape, to prevent inadvertent
decompression of the gastric balloon.
Question 1
• Regarding the management of Crohn’s disease, which of
the following are not true?
– A. If you operate on a patient that you suspect has appendicitis and you find
inflammation consistent with Crohn’s that does not affect the cecum, you
should still do an appendectomy as the fistula rate will not increase
– B. Perianal abscesses in Crohn’s should be treated the same as other perianal
abscesses
– C. Aphthous ulcers is more common in ulcerative colitis
– D. A patient that is found to have granulomas, fissures, and submucosal fibrosis
is more likely to have Crohn’s
Answer 1
• C is not true. All of the rest are accurate
statements regarding Crohn’s disease
Question 2
• Which of the following regarding lung physiology is
not true?
– A. The minimal pre-thoracotomy PFT’s need to be an FEV1
of at least > 2L / 1L / 0.6L for
pneumonectomy/lobectomy/wedge resection
consecutively
– B. Small cell lung cancer is the most common as well as
the most likely to demonstrate neoplastic syndrome
– C. Type 2 alveoli make surfactant
– D. Pancoast tumors involve sympathetic chain and/or
ulnar nerve
Answer 2
• B is not true. Although small cell neoplasms
are the most likely to cause paraneoplastic
syndromes, they are not the most common
lung cancer
• Adenocarcinoma is the most common lung
cancer
Question 3
• You perform a core needle biopsy in a patient
with an axillary mass and it comes back as a
melanoma. You cannot find any skin lesions
over the patients body. Which of the following
is the most appropriate next step?
– A. Formal axillary lymph node dissection
– B. Surveillance
– C. Mastectomy
– D Chemotherapy and radiation
Answer 3
• A. ALND is the most appropriate in this case.
• There are a number of possibilities in a scenario like
this
• 1. The patient has a primary somewhere that you
can’t find as some melanomas are not pigmented.
• 2. The primary lesion could have regressed
spontaneously
• 3. Different from breast cancer you do a I, II, & III
axillary dissection with melanoma
Question 4
• The earliest and most specific sign of malignant
hyperthermia is:
– A. High fever
– B. Hypotension
– C. Increase in end tidal CO2
– D. Tachycardia
– E. Hypoxia
Answer 4
• C. Increase in end-tidal CO2
• Malignant hyperthermia is characterized by:
– A defect in calcium metabolism. Calcium released from
sarcoplasmic reticulum causes muscle excitation-
contraction syndrome
– After an increase in ETCO2, fever, tachycardia, rigidity,
acidosis, and hyperkalemia begin
– 10/mg dantrolene inhibits calcium release. Also use
cooling blankets, fluids, bicarb
– Is associated with succinylcholine (the only depolarizing
agent)
Question 5
• The most frequent manifestation of blunt
myocardial contusion is:
– A. AV block
– B. Atrial flutter
– C. Premature ventricular contractions
– D. Premature atrial contractions
– E. Atrial fibrillation
Answer 5
• C. Premature ventricular contractions.
• V-tach and V-fib are the most common
arrhythmias with this injury that lead to death
• Risk of death highest in the 1st 24 hrs
• SVT is the most common arrhythmia overall
• Monitor these patients closely
Question 6
• The most commonly injured nerve under
general anaesthesia is:
– A. Radial nerve
– B. Ulnar nerve
– C. Median nerve
– D. Brachial plexus
– E. Common peroneal nerve
Answer 6
• B. The ulnar nerve. It is important to make
sure the arms are positioned carefully to
prevent ulnar injury
Question 7
• Respiratory distress associated with goiter is
most commonly caused by:
– A. Recurrent laryngeal nerve palsy
– B./ Malignant tracheal invasion
– C. Retrosternal extension of goiter
– D. Hemorrhage in a large goiter
Answer 7
• C. Retrosternal extension of goiter.
• Goiter describes any abnormal thyroid enlargement
• Most identifiable cause- iodine deficiency
• Diffuse enlargement without functional
abnormality- non-toxic goiter
• Can be treated with iodine suppression, thioamides,
subtotal thyroidectomy, or lobectomy if medical
therapy is ineffective
Answer 7
• Substernal goiter
– Usually secondary- vessels originate from superior
and inferior thyroid arteries
– Primary substernal goiter is rare; blood supply
originates from innominate
– Can cause airway compression
– Tx similar to other goiters
Question 8
• Malignant hyperthermia:
– A. Can be induced by local anasthetics
– B. Can be induced by nondepolarizing muscle
relaxants
– C. Can be induced by nitrous oxide
– D. Is related to disordered K+ metabolism
– E. Is more common in children than adults
Answer 8
• E. Malignant hyperthermia is more common in
children than in adults
Question 9
• The most common cardiac anomaly found in
adults is:
– A. Atrial septal defect
– B. Ventricular septal defect
– C. Transposition of great vessels
– D. Coarctation of the aorta
Answer 9
• A. Atrial septal defect
Question 10
• The most common complication of epidural
analgesia is:
– A. Hypotension
– B. Nausea
– C. Respiratory distress
– D. Deep vein thrombosis
Answer 10
• C. Respiratory distress is the most frequent
complication following epidural anaesthesia
• Epidurals cause sympathetic denervation and
vasodilation
• Morphine in epidural can cause respiratory
depression
• Lidocaine in epidural can cause decreased HR and
BP
• Dilute concentrations can allow sparing of motor
function
Answer 10
• Tx for acute hypotension and bradycardia- turn
epidural down, give fluids, phenylephrine, atropine
• T5 epidural can affect cardiac accelerator nerves
• Epidural is contraindicated with:
– Hypertrophic cardiomyopathy
– Cyanotic heart disease-> can get inadvertent spinal
anaesthesia
Question 11
• The lateral boundary of a femoral hernia is:
– A. The femoral nerve
– B. The femoral artery
– C. The femoral vein
– D. The lacunar ligament
Answer 11
• C. The femoral vein
• Boundaries for femoral canal: Cooper’s ligament, the
inguinal ligament, and femoral vein
• Femoral sheath- artery and femoral branch of
genitofemoral nerve is most lateral
• Vein is medial to artery
• Femoral canal is medial to vein- contains preperitoneal
fat, lymph nodes (Cloquet’s), and connective tissue.
• Femoral hernia occurs when the blind end of the
femoral canal becomes a opening through which
peritoneal sac can protrude.
Answer 11
• Femoral hernia boundaries:
– superior- Inguinal ligament
– inferior- pectineal ligament
– medial- lacunar ligament (attaches to pubis &
connects inguinal and pectineal ligaments)
– lateral- femoral vein
Question 12
• A characteristic of primary hyperaldosteronism
is:
– A. Hyperkalemia
– B. Hyperreninism
– C. Hypertension
– D. Hyperplasia of zona reticularis
– E. Hyperplasia of zona fasciculata
Answer 12
• C. Hypertension
• Hyperaldosteronism (Conn’s syndrome)
– Characterized by hypertension secondary to sodium
retention without edema
– Hypokalemia, weakness, polydipsia, polyuria
– Primary disease ~80% because of adenomas;
hyperplasia ~10%, ovarian tumors and carcinoma
less common
Question 13
• Regarding the adrenal gland:
– A. The adrenal cortex does not have nerve supply
– B. The adrenal medulla is supplied by postganglionic
adrenergic fibers
– C. The right adrenal vein drains into the renal vein
– D. The left adrenal vein drains into the IVC
Answer 13
• A. The adrenal cortex does not have a nerve
supply
• The right adrenal vein drains into the IVC
where the left drains into the renal
Question 14
• During an operation for appendicitis, the
appendix is noted to be normal but the
fallopian tube is thickened and surrounded by
purulent exudate. The operative management
should be:
– A. Appendectomy
– B. Appendectomy and salpingectomy
– C. Salpingectomy
– D. No operative intervention
Answer 14
• A. Perform an appendectomy and treat the
salpingitis/ PID with antibiotics. It is safe to
perform an appendectomy in these
circumstances.
Question 15
• In the preoperative preparation of
pheochromocytoma, medications should be
give in which order?
– A. Diuretics and then alpha blockers
– B. Alpha blockers and then beta blockers
– C. Beta blockers and then alpha blockers
– D. Diuretics and then beta blockers
Answer 15
• B. The appropriate treatment preoperatively is
to first start alpha blockers and then beta
blockers if needed.
• Treating with a beta blocker first may lead to
unopposed alpha stimulation, hypertensive
crisis, and heart failure in pts with a
cardiomyopathy
Question 16
• Hyperinsulinism in a newborn is most likely
caused by:
– A. Nesidioblastosis
– B. Glycogen storage disease
– C. Benign insulinoma
– D. Malignant insulinoma
Answer 16
• A. Nesidioblastosis (an old-school term I want you to be familiar with
in case it comes up…)
• Nesidioblastosis is a term for hyperinsulinemic hypoglycemia
attributed to excessive function of pancreatic beta cells
• The abnormal histological aspects of the tissue included the presence
of islet cell enlargement, islet cell dysplasia, beta cells budding from
ductal epithelium, and islets in apposition to ducts.
• Most congenital hyperinsulinism is caused by different mechanisms
than excessive proliferation of beta cells in a fetal pattern
• This term fell into disfavor after it was recognized in the late 1980s
that the characteristic tissue features were sometimes seen in
pancreatic tissue from normal infants and even adults, and is not
consistently associated with hyperinsulinemic hypoglycemia.
Question 17
• A patient with abdominal wall desmoid tumor
should be screened for:
– A. Lung cancer
– B. Colon polyps
– C. Breast cancer
– D. Medullary thyroid cancer
– E. Pancreatic cancer
Answer 17
• B. Colon polyps
• Gardner’s syndrome- FAP + desmoid tumors
• FAP
– An autosomal dominant disease
– All get colon cancer by age 40
– APC gene on chromosome 5
– 20% of cases are spontaneous
– Polyps are not present at birth; form in puberty
Answer 17
• Perform flex sig to check for polyps in pts
suspected to have FAP
• Need prophylactic colectomy at age 20
• Surgical options- j-pouch; total
proctocolectomy with ileostomy
• Following colectomy, need to watch for
periampullary tumors of the duodenum
• Turcot’s syndrome- colon cancer a/w APC
gene, brain tumors
Question 18
• Neutropenic enterocolitis is a complication of:
– A. Cytarabine
– B. Cyclophosphamide
– C. Doxorubicin
– D. Cisplatin
Answer 18
• A. Cytarabine
• Bleomycin and busulfan- pulm fibrosis
• Cisplatin- nephrotoxic, ototoxic, neurotoxic
• Carboplatin- myelosuppresion
• Vincristine- peripheral neuropathy, neurotoxic
• Vinblastine- myelosuppression
• Cyclophosphamide- SIADH, gonadal dysfunction,
hemorrhagic cystitis
• Methotrexate- renal toxicity and radiation recall
Question 19
• The most common intra-abdominal solid
tumor in children is:
– A. Nephroblastoma
– B. Neuroblastoma
– C. Rhabdomyosarcoma
– D. Fibrosarcoma
Answer 19
• B. Neuroblastomas are the most common
intra-abdominal solid tumors in children
Question 20
• An indication for laparotomy in neonatal
necrotizing enterocolitis is:
– A. Distended bowel loops
– B. Thickened bowel wall
– C. Abdominal wall erythema
– D. Pneumatosis intestinalis
Answer 20
• C. Abdominal wall erythema
• Other indications to operate- free air,
peritonitis, clinical deterioration
• Need barium contrast enema before taking
down ostomies in patients that formerly had
NEC to rule out distal stenoses
1. A 55 year-old male, who is noted to have diarrhea, flushing and
bronchoconstriction as well as right-sided cardiac valvular disease, is most
likely to have his primary tumor where?

a) appendix
b) bronchus
c) rectum
d) stomach
e) ileum
1. A 55 year-old male, who is noted to have diarrhea, flushing and
bronchoconstriction as well as right-sided cardiac valvular disease, is most
likely to have his primary tumor where?

a) appendix
b) bronchus
c) rectum
d) stomach
e) ileum
2. A patient has a serum sodium of 115 but is asymptomatic.
His sodium is corrected by the next day and is 135. However,
the patient develops seizures and cerebral edema. What is
the most likely etiology of this?

a) central pontine myelinolysis


b) hepatic encephalopathy from unrecognized liver failure and
poor sodium homeostasis
c) hypercalcemia
d) hypomagnesemia
e) Wernicke’s encephalopathy
2. A patient has a serum sodium of 115 but is asymptomatic.
His sodium is corrected by the next day and is 135. However,
the patient develops seizures and cerebral edema. What is
the most likely etiology of this?

a) central pontine myelinolysis


b) hepatic encephalopathy from unrecognized liver failure and
poor sodium homeostasis
c) hypercalcemia
d) hypomagnesemia
e) Wernicke’s encephalopathy
3. Which hormone can be used to stimulate small bowel
hypertrophy in order to lessen the symptoms of short gut
syndrome?

a) glucagon
b) enteroglucagon
c) somatostatin
d) growth hormone
e) insulin-like growth factor
3. Which hormone can be used to stimulate small bowel
hypertrophy in order to lessen the symptoms of short gut
syndrome?

a) glucagon
b) enteroglucagon
c) somatostatin
d) growth hormone
e) insulin-like growth factor
4. Which is the only pancreatic enzyme which is not
secreted as a prohormone? It is active at the time of
secretion into the pancreatic duct.

a) phospholipase A2
b) chymotrypsin
c) colipase
d) carboxypeptidase A
e) lipase
4. Which is the only pancreatic enzyme which is not
secreted as a prohormone? It is active at the time of
secretion into the pancreatic duct.

a) phospholipase A2
b) chymotrypsin
c) colipase
d) carboxypeptidase A
e) lipase
5. In which portion of the gastrointestinal tract are
proteins mostly absorbed?

a) stomach
b) duodenum
c) jejunum
d) ileum
e) colon
5. In which portion of the gastrointestinal tract are
proteins mostly absorbed?

a) stomach
b) duodenum
c) jejunum
d) ileum
e) colon
6. Which of the following is the toxic component of
lipopolysaccharide (LPS)?

a) O-specific antigen
b) lipid A moiety
c) core M protein
d) M protein coat
e) capsule wall
6. Which of the following is the toxic component of
lipopolysaccharide (LPS)?

a) O-specific antigen
b) lipid A moiety
c) core M protein
d) M protein coat
e) capsule wall
7. Mechanism of inheritance in MEN I, MEN IIa and
MEN IIb?

a) genomic, non-dominant
b) X-linked
c) autosomal dominant
d) autosomal recessive
e) sex-linked
7. Mechanism of inheritance in MEN I, MEN IIa and
MEN IIb?

a) genomic, non-dominant
b) X-linked
c) autosomal dominant
d) autosomal recessive
e) sex-linked
8. Which bacterial genus produces the most potent
exotoxins?

a) Staphylococcus
b) Pseudomonas
c) Klebsiella
d) Streptococcus
e) Serratia
8. Which bacterial genus produces the most potent
exotoxins?

a) Staphylococcus
b) Pseudomonas
c) Klebsiella
d) Streptococcus
e) Serratia
9. A 22 year-old male undergoes extensive small bowel
resection for complications related to Crohn’s disease.
He develops short gut syndrome. Which vitamin is he
most likely to malabsorb?

a) riboflavin
b) niacin
c) vitamin B6
d) vitamin B12
e) vitamin C
9. A 22 year-old male undergoes extensive small bowel
resection for complications related to Crohn’s disease.
He develops short gut syndrome. Which vitamin is he
most likely to malabsorb?

a) riboflavin
b) niacin
c) vitamin B6
d) vitamin B12
e) vitamin C
10. What nerve injury is associated with a proximal
shaft of humerus fracture?

a) radial
b) musculocutaneous
c) axillary
d) median
e) long thoracic
10. What nerve injury is associated with a proximal
shaft of humerus fracture?

a) radial
b) musculocutaneous
c) axillary
d) median
e) long thoracic
11. What is the mechanism for suppression of gastrin
release?

a) antral distention
b) elaboration of secretin which has a reciprocal
relationship with gastrin
c) decreased levels of pancreatic polypeptide
d) direct vagal inhibition via release of gastrin-inhibiting
peptide (GIP)
e) antral acidification
11. What is the mechanism for suppression of gastrin
release?

a) antral distention
b) elaboration of secretin which has a reciprocal
relationship with gastrin
c) decreased levels of pancreatic polypeptide
d) direct vagal inhibition via release of gastrin-inhibiting
peptide (GIP)
e) antral acidification
12. Cholesterol solubility depends on which three
factors?

a) cholesterol, bile salts, and phospholipids


b) cholesterol, bilirubin, and phospholipids
c) cholesterol, proteins, and bile salts
d) cholesterol, proteins, and phospholipids
e) cholesterol, bilirubin, and proteins
12. Cholesterol solubility depends on which three
factors?

a) cholesterol, bile salts, and phospholipids


b) cholesterol, bilirubin, and phospholipids
c) cholesterol, proteins, and bile salts
d) cholesterol, proteins, and phospholipids
e) cholesterol, bilirubin, and proteins
13. The surface gastric epithelial cell secretes which
substance?

a) mucus
b) HCl
c) pepsin
d) intrinsic factor
e) bicarbonate
13. The surface gastric epithelial cell secretes which
substance?

a) mucus
b) HCl
c) pepsin
d) intrinsic factor
e) bicarbonate
14. Mass movement is the characteristic motility pattern
of which portion of the gastrointestinal tract?

a) stomach
b) duodenum
c) jejunum
d) ileum
e) colon
14. Mass movement is the characteristic motility pattern
of which portion of the gastrointestinal tract?

a) stomach
b) duodenum
c) jejunum
d) ileum
e) colon
15. The characteristic pattern of motility in the fasted
state is the migrating myoelectric complex (MMC).
Which hormone is responsible for regulating the MMC?

a) pancreatic polypeptide
b) bombesin
c) somatostatin
d) motilin
e) neurotensin
15. The characteristic pattern of motility in the fasted
state is the migrating myoelectric complex (MMC).
Which hormone is responsible for regulating the MMC?

a) pancreatic polypeptide
b) bombesin
c) somatostatin
d) motilin
e) neurotensin
16. When there is an increased stimulus for pancreatic
exocrine secretions, which electrolyte will decrease the
most in the pancreatic effluent?

a) sodium
b) chloride
c) bicarbonate
d) potassium
e) calcium
16. When there is an increased stimulus for pancreatic
exocrine secretions, which electrolyte will decrease the
most in the pancreatic effluent?

a) sodium
b) chloride
c) bicarbonate
d) potassium
e) calcium
17. What is the mechanism of activation of the
pancreatic proenzymes?

a) activation by enterokinase
b) activation by pepsin
c) activation by duodenal acidification
d) activation by trypsin
e) activation by lipase
17. What is the mechanism of activation of the
pancreatic proenzymes?

a) activation by enterokinase
b) activation by pepsin
c) activation by duodenal acidification
d) activation by trypsin
e) activation by lipase
18. Which of the following is the best indicator of
sepsis?

a) tachycardia
b) decreased oxygen consumption
c) decreased cardiac output
d) peripheral vasoconstriction
e) decreased systemic vascular resistance
18. Which of the following is the best indicator of
sepsis?

a) tachycardia
b) decreased oxygen consumption
c) decreased cardiac output
d) peripheral vasoconstriction
e) decreased systemic vascular resistance
19. What is the most common mechanism for the
development of resistance by a bacterial cell?

a) spontaneous mutation
b) development of multidrug resistance phenotype
c) chromosome transfer
d) bacteriophage infection
e) plasmid acquisition
19. What is the most common mechanism for the
development of resistance by a bacterial cell?

a) spontaneous mutation
b) development of multidrug resistance phenotype
c) chromosome transfer
d) bacteriophage infection
e) plasmid acquisition
20. You can declare someone brain-dead if they
exhibit one of these?

a) hypothermia
b) gag reflex
c) corneal reflex
d) brisk deep tendon reflexes
e) pupillary light reflex
20. You can declare someone brain-dead if they
exhibit one of these?

a) hypothermia
b) gag reflex
c) corneal reflex
d) brisk deep tendon reflexes
e) pupillary light reflex
21. A cirrhotic patient with intractable ascites has a
peritoneovenous shunt placed and is noted to begin
oozing from many sites including his incisions. What
is the best explanation for this?

a) disseminated intravascular coagulation


b) unrecognized preoperative coagulopathy
c) dilutional thrombocytopenia
d) heparin-induced thrombocytopenia
e) hypothermia-induced thrombocytopenia
21. A cirrhotic patient with intractable ascites has a
peritoneovenous shunt placed and is noted to begin
oozing from many sites including his incisions. What
is the best explanation for this?

a) disseminated intravascular coagulation


b) unrecognized preoperative coagulopathy
c) dilutional thrombocytopenia
d) heparin-induced thrombocytopenia
e) hypothermia-induced thrombocytopenia
22. A 70 year-old male is 3 months postoperative from an
aortobifemoral revascularization for aortoiliac occlusive disease.
He returns to the office complaining of pain and swelling in his right
groin. On physical examination, the right groin appears fluctuant
and tender. It drains 10 cc of clear, non-foul smelling material.
What bacterial species is most likely to cause this infection?

a) Clostridium perfringens
b) Staphylococcus aureus
c) Staphylococcus epidermidis
d) Streptococcus pneumoniae
e) Group A Streptococci
22. A 70 year-old male is 3 months postoperative from an
aortobifemoral revascularization for aortoiliac occlusive disease.
He returns to the office complaining of pain and swelling in his right
groin. On physical examination, the right groin appears fluctuant
and tender. It drains 10 cc of clear, non-foul smelling material.
What bacterial species is most likely to cause this infection?

a) Clostridium perfringens
b) Staphylococcus aureus
c) Staphylococcus epidermidis
d) Streptococcus pneumoniae
e) Group A Streptococci
23. From a hematologic viewpoint, the loss of which
immune function performed by the spleen makes
patients susceptible to overwhelming post-splenectomy
infection (OPSI)?

a) loss of IgA
b) loss of IgM
c) loss of IgG
d) loss of IgE
e) loss of IgD
23. From a hematologic viewpoint, the loss of which
immune function performed by the spleen makes
patients susceptible to overwhelming post-splenectomy
infection (OPSI)?

a) loss of IgA
b) loss of IgM
c) loss of IgG
d) loss of IgE
e) loss of IgD
24. Which of the following statements is true concerning
intraperitoneal bacteria?

a) They are absorbed by the diaphragmatic lymphatics.


b) It is normal to have bacteria in the peritoneal cavity.
c) They are cleared by intraperitoneal lymphocytes.
d) Characteristically, secondary peritonitis is caused by a single
organism and it is usually a gram negative infection.
e) Secondary peritonitis is usually a blood borne infection from
a primary pulmonary focus.
24. Which of the following statements is true concerning
intraperitoneal bacteria?

a) They are absorbed by the diaphragmatic lymphatics.


b) It is normal to have bacteria in the peritoneal cavity.
c) They are cleared by intraperitoneal lymphocytes.
d) Characteristically, secondary peritonitis is caused by a single
organism and it is usually a gram negative infection.
e) Secondary peritonitis is usually a blood borne infection from
a primary pulmonary focus.
25. An axial groin flap is based on the blood supply of
which artery?

a) superficial epigastric
b) superficial circumflex
c) deep circumflex
d) pudendal
e) deep inferior epigastric
25. An axial groin flap is based on the blood supply of
which artery?

a) superficial epigastric
b) superficial circumflex
c) deep circumflex
d) pudendal
e) deep inferior epigastric
26. What is the most common type of bacteria in the
colon?

a) Escherichia coli
b) Lactobacillus spp.
c) Peptostreptococcus spp.
d) Peptococcus spp.
e) Bacteroides spp.
26. What is the most common type of bacteria in the
colon?

a) Escherichia coli
b) Lactobacillus spp.
c) Peptostreptococcus spp.
d) Peptococcus spp.
e) Bacteroides spp.
27. A patient undergoes total thyroidectomy with right neck
dissection. The surgeon injures the R hypoglossal nerve, the R
superior laryngeal nerve, and the L recurrent laryngeal nerve.
What deficits would you expect to find postoperatively?

Cord Tongue deviation


a) L cord tensed L
b) L cord tensed R
c) R cord tensed L
d) R cord tensed R
e) both cords tensed L
27. A patient undergoes total thyroidectomy with right neck
dissection. The surgeon injures the R hypoglossal nerve, the R
superior laryngeal nerve, and the L recurrent laryngeal nerve.
What deficits would you expect to find postoperatively?

Cord Tongue deviation


a) L cord tensed L
b) L cord tensed R
c) R cord tensed L
d) R cord tensed R
e) both cords tensed L
28. Which of the following sarcomas has the greatest
tendency to metastasize to regional lymph nodes?

a) liposarcoma
b) malignant fibrous histiocytoma
c) schwannoma
d) epitheloid sarcoma
e) dermoid sarcoma
28. Which of the following sarcomas has the greatest
tendency to metastasize to regional lymph nodes?

a) liposarcoma
b) malignant fibrous histiocytoma
c) schwannoma
d) epitheloid sarcoma
e) dermoid sarcoma
29. A 78 year-old female has a node-negative breast
cancer with positive ER/PR. Her stage is T1C N0 M0. What
is the most appropriate therapy?

a) tamoxifen for 1 year


b) tamoxifen for 5 years
c) CMF followed by tamoxifen for 1 year
d) CMF followed by tamoxifen for 5 years
e) no further therapy
29. A 78 year-old female has a node-negative breast
cancer with positive ER/PR. Her stage is T1C N0 M0. What
is the most appropriate therapy?

a) tamoxifen for 1 year


b) tamoxifen for 5 years
c) CMF followed by tamoxifen for 1 year
d) CMF followed by tamoxifen for 5 years
e) no further therapy
30. Where is the most common location to find an
accessory spleen?

a) gastrocolic ligament
b) splenocolic ligament
c) gastrosplenic ligament
d) greater omentum
e) splenic hilum
30. Where is the most common location to find an
accessory spleen?

a) gastrocolic ligament
b) splenocolic ligament
c) gastrosplenic ligament
d) greater omentum
e) splenic hilum
31. A patient undergoes laparoscopic cholecystectomy and 5
days later the pathology report reveals that there was a 0.9
mm foci of adenocarcinoma limited to the mucosa. What is
the next appropriate step in management?

a) observation
b) wedge resection of the liver of segment V with
lymphadenectomy
c) radiation
d) chemotherapy
e) hepatic infuse-aid pump
31. A patient undergoes laparoscopic cholecystectomy and 5
days later the pathology report reveals that there was a 0.9
mm foci of adenocarcinoma limited to the mucosa. What is
the next appropriate step in management?

a) observation
b) wedge resection of the liver of segment V with
lymphadenectomy
c) radiation
d) chemotherapy
e) hepatic infuse-aid pump
32. Which of the following is an absolute contraindication
to the use of an intra-aortic balloon pump (IABP)?

a) mitral stenosis
b) pulmonic stenosis
c) aortic stenosis
d) aortic insufficiency
e) mitral insufficiency
32. Which of the following is an absolute contraindication
to the use of an intra-aortic balloon pump (IABP)?

a) mitral stenosis
b) pulmonic stenosis
c) aortic stenosis
d) aortic insufficiency
e) mitral insufficiency
33. A 21 year-old jogger develops claudication after 100 yards. On
physical examination, he has normal pulses in his lower extremities
including the affected leg. What is the next most appropriate step
in management?

a) place the patient on Trental


b) obtain ABIs
c) arteriogram looking for deviation of the popliteal artery since this
most likely represents popliteal entrapment syndrome
d) graded exercise regimen
e) reassurance and observation
33. A 21 year-old jogger develops claudication after 100 yards. On
physical examination, he has normal pulses in his lower extremities
including the affected leg. What is the next most appropriate step
in management?

a) place the patient on Trental


b) obtain ABIs
c) arteriogram looking for deviation of the popliteal artery since
this most likely represents popliteal entrapment syndrome
d) graded exercise regimen
e) reassurance and observation
34. Which of the following drugs increases the cardiac
output and increases the systemic vascular resistance?

a) isoproterenol
b) milrinone
c) amrinone
d) norepinephrine
e) dobutamine
34. Which of the following drugs increases the cardiac
output and increases the systemic vascular resistance?

a) isoproterenol
b) milrinone
c) amrinone
d) norepinephrine
e) dobutamine
35. What period of time in wound healing is
synthesis of collagen at its maximum?

a) 0 –1 week
b) 3 – 6 weeks
c) 3 – 6 months
d) 6 – 9 months
e) 9 – 12 months
35. What period of time in wound healing is
synthesis of collagen at its maximum?

a) 0 –1 week
b) 3 – 6 weeks
c) 3 – 6 months
d) 6 – 9 months
e) 9 – 12 months
36. Which statement concerning the relationships in the
hepatoduodenal ligament is true?

a) The portal vein is posterior to the CBD and hepatic artery; CBD is
medial to the hepatic artery.
b) The portal vein is posterior to the CBD and hepatic artery; CBD is
lateral to the hepatic artery.
c) The portal vein is anterior to the CBD and hepatic artery.
d) The portal vein is lateral to the CBD and both are anterior to the
hepatic artery.
e) The portal vein is medial to the CBD and both are anterior to the
hepatic artery.
36. Which statement concerning the relationships in the
hepatoduodenal ligament is true?

a) The portal vein is posterior to the CBD and hepatic artery; CBD is
medial to the hepatic artery.
b) The portal vein is posterior to the CBD and hepatic artery; CBD is
lateral to the hepatic artery.
c) The portal vein is anterior to the CBD and hepatic artery.
d) The portal vein is lateral to the CBD and both are anterior to the
hepatic artery.
e) The portal vein is medial to the CBD and both are anterior to the
hepatic artery.
37. A patient who has suffered multi-organ trauma is taken back to
the operating room to repair intra-abdominal injuries. He receives
multiple transfusions in the OR and it is noted that there is oozing
from the retroperitoneum where the R colon was mobilized. After
packing and allowing adequate time to stop, it is still noted to be
oozing. Intra-operatively his coags and platelets are checked and
found to be within normal limits. What should be done at this
time?
a) administer platelets
b) administer FFP
c) administer cryoprecipitate
d) administer DDAVP
e) continue to cauterize the area of bleeding
37. A patient who has suffered multi-organ trauma is taken back to
the operating room to repair intra-abdominal injuries. He receives
multiple transfusions in the OR and it is noted that there is oozing
from the retroperitoneum where the R colon was mobilized. After
packing and allowing adequate time to stop, it is still noted to be
oozing. Intra-operatively his coags and platelets are checked and
found to be within normal limits. What should be done at this
time?
a) administer platelets
b) administer FFP
c) administer cryoprecipitate
d) administer DDAVP
e) continue to cauterize the area of bleeding
38. Which of the following tends to make bile
lithogenic?

a) oral chenodeoxylate
b) resection of the jejunum
c) resection of the distal stomach
d) resection of the terminal ileum
e) resection of the colon
38. Which of the following tends to make bile
lithogenic?

a) oral chenodeoxylate
b) resection of the jejunum
c) resection of the distal stomach
d) resection of the terminal ileum
e) resection of the colon
39. What is the most frequent complication after a level I
and II axillary lymph node dissection?

a) seroma
b) hematoma
c) winged scapula
d) loss of sensation on the posterior aspect of the upper
arm
e) lymphedema
39. What is the most frequent complication after a level I
and II axillary lymph node dissection?

a) seroma
b) hematoma
c) winged scapula
d) loss of sensation on the posterior aspect of the upper
arm
e) lymphedema
40. A patient sustained a blunt MVA and was noted to have a
liver injury which was managed non-operatively. Two months
later, he develops hematemesis and melena. EGD and
colonoscopy are noncontributory. What is the next most
appropriate step in management?

a) vagotomy and antrectomy


b) vagotomy and pyloroplasty
c) exploratory laparotomy and liver resection
d) angiography and embolization
e) observation
40. A patient sustained a blunt MVA and was noted to have a
liver injury which was managed non-operatively. Two months
later, he develops hematemesis and melena. EGD and
colonoscopy are noncontributory. What is the next most
appropriate step in management?

a) vagotomy and antrectomy


b) vagotomy and pyloroplasty
c) exploratory laparotomy and liver resection
d) angiography and embolization
e) observation
41. A patient with Crohn’s disease is operated on for
symptoms of bowel obstruction. At the time of operation, it
is noted that there are two areas of stricture in the small
intestine and severe ileocecal Crohn’s disease. What is the
next most appropriate step in management?

a) ileocecectomy and two enterectomies


b) ileocecectomy and two stricturoplasties
c) ileocecectomy and bypass of the strictures
d) three stricturoplasties
e) enterectomy encompassing all diseased segments
41. A patient with Crohn’s disease is operated on for
symptoms of bowel obstruction. At the time of operation, it
is noted that there are two areas of stricture in the small
intestine and severe ileocecal Crohn’s disease. What is the
next most appropriate step in management?

a) ileocecectomy and two enterectomies


b) ileocecectomy and two stricturoplasties
c) ileocecectomy and bypass of the strictures
d) three stricturoplasties
e) enterectomy encompassing all diseased segments
42. A woman presents after blunt MVA. Abdominal CT shows
a 3 cm right adrenal mass. All endocrinologic studies are
negative and she has no identifiable signs of hormone excess.
What is the next most appropriate step in management?

a) transperitoneal adrenalectomy
b) extraperitoneal adrenalectomy
c) MRI of the abdomen
d) selective venous sampling
e) observation and repeat CT scan in 6 months
42. A woman presents after blunt MVA. Abdominal CT shows
a 3 cm right adrenal mass. All endocrinologic studies are
negative and she has no identifiable signs of hormone excess.
What is the next most appropriate step in management?

a) transperitoneal adrenalectomy
b) extraperitoneal adrenalectomy
c) MRI of the abdomen
d) selective venous sampling
e) observation and repeat CT scan in 6 months
43. A 25 year-old female undergoes resection of one foot
of small intestine including her terminal ileum. Several
months later, she presents with renal stones. What kind
of renal stones did she most likely develop?

a) calcium phosphate
b) calcium oxalate
c) struvite
d) uric acid
e) magnesium ammonium phosphate
43. A 25 year-old female undergoes resection of one foot
of small intestine including her terminal ileum. Several
months later, she presents with renal stones. What kind
of renal stones did she most likely develop?

a) calcium phosphate
b) calcium oxalate
c) struvite
d) uric acid
e) magnesium ammonium phosphate
44. A patient with a lateral neck mass undergoes FNA which
reveals squamous cell carcinoma. Further work-up is
negative for the primary tumor. The patient’s Epstein-Barr
virus (EBV) titers are noted to be high. Where is the most
likely source of the primary?

a) tonsillar pillar
b) hypopharnyx
c) nasopharnyx
d) floor of mouth
e) pyriform sinus
44. A patient with a lateral neck mass undergoes FNA which
reveals squamous cell carcinoma. Further work-up is
negative for the primary tumor. The patient’s Epstein-Barr
virus (EBV) titers are noted to be high. Where is the most
likely source of the primary?

a) tonsillar pillar
b) hypopharnyx
c) nasopharnyx
d) floor of mouth
e) pyriform sinus
45. von Willebrand factor (vWF) is synthesized in
which of the following?

a) platelet
b) macrophage
c) endothelial cell
d) RBC
e) hepatocyte
45. von Willebrand factor (vWF) is synthesized in
which of the following?

a) platelet
b) macrophage
c) endothelial cell
d) RBC
e) hepatocyte
46. In performing a pericardiocentesis, the most
appropriate manner to ensure the needle is in the
pericardial sac is to:

a) check for pulsatile flow


b) do it under fluoroscopy
c) connect it to an EKG lead
d) check to see if the blood clots
e) never do it unless there is ultrasound guidance
available
46. In performing a pericardiocentesis, the most
appropriate manner to ensure the needle is in the
pericardial sac is to:

a) check for pulsatile flow


b) do it under fluoroscopy
c) connect it to an EKG lead
d) check to see if the blood clots
e) never do it unless there is ultrasound guidance
available
47. The initial site of blood cell production in the
embryo occurs in which of the following?

a) liver
b) yolk sac
c) bone marrow
d) spleen
e) thymus
47. The initial site of blood cell production in the
embryo occurs in which of the following?

a) liver
b) yolk sac
c) bone marrow
d) spleen
e) thymus
48. Which of the following electrolyte abnormalities
exacerbates digitalis toxicity and also maintains
metabolic alkalosis?

a) hypernatremia
b) hyponatremia
c) hyperkalemia
d) hypokalemia
e) hypercalcemia
48. Which of the following electrolyte abnormalities
exacerbates digitalis toxicity and also maintains
metabolic alkalosis?

a) hypernatremia
b) hyponatremia
c) hyperkalemia
d) hypokalemia
e) hypercalcemia
49. In a patient with renal failure, which neuromuscular
blocking agent would you give in order to maintain
constant plasma levels?

a) succinylcholine
b) D-tubocurarine
c) pancuronium
d) atracurium
e) vecuronium
49. In a patient with renal failure, which neuromuscular
blocking agent would you give in order to maintain
constant plasma levels?

a) succinylcholine
b) D-tubocurarine
c) pancuronium
d) atracurium
e) vecuronium
50. Which of the following amino acids is an essential
amino acid?

a) tyrosine
b) proline
c) glutamine
d) valine
e) alanine
50. Which of the following amino acids is an essential
amino acid?

a) tyrosine
b) proline
c) glutamine
d) valine
e) alanine
51. Patients with kidney transplants have an increased
incidence of developing cancer. Which of the following
cancers has the highest incidence?

a) Kaposi’s sarcoma
b) lymphomas
c) skin cancer
d) lip cancer
e) vulvar cancer
51. Patients with kidney transplants have an increased
incidence of developing cancer. Which of the following
cancers has the highest incidence?

a) Kaposi’s sarcoma
b) lymphomas
c) skin cancer
d) lip cancer
e) vulvar cancer
52. An otherwise healthy 22 year-old pregnant female
who develops C. difficile colitis after antibiotic treatment
should receive which of the following regimens?

a) PO flagyl
b) PO vancomycin
c) IV flagyl
d) IV vancomycin
e) No treatment in pregnant patients
52. An otherwise healthy 22 year-old pregnant female
who develops C. difficile colitis after antibiotic treatment
should receive which of the following regimens?

a) PO flagyl
b) PO vancomycin
c) IV flagyl
d) IV vancomycin
e) No treatment in pregnant patients
53. What is the initial management of a patient with
a 5 cm amebic liver abscess?

a) metronidazole
b) percutaneous drainage
c) open surgical drainage
d) mebendazole
e) ampicillin, gentamicin, and flagyl
53. What is the initial management of a patient with
a 5 cm amebic liver abscess?

a) metronidazole
b) percutaneous drainage
c) open surgical drainage
d) mebendazole
e) ampicillin, gentamicin, and flagyl
54. Which of the following equations is correct for
the determination of oxygen delivery?

a) [SaO2 x Hgb x 1.34] + PaO2 (0.003)


b) [PaO2 x Hgb x 1.34] + SaO2 (0.003)
c) [PaO2/Hgb x 1.34] + SaO2 (0.003) x CO
d) [SaO2 x Hgb x 1.34] + PaO2 (0.003) x CO
e) [SaO2 x PaO2 x 0.003] + 1.34 (Hgb) x CO
54. Which of the following equations is correct for
the determination of oxygen delivery?

a) [SaO2 x Hgb x 1.34] + PaO2 (0.003)


b) [PaO2 x Hgb x 1.34] + SaO2 (0.003)
c) [PaO2/Hgb x 1.34] + SaO2 (0.003) x CO
d) [SaO2 x Hgb x 1.34] + PaO2 (0.003) x CO
e) [SaO2 x PaO2 x 0.003] + 1.34 (Hgb) x CO
55. Which of the following is true concerning meperidine
(demerol)?

a) contraindicated in patients who are taking MAO


inhibitors
b) requires less naloxone to reverse than morphine
c) preferable to use in renal failure over morphine
d) is an agonist/antagonist drug
e) none of the statements are true
55. Which of the following is true concerning meperidine
(demerol)?

a) contraindicated in patients who are taking MAO


inhibitors
b) requires less naloxone to reverse than morphine
c) preferable to use in renal failure over morphine
d) is an agonist/antagonist drug
e) none of the statements are true
56. When looking at carotid endarterectomies
statistically, which of the following could be considered a
continuous variable?

a) smoking
b) ulcerated plaque
c) race
d) TIAs
e) blood pressure
56. When looking at carotid endarterectomies
statistically, which of the following could be considered a
continuous variable?

a) smoking
b) ulcerated plaque
c) race
d) TIAs
e) blood pressure
57. In which portion of the pancreas are insulinomas
predominantly found?

a) head
b) neck
c) body
d) tail
e) equal frequency throughout pancreas
57. In which portion of the pancreas are insulinomas
predominantly found?

a) head
b) neck
c) body
d) tail
e) equal frequency throughout pancreas
58. A 45 year-old female is noted to have inflammatory
breast cancer without ulceration. Which of the following
is the most appropriate initial treatment?

a) “toilet mastectomy”
b) modified radical mastectomy
c) radical mastectomy
d) neoadjuvant chemotherapy
e) neoadjuvant radiation therapy
58. A 45 year-old female is noted to have inflammatory
breast cancer without ulceration. Which of the following
is the most appropriate initial treatment?

a) “toilet mastectomy”
b) modified radical mastectomy
c) radical mastectomy
d) neoadjuvant chemotherapy
e) neoadjuvant radiation therapy
59. Which of the following is true concerning splenic artery
aneurysms?

a) They have no predilection for either sex.


b) When symptomatic, these aneurysms can often be
managed conservatively.
c) Women of childbearing age are at increased risk of rupture
and should have elective repair.
d) Surgical treatment should never include splenectomy.
e) Proximal aneurysms are best managed by exclusion and
vascular reconstruction in order for splenic salvage.
59. Which of the following is true concerning splenic artery
aneurysms?

a) They have no predilection for either sex.


b) When symptomatic, these aneurysms can often be
managed conservatively.
c) Women of childbearing age are at increased risk of rupture
and should have elective repair.
d) Surgical treatment should never include splenectomy.
e) Proximal aneurysms are best managed by exclusion and
vascular reconstruction in order for splenic salvage.
60. Which of the following is true concerning the thyroid gland?

a) Thyroid tissue found in the lateral neck compartments is known


as lateral aberrant thyroid tissue and is an embryologic variation.
b) Any thyroid tissue found in lymph node tissue in the neck which is
extrathyroid represents metastatic deposits from thyroid
carcinoma.
c) Lingual thyroid results from a failure of the third pharyngeal
pouch to descend in a normal fashion.
d) Lingual thyroid usually occurs in conjunction with normal
anatomic thyroid tissue.
e) Lingual thyroid is best treated by surgical excision.
60. Which of the following is true concerning the thyroid gland?

a) Thyroid tissue found in the lateral neck compartments is known


as lateral aberrant thyroid tissue and is an embryologic variation.
b) Any thyroid tissue found in lymph node tissue in the neck which
is extrathyroid represents metastatic deposits from thyroid
carcinoma.
c) Lingual thyroid results from a failure of the third pharyngeal
pouch to descend in a normal fashion.
d) Lingual thyroid usually occurs in conjunction with normal
anatomic thyroid tissue.
e) Lingual thyroid is best treated by surgical excision.
61. Which of the following is the initial site of spread
of ovarian cancer?

a) bone
b) liver
c) rectum
d) bladder
e) peritoneum
61. Which of the following is the initial site of spread
of ovarian cancer?

a) bone
b) liver
c) rectum
d) bladder
e) peritoneum
62. Resection of the terminal ileum results in which
one of the following?

a) iron-deficiency anemia
b) sideroblastic anemia
c) macrocytic anemia
d) pernicious anemia
e) calcium phosphate renal stones
62. Resection of the terminal ileum results in which
one of the following?

a) iron-deficiency anemia
b) sideroblastic anemia
c) macrocytic anemia
d) pernicious anemia
e) calcium phosphate renal stones
63. A critically ill patient has been receiving TPN via a two week-old
central line. Her TPN is turned off so that she may be given a blood
transfusion. She receives one month-old blood through her central
venous catheter. Toward the end of her four hour transfusion, she
is noted to be hypothermic, hypotensive and comatose. What is
the most likely explanation?

a) transfusion reaction
b) hypoglycemia
c) hyperkalemia
d) candidemia
e) line sepsis with bacteremia
63. A critically ill patient has been receiving TPN via a two week-old
central line. Her TPN is turned off so that she may be given a blood
transfusion. She receives one month-old blood through her central
venous catheter. Toward the end of her four hour transfusion, she
is noted to be hypothermic, hypotensive and comatose. What is
the most likely explanation?

a) transfusion reaction
b) hypoglycemia
c) hyperkalemia
d) candidemia
e) line sepsis with bacteremia
64. What is the chance that any two siblings share
one haplotype?

a) 0%
b) 25%
c) 50%
d) 100%
e) not enough information
64. What is the chance that any two siblings share
one haplotype?

a) 0%
b) 25%
c) 50%
d) 100%
e) not enough information
65. Which of the following statements is true concerning congenital
lobar emphysema?

a) It is usually limited to the upper lobes and characterized by severe


air trapping.
b) Lung destruction associated with this condition is irreversible and
often requires pneumonectomy.
c) Treatment consists of conservative measures and rarely requires
excision of the affected lobe.
d) Most are located in the anterior mediastinum.
e) Like intralobar pulmonary sequestrations, they drain into the
azygous veins.
65. Which of the following statements is true concerning congenital
lobar emphysema?

a) It is usually limited to the upper lobes and characterized by


severe air trapping.
b) Lung destruction associated with this condition is irreversible and
often requires pneumonectomy.
c) Treatment consists of conservative measures and rarely requires
excision of the affected lobe.
d) Most are located in the anterior mediastinum.
e) Like intralobar pulmonary sequestrations, they drain into the
azygous veins.
66. T3 lesion of the colon or rectum means:

a) limited to the mucosa


b) invasion of submucosa
c) invasion of muscularis propria
d) invasion of subserosa or nonperitonealized pericolic fat
e) invasion of contiguous structures
66. T3 lesion of the colon or rectum means:

a) limited to the mucosa


b) invasion of submucosa
c) invasion of muscularis propria
d) invasion of subserosa or nonperitonealized pericolic
fat
e) invasion of contiguous structures
67. During normal activity, energy expenditure in a
healthy adult male in kcal/kg/day is approximately?

a) 15 kcal
b) 20 kcal
c) 35 kcal
d) 50 kcal
e) none of the above
67. During normal activity, energy expenditure in a
healthy adult male in kcal/kg/day is approximately?

a) 15 kcal
b) 20 kcal
c) 35 kcal
d) 50 kcal
e) none of the above
68. In the performance of a femoral hernia repair,
which of the following structures must be divided?

a) transversalis fascia
b) lacunar ligament
c) inguinal ligament
d) internal oblique muscle
e) cooper’s ligament
68. In the performance of a femoral hernia repair,
which of the following structures must be divided?

a) transversalis fascia
b) lacunar ligament
c) inguinal ligament
d) internal oblique muscle
e) cooper’s ligament
69. This is the normal ultrasound anatomy of the rectal wall.
Which of the rings represents the muscularis propria?

a) inner white line


b) first black line
c) middle white line
d) second black line
e) outer white line
69. This is the normal ultrasound anatomy of the rectal wall.
Which of the rings represents the muscularis propria?

a) inner white line


b) first black line
c) middle white line
d) second black line
e) outer white line
70. Concerning a femoropopliteal bypass: Which
structure is anterior to the graft at midthigh?

a) gracilis
b) adductor magnus
c) rectus femoris
d) femoral vein
e) deep femoral artery
70. Concerning a femoropopliteal bypass: Which
structure is anterior to the graft at midthigh?

a) gracilis
b) adductor magnus
c) rectus femoris
d) femoral vein
e) deep femoral artery
71. Which is the most common benign tumor of
the parotid?
A) Adenoid cystic
B) Acinic cell
C) Mucoepidermoid
D) Pleiomorphic adenoma
E) Warthins tumor
71. Which is the most common benign tumor of
the parotid?
A) Adenoid cystic
B) Acinic cell
C) Mucoepidermoid
D) Pleiomorphic adenoma
E) Warthins tumor
72. During the performance of a parotidectomy, you notice
facial nerve involvement by the tumor. What is the most
appropriate step in the management of the tumor.
A) Resect the superficial parotid gland and radiate the facial
nerve and deep portion of the gland.
B) Abort the procedure since the patient is incurable.
C) Resect the nerve and perform a nerve graft.
D) Resect nothing and give chemotherapy and radiation
therapy to the entire gland.
E) Resect the nerve and plan on a staged repair of the facial
nerve.
72. During the performance of a parotidectomy, you notice
facial nerve involvement by the tumor. What is the most
appropriate step in the management of the tumor.
A) Resect the superficial parotid gland and radiate the facial
nerve and deep portion of the gland.
B) Abort the procedure since the patient is incurable.
C) Resect the nerve and perform a nerve graft.
D) Resect nothing and give chemotherapy and radiation
therapy to the entire gland.
E) Resect the nerve and plan on a staged repair of the facial
nerve.
73. After a neck dissection, you notice the patient
has lost taste over the anterior ipsilateral tongue.
What nerve has most likely been injured?
A) Buccal
B) Masseter
C) Hypoglossal
D) Lingual
E) Marginal mandibular
73. After a neck dissection, you notice the patient
has lost taste over the anterior ipsilateral tongue.
What nerve has most likely been injured?
A) Buccal
B) Masseter
C) Hypoglossal
D) Lingual
E) Marginal mandibular
74. You have performed a laryngectomy/
pharyngectomy and post-operatively you notice that there is
cellulitis and purulent drainage from your neck incision. The patient is
febrile with an elevated WBC. What is the most appropriate step?

A) Open the wound in the OR and attempt suture repair of


the pharyngeal anastomosis.
B) Open the wound in the OR and obtain good drainage
without attempting to repair the pharyngeal anastomosis.
C) Start the patient on IV antibiotics only.
D) Place PEG and start IV antibiotics.
E) Place PEG and attempt suture repair of the pharyngeal
anastomosis.
74. You have performed a laryngectomy/
pharyngectomy and post-operatively you notice that there is
cellulitis and purulent drainage from your neck incision. The patient is
febrile with an elevated WBC. What is the most appropriate step?

A) Open the wound in the OR and attempt suture repair of


the pharyngeal anastomosis.
B) Open the wound in the OR and obtain good drainage
without attempting to repair the pharyngeal
anastomosis.
C) Start the patient on IV antibiotics only.
D) Place PEG and start IV antibiotics.
E) Place PEG and attempt suture repair of the pharyngeal
anastomosis.
75. A patient presents with a neck mass and FNA shows
squamous cell carcinoma. The patient has extremely
high EBV titers. Further work-up does not show any
primary source of tumor. What is the most likely source
of the primary?
A) Nasopharynx
B) Tonsillar pillar
C) Hard Palate
D) Soft Palate
E) Epiglottis
75. A patient presents with a neck mass and FNA shows
squamous cell carcinoma. The patient has extremely
high EBV titers. Further work-up does not show any
primary source of tumor. What is the most likely source
of the primary?
A) Nasopharynx
B) Tonsillar pillar
C) Hard Palate
D) Soft Palate
E) Epiglottis
76. A patient presents with an inguinal lymph node which is biopsied
and is positive for melanoma. Further work-up and physical exam does
not reveal a primary source of the tumor. What is the next most
appropriate step in management?

A) Sentinel lymph node biopsy of the inguinal lymph


node basin.
B) Sentinel lymph node biopsy of the contralateral
nodal basin.
C) Ipsilateral inguinal lymphadenectomy.
D) IFN-alpha if this is Stage 3 melanoma.
E) Il-2 treatment if this is Stage 4 melanoma.
76. A patient presents with an inguinal lymph node which is biopsied
and is positive for melanoma. Further work-up and physical exam does
not reveal a primary source of the tumor. What is the next most
appropriate step in management?

A) Sentinel lymph node biopsy of the inguinal lymph


node basin.
B) Sentinel lymph node biopsy of the contralateral
nodal basin.
C) Ipsilateral inguinal lymphadenectomy.
D) IFN-alpha if this is Stage 3 melanoma.
E) Il-2 treatment if this is Stage 4 melanoma.
77. A woman on mammography is noted to have a cluster
of microcalcifications. Excisional biopsy shows
atypical hyperplasia with LCIS. The margins are
negative. What is the next most appropriate step in
management?
A) Modified radical mastectomy
B) Lumpectomy alone
C) Lumpectomy and radiation
D) Lumpectomy and Tamoxifen
E) Observe
77. A woman on mammography is noted to have a cluster
of microcalcifications. Excisional biopsy shows
atypical hyperplasia with LCIS. The margins are
negative. What is the next most appropriate step in
management?
A) Modified radical mastectomy
B) Lumpectomy alone
C) Lumpectomy and radiation
D) Lumpectomy and Tamoxifen
E) Observe
78. A woman presents with a scaly rash on her nipple which is
tender and itches. Biopsy shows Paget’s disease of the
breast. What would you expect of a mass on histology if
found in the excisional biopsy specimen?
A) Invasive lobular carcinoma
B) Invasive ductal carcinoma
C) Tubular carcinoma of the breast
D) Papillary carcinoma of the breast
E) Eczema
78. A woman presents with a scaly rash on her nipple which is
tender and itches. Biopsy shows Paget’s disease of the
breast. What would you expect of a mass on histology if
found in the excisional biopsy specimen?
A) Invasive lobular carcinoma
B) Invasive ductal carcinoma
C) Tubular carcinoma of the breast
D) Papillary carcinoma of the breast
E) Eczema
79. A woman has an 8 cm cystosarcoma phyllodes tumor of
the breast and is small-breasted. What is the most
appropriate management?
A) Quandrantectomy
B) Wide local excision with 2 cm margins
C) Radiation and Tamoxifen
D) Total mastectomy
E) None of the above
79. A woman has an 8 cm cystosarcoma phyllodes tumor of
the breast and is small-breasted. What is the most
appropriate management?
A) Quandrantectomy
B) Wide local excision with 2 cm margins
C) Radiation and Tamoxifen
D) Total mastectomy
E) None of the above
80. A woman presents with peau-d’ orange in the lower half
of her breast. Her breast is markedly abnormal on
examination. There is no palpable lymphadenopathy.
What is the next most appropriate step in management?
A) Total mastectomy
B) Toilet mastectomy
C) Modified radical mastectomy
D) Radical mastectomy
E) Chemotherapy
80. A woman presents with peau-d’ orange in the lower half
of her breast. Her breast is markedly abnormal on
examination. There is no palpable lymphadenopathy.
What is the next most appropriate step in management?
A) Total mastectomy
B) Toilet mastectomy
C) Modified radical mastectomy
D) Radical mastectomy
E) Chemotherapy
81. A woman has microcalcifications on mammogram
and biopsy shows DCIS. Which algorithim is
associated with the lowest rate of recurrence?
A) Lumpectomy and RT
B) Lumpectomy, RT and axillary dissection
C) Lumpectomy, RT and Tamoxifen
D) Lumpectomy and Tamoxifen
E) Total mastectomy
81. A woman has microcalcifications on mammogram
and biopsy shows DCIS. Which algorithim is
associated with the lowest rate of recurrence?
A) Lumpectomy and RT
B) Lumpectomy, RT and axillary dissection
C) Lumpectomy, RT and Tamoxifen
D) Lumpectomy and Tamoxifen
E) Total mastectomy
82. A 78 y.o. woman has a node-negative breast
cancer with positive ER/PR. Her stage is T1CN0M0.
What is the most appropriate therapy?
A) Tamoxifen for 1 year
B) Tamoxifen for 5 years
C) CMF followed by Tamoxifen for 1 year
D) CMF followed by Tamoixfen for 5 years
E) None of the above
82. A 78 y.o. woman has a node-negative breast
cancer with positive ER/PR. Her stage is T1CN0M0.
What is the most appropriate therapy?
A) Tamoxifen for 1 year
B) Tamoxifen for 5 years
C) CMF followed by Tamoxifen for 1 year
D) CMF followed by Tamoixfen for 5 years
E) None of the above
83. Which drug increases the cardiac output and
increases the systemic vascular resistance?
A) Isoproterenol
B) Milrinone
C) Amrinone
D) Norepinephrine
E) Dobutamine
83. Which drug increases the cardiac output and
increases the systemic vascular resistance?
A) Isoproterenol
B) Milrinone
C) Amrinone
D) Norepinephrine
E) Dobutamine
84. In a patient who is POD #6 from an abdominal
surgery, what is the most likely cause of renal and
liver failure?
A) Infection
B) CHF
C) Drug toxicity
D) Abdominal compartment syndrome
E) Anesthetic reaction and “shock”liver
84. In a patient who is POD #6 from an abdominal
surgery, what is the most likely cause of renal and
liver failure?
A) Infection
B) CHF
C) Drug toxicity
D) Abdominal compartment syndrome
E) Anesthetic reaction and “shock”liver
85. A patient is given Cefotan for antibiotic prophylaxis and has an
uneventful operation. On POD # 3, the patient develops
jaundice and has an elevated bilirubin and dark urine. What has
most likely occurred?

A) Liver failure secondary to anesthetics given at


surgery.
B) Hemolytic immune reaction to Cefotan.
C) Hemolytic immune reaction to anesthetics.
D) TTP secondary to Cefotan.
E) Development of idiopathic hemolytic uremia
secondary to Cefotan.
85. A patient is given Cefotan for antibiotic prophylaxis and has an
uneventful operation. On POD # 3, the patient develops
jaundice and has an elevated bilirubin and dark urine. What has
most likely occurred?

A) Liver failure secondary to anesthetics given at


surgery.
B) Hemolytic immune reaction to Cefotan.
C) Hemolytic immune reaction to anesthetics.
D) TTP secondary to Cefotan.
E) Development of idiopathic hemolytic uremia
secondary to Cefotan.
86. A 68 y.o. man who had a recent MI develops massive
UGIB. It is noted to be coming from esophageal
varices. What is the best agent to treat this patient
with?
A) Somatostatin
B) Nitrates
C) Vasopressin
D) Dopamine
E) Dobutamine
86. A 68 y.o. man who had a recent MI develops massive
UGIB. It is noted to be coming from esophageal
varices. What is the best agent to treat this patient
with?
A) Somatostatin
B) Nitrates
C) Vasopressin
D) Dopamine
E) Dobutamine
87. Which is an absolute contraindication to the use
of the intra-aortic balloon pump (IABP)?
A) Aortic insufficiency
B) Aortic stenosis
C) Mitral insufficiency
D) Mitral stenosis
E) Pulmonary stenosis
87. Which is an absolute contraindication to the use
of the intra-aortic balloon pump (IABP)?
A) Aortic insufficiency
B) Aortic stenosis
C) Mitral insufficiency
D) Mitral stenosis
E) Pulmonary stenosis
88. A patient is on TPN and has an RQ of 1.05. He is also
hypercarbic and has a high respiratory rate of 36
bpm. What is most likely the explanation for this
occurrence?
A) excessive lipid ingestion
B) deficient lipid ingestion
C) excessive carbohydrate administration
D) insufficient carbohydrate administration
E) lipogenesis
88. A patient is on TPN and has an RQ of 1.05. He is also
hypercarbic and has a high respiratory rate of 36
bpm. What is most likely the explanation for this
occurrence?
A) excessive lipid ingestion
B) deficient lipid ingestion
C) excessive carbohydrate administration
D) insufficient carbohydrate administration
E) lipogenesis
89. Vinyl chloride is associated with which
malignancy?
A) Small cell carcinoma of the lung
B) Mesothelioma
C) Struma ovarii
D) Hemangiosarcoma
E) Angiosarcoma of the liver
89. Vinyl chloride is associated with which
malignancy?
A) Small cell carcinoma of the lung
B) Mesothelioma
C) Struma ovarii
D) Hemangiosarcoma
E) Angiosarcoma of the liver
90. Which of the following is the most appropriate candidate for gastric
bypass?
A) 70 y.o. healthy woman with no medical problems and BMI = 42.
B) 35 y.o. woman with diabetes, severe OSA, HTN, high chol with
BMI = 34.
C) 42 y.o. man s/p recent MI 2 months ago with diabetes, high chol
and BMI = 45.
D) 55 y.o. woman on prednisone for severe asthma and BMI = 44.
E) 47 y.o woman with HTN, diabetes, severe OSA, arthritis and GERD
with BMI = 36.
90. Which of the following is the most appropriate candidate for gastric
bypass?
A) 70 y.o. healthy woman with no medical problems and BMI = 42.
B) 35 y.o. woman with diabetes, severe OSA, HTN, high chol with
BMI = 34.
C) 42 y.o. man s/p recent MI 2 months ago with diabetes, high chol
and BMI = 45.
D) 55 y.o. woman on prednisone for severe asthma and BMI = 44.
E) 47 y.o woman with HTN, diabetes, severe OSA, arthritis and
GERD with BMI = 36.
91. What is the next appropriate step in the management of a
21 y.o woman with a lateral anal fissure?
A) Lateral internal sphincterotomy.
B) Wide incision and drainage.
C) Wide incision and drainage with seton placement.
D) Seton placement without I & D.
E) Barium enema.
91. What is the next appropriate step in the management of a
21 y.o woman with a lateral anal fissure?
A) Lateral internal sphincterotomy.
B) Wide incision and drainage.
C) Wide incision and drainage with seton placement.
D) Seton placement without I & D.
E) Barium enema.

You might also like